Vous êtes sur la page 1sur 195

Get to the heart of English language grammar in easy way

1 of 195

PART OF SPEECH

Part of speech: words divided into different classes according to their usage. In
other words, according to the work they do in a sentence.
The Part of speech is eight in number: 1) Noun
2) Pronoun
3) Verb
4) Adjective
5) Adverb 6) Preposition 7) Interjection. 8) conjunction.
1) The NOUN : can be:
The name of a person (like, Nabeel Amer)
A job title(like doctor)
The name of a thing (like, Radio)
The name of place (Like, London)
The name of quality (like, courage)
The action (like, laughter/ laughing)
The subject of the verb ( Osama Nabeel Amer is a doctor)
The direct object (He helped me)
The indirect object (I gave him a gift)
The object of preposition (I read about it in the newspaper)
The complete of be (he is our guest).
Adverb of time (tomorrow is a holiday)
A) Subject: is the doer who performs the action and starts from the beginning
of the sentence until the beginning of the verb like:
A noun like {Osama or Maram} (proper noun)
A pronoun like { he or she}
A noun phrase { the old building}
**(Note) The subject of a sentence can be a hidden subject or one word
or more and usually comes first, but occasionally it is put
after the predicate; as in,
Be calm. (The subject is hidden) =(you be calm)
Ali is a student. (One subject only)
Ibraheem Al-Shami is my best friend. (more than one word)
My friend Osamah AL-Kibsi is a kind person. (phrase)
Here comes the bus. (at the end)

A n Object : Is the person or thing that is affected by the action and it


can be:.
Direct object (like, I played basketball.)
Indirect object (like, I bought him a bike) = (I bought a bike for him)
Object of preposition (like, I live in Sanaa)
Noun phrase (like, I met him)
The cost of this book is to pray for me you have the permission to print it or do whatever you want

Get to the heart of English language grammar in easy way

2 of 195

1-1)Kinds of noun:
a) Proper noun: is the name of a particular person or place and always
begin with a capital letter.
Example:
Osama is a good teacher.
Faheem Al-Hamadi is a good accountant.
I live in Sanaa.
b) Common noun: is a name given to every person or thing or place of the
same class or kind.
Example:
That man is a good teacher.
That girl is a polite student.
Sanaa is a good city.
(NOTE) proper noun are sometimes used as a common noun; as,
He is the Shakespeare of his country. (=The greatest dramatist)
A-1) common noun: consists of two kinds:
Collective nouns: are names of a number of persons or things taken together
and spoken of as one whole; as, (crowd, mob, team, flock, army, family,
nation parliament, committee, herd, etc)
2)An abstract nouns: are usually the name of a quality, action or state considered
apart from the object to which it belongs; as,
1) Quality from the adjective; as, (goodness, kindness, darkness,
etc)
2) Action from the verb; as, (laugher, theft, movement, judgment, etc)
3) State from common noun; as, (childhood, silvery, youth, sickness,
sleep, etc)
Some words can be either a noun or a verb. We can tell the difference from the way
they are stressed or pronounced:
Nouns and Verbs distinguished by stress; as,
NOUN `Discount `entrance- `export- `import- `object- `present`progress.
VERB Dis`count en`trance- ex`port- im`port- ob`ject- pre`sentpro`gress.
By pronunciation; as,
/S/ , /f/ , / / abuse- advice- house- use- belief- proof- shelf- teeth.
/z/ , /v/ ,// abuse- advise- house- use- believe- prove- shelve, teethe
Noun and Verb with same spelling and pronunciation:
The cost of this book is to pray for me you have the permission to print it or do whatever you want

Get to the heart of English language grammar in easy way

3 of 195

Answer- change- dream- end- hope- offer- trouble


1) 2) SINGULAR AND PLURAL NOUNS:
HOW TO FORM THE PLURALR:

1) When we want to change a word from singular to a plural, most


of the time we should add (s) to the end of it:
a) Book
books
b) car
cars
2) Nouns ending in ( s, sh, ch, *o, x, ) the plural end by adding (es):
c) Class
classes
d) brush
brushes
e) Branch
branches f) tax
taxes
g) Cargo
cargoes
h) echo
echoes
3) But Some nouns end in ( o ) we add only ( s );
i) Kilo
kilos
j) photo
photos
4) Nouns ending in (consonant+ y ) we should change it into (ies);
k) Baby
babies l) city
cities m) lady
ladies
5) Nouns ending in (*f/*fe) we should change it to (ves);
n) Thief
thieves o) knife
knives
6) But some nouns ending in ( f/fe) add only (s);
Chief
Safe

Chiefs
Safes

Gulf
Proof

Gulfs
Proofs

7) A few nouns form their plural by changing the inside vowel;


Singular
Man
Foot
Goose
Louse

Plural
Men
Feet
Geese
Lice

Singular
Woman
Tooth
Mouse
Client

Plural
Women
Teeth
Mice
Clientele

8) Some nouns formed by adding (en) to the end:


q) Child
children p) ox
oxen
9) Some nouns are used only in the plural, such as:
The cost of this book is to pray for me you have the permission to print it or do whatever you want

Get to the heart of English language grammar in easy way

4 of 195

Some instrument;
{Bellow, scissors, tongs, pincers, spectacles}
1. 2-9) Certain articles of dress;
{Pants, trousers, drawers, jeans, tights, shorts, pajamas}
10) Certain collective nouns are singular in shape but plural in
meaning;
People
Poultry

Family
Gentry

Cattle

Vermin

Who are those people?


Whose are these cattle?
These poultry are mine.
Vermin destroy our property and carry disease.
There are few gentry in this town.
11) Phrases beginning with (a congregation of/ a crowd of/ a folk of/ a group
of/ a herd of/ a regiment of}
Usually take a singular verb.
0
A crowd of people was shouting.
A group of people is going to oppose this overtax.
A flock of sheep was grazing near the foot of the mountain.
12) Some nouns have no plural such as:
1-12) Whole groups made up of similar items:
Machinery
Equipment
Furniture
Hardware

Clothing
Garbage
Jewelry
Makeup

Baggage
Mail
Money
Luggage

Change
Postage
Scenery
Traffic

Junk
Fruit
Food
Cash

2-12) Fluids: example;


Water
Coffee

Tea
Milk

Oil
Gasoline

Soap
Blood

Butter
Cotton

Cheese
Glass

Smoke

Pollution

3-12) Solid: example;


Ice
Meat

Bread
Gold

4-12) Gases: example;


Air

Oxygen

The cost of this book is to pray for me you have the permission to print it or do whatever you want

Get to the heart of English language grammar in easy way

Steam

Nitrogen

5 of 195

Smog

5-12) Particles: example;


Rice
Chalk
Grass
Sugar

Corn
Dirt
Hair

Dust
Wheat
Pepper

Flour
Salt
Sand

6-12) Abstraction:
Intelligence
Education
Happiness
Knowledge
Hospitality
Recreation

Confidence
Enjoyment
Importance
Significance
Laughter
Patience

Courage
Beauty
Health
Honest
Violence
Wealth

Fun
Sleep
Truth
Justice
Progress

Pride
Peace
Luck
Music
Help

7-12) Languages:
Arabic
French

English
Japanese

Chinese
Russian

Italy
Spanish

8-12) Fields of study:


Chemistry
Mathematics

Engineering
Sciences

History
Medicine

9-12) RECREATION:
Football
Chess

Basketball
Baseball

Billiards
Tennis

Droughts
Soccer

10-12) GERUNDS IN GENERAL:


Driving
Reading

Traveling
Running

Walking
Playing

Writing
Shopping

11-12) NATURAL PHENOMENA:


Weather
Humidity
Darkness
Thunder

Lightening
Electricity
Sunshine
Gravity

Hail
Heat
Sleet
Rain

Fire
Light
Snow
Dew

The cost of this book is to pray for me you have the permission to print it or do whatever you want

Get to the heart of English language grammar in easy way

Fog

6 of 195

Wind

13) Some nouns look plural but they are in fact singular:
The word {news}
Names of some games {billiards}
Names of some disease {measles, mumps}
Name of some subjects {mathematics, etc}
13) A compound noun generally forms its plural by adding(s) to
the principle word; as,
No
1)
2)
3)
4)
5)
6)
7)
8)
9)
10)
11)
12)
13)
14)
15)
16)
17)

Singular
Commandeer-in-chief
Coat-of-mail
Son-in-law
Step-son
Maid-servant
Man-servant
Passer-by (after noun)
Looker-on (after noun)
Man-of-war
Boyfriend
Flower shop ( 2nd noun)
Matchbox
On looker (after noun)
Looker on
Break down (no noun)
Forget-me-not (no noun)
Grown-up (no noun)

Plural
Commanders-in-chief
Coats-of-mail
Sons-in-law
Step-sons
Maid-servants
Men-servant
Passers-by
Lookers-on
Men-of-war
Boyfriends
Flower shops
Matchboxes
On lookers
Lookers on
Break downs
Forget-me-nots
Grown-ups

13) FIGURES, LETTERS are made plural by adding (apostrophe and S);
a) Two 3s b) four 5s c) there are more es than as.
14) Some nouns have the same shape in plural and singular:
(Dear, Fish, Means, Series, Sheep, Species)
15) When we add the article to some nouns or adjectives it has the
meaning of plural:
1-15) Miss. Jane and Mary smith
The Miss. Smith.
2-15) Smith family
The Smith.
{NOTE} when a noun is used as a modifier, it is in its singular:
2.6)
The soap has vegetables in it.
The cost of this book is to pray for me you have the permission to print it or do whatever you want

Get to the heart of English language grammar in easy way

7 of 195

2.7)
It is a vegetable soap.
2.8)
The building has offices.
2.9)
It is an office building.
{NOTE} when a noun is used as a modifier is combined with a number
expression, the noun is singular and a hyphen (-) is used:
The test lasted two hours.
It was a two-hour test.
He is five years old.
He is a five-year-old.
{NOTE} many nouns can be used as either uncountable or Count nouns,
but the meaning is different; as,
Uncountable : Osama has brown hair.
Countable : Osama has a hair on his coat.
Uncountable : I opened the curtain to let in some light.
Countable : Dont forget to turn the lights off when you go out.
Uncountable : I bought some coffee.
Countable
: Two coffees and four teas, please.
Uncountable : This is made of glass.
Count able
: I broke a glass this morning.
Uncountable
: It is hard Iron.
Countable
: I have an iron.
Uncountable
Countable

: Education is important.
: You have a good Education.

12) The use of {how many & how much}


1-12) How many + plural noun+ do/ does+ subject?
How many friends do you have?
I have thirty friends.
I have one friend.
2-12) How much + uncountable noun+ do/does+
subject?
How much sugar do you have?
I have a lot of sugar. I dont have any sugar.
19) { NOTE} Many nouns can be used as a countable noun by adding
particular phrase;
The cost of this book is to pray for me you have the permission to print it or do whatever you want

Get to the heart of English language grammar in easy way

A brood of chicken
A bag of orange
A bottle of Pepsi
A bowl of soup
A box of sweet

A range of
mountains/cliffs
A sack of coal
A series of events
A sort of cake
A drop of water

A bag of flour
A bunch of banana
A bunch of grapes
A bunch of keys
A bundle of hay
A flock of geese

A cluster of stars
A game of football
A clump of trees

A flight of stairs
A flight of birds
A chain of mountains
A flock of sheep
Galaxy of stars
A gallon of petrol
A collection of
relics/curiosities
A gang of laborers
A gang of thieves
A glass of water

A cube of ice

A piece of jewelry

A cup of tea
A pocketful of money
A pot of tea

A group of island
A heap of ruins
A heap of sand
A herd of
cattle/deer/swine

A covey of partridges

A pound of meat
A quart of milk

A jug of milk

8 of 195

A spoonful of
sugar
A spell of work
A stack of wood
A tone of iron
A swarm of bees
A kilo of meat
A litter of puppies
A loaf of bread
A make of car
A meter of cloth
A nest of ants
A pack of wolves
A pair of gloves
A pair of jeans
A period of calm
A piece of
furniture
A grain of rice
A piece of paper
A pile of books
A pinch of salt
A pocket of
cigarettes

20) Some expressions of quantity;

No

Expression of
quantity

Used with count


nouns

1)
2)

One
Each

One apple
Each apple

Used with
uncountable
noun
(x)
(x)

The cost of this book is to pray for me you have the permission to print it or do whatever you want

Get to the heart of English language grammar in easy way

No

Expression of
quantity

3)
4)
5)
6)
7)
8)
9)
10)
11)
12)
13)
14)
15)
16)
17)
18)
19)
20)
21)

Every
Two
Both
A couple
Three, etc.
A few
Several
Many
A number
A little
Much
A great deal
Not any
No
Some
A lot of
Plenty
Most
All

Used with count


nouns
Every apple
Two apples
Both apples
A couple of apples
Three apples
A few apples
Several apples
Many apples
A number of
(x)
(x)
(x)
Not any apple
No apple
Some apples
A lot of apples
Plenty of apples
Most apples
All apples

9 of 195

Used with
uncountable
noun
(x)
(x)
(x)
(x)
(x)
(x)
(x)
(x)
(x)
A little of rice
Much rice
A great deal of
Not any rice
No rice
Some rice
A lot of rice
Plenty of rice
Most rice
All rice

{NOTE} you may use {a lot of} or {great deal of} instead of {much} in
positive sentences.
But in questions and negative sentences we use {much} instead:
O
There is a great deal of rice in that plate.
There is a lot of rice in that plate
There is not much rice in that plate.
Is there much rice in that plate?

Agreement of the verb with the subject:


No
1)
2)

Singular
My friend is happy
That book on political parties is

Plural
My friends are happy.
Those books on politics are

The cost of this book is to pray for me you have the permission to print it or do whatever you want

Get to the heart of English language grammar in easy way

No

6)

Singular
interesting
A poor needs help
My family is big
There is a book, a pen and a
copybook
Some of the book is good

7)

A lot of the equipment is new

8)

Two-thirds of the money is mine

9)

26)

One of my friends is here


English is spoken in many
countries
None of the boys is here
(informal)
The number of students in this
class is fifteen
The news is interesting
The united states is big
Eight hours of sleep is enough
Ten dollars is enough
Five thousand miles is too far to
travel
(X)
Neither of them is my friend
Either Osama or Ali is my friend
Each of my friends is here
Every one of my friends is here
Every man, woman and child is
happy
Each man, woman and child is
happy
Growing flowers is her hobby

26)

(X)

3)
4)
5)

10)
11)
12)
13)
14)
15)
16)
17)
18)
19)
20)
21)
22)
23)
24)

27)
28)

Two years is enough to build this


school
It is this acids that I want

10 of 195

Plural
interesting
The poor need help
My family are eating
There are a book, a pen and
a copybook
Some of the books are good
A lot of my equipments are
new
Two-thirds of the pennies
are mine
My friends are here
The English are good people
None of the boys are here
A number of student are
absent today
(X)
(X)
(X)
(X)
(X)
The police have been called
(X)
(X)
(X)
(X)
(X)
(X)
(X)
None of them are eating.
None are so deaf as those
who will not hear
(X)
It is these acids that I want

The cost of this book is to pray for me you have the permission to print it or do whatever you want

Get to the heart of English language grammar in easy way

No

Singular

29)

There is no girl in this school

30)
31)
32)
33)
34)

35)

36)

The committee has issued its


report
Three parts of the business is left
for me to do
Much pains has been taken
He is mistaken, or else I am
The house, with its contains, was
insured.
Silver, as well as cotton, has fallen
in price.
The professor as well as many of
his colleagues is going to attend
the conference.
The prime minister, along with
some of his cabinet colleagues, is
coming here tomorrow.

11 of 195

Plural
There are no girls in this
school
The committee is divided on
one minor point.
(X)
Great pains have been taken
(X)
(X)

(X)

(X)

Supply a verb in agreement with its subject:


To take pay and then not to do work .dishonest.
The cost of all these articles.. risen.
The jury.divided in their opinion.
That night every one of the boats crew..down with fever.
One or the other of those fellowsstolen the watch.
The strain of all the difficulties and vexations and
anxieties.more than he could bear.
No news good news.
The accountant and the cashier..absconded.
A good man and useful citizen passed away.
The famous juggler and conjurer..too unwell to perform.
The three Musketeers .written by Adams.
Adams..famous writer.
Each of the suspected men arrested.
The ebb and flow of tidesexplained by Newton.
Nine dollars..enough for lunch.
The cow as well as the horse grass.
Neither his father nor his mother.alive.
There ..many objections to the plan.
Two-thirds of the city..in ruins.
The formation of the paragraphs. Very important.
Mans happiness or misery.in a great measure in his own hand.
The cost of this book is to pray for me you have the permission to print it or do whatever you want

Get to the heart of English language grammar in easy way

12 of 195

Ten kilometers..a long walk.


Fifteen minutes.allowed to each speaker.
Five thousands Rials.. a large mount.
The Arabian Nights .still a great favorite.
Two thirds of the oranges .. rotten.
The register but not the Finance officer .. unhappy about the
delay in payment.
Each faculty member as well as most of the students . Of
the view that there should be many more new courses.
Six weeks in summer.. The normal vocation period in
collages and university.
There .. plenty of job opportunities for people in the region.
There plenty of room for improvement in her work.
Mumps.. considered a serious disease for adults.
Your trousers .. torn.
The whole herd of cattle .sold.
A pair of trousers hanging over the chair.
Bad news always (travel/travels) fast.
A lot of people (prefer/prefers) coffee to tea.
What (make/makes) it impossible for him to save any money (is/are)
excessive smoking and drinking.
Every man and woman who . Working in the factory
(deserve/deserves) to be congratulated on the increase in output this
year.
About fifty per cent of the committee members (have/has) promised
to support my proposal.
It (is/are) these three students who started the quarrel.
It (is/are) you who (is/are) to be blamed for the accident.
He is one of those students who (do/does) not do homework
regularly.
His participation in those undesirable meetings during the last three
weeks (has/have) annoyed everyone.
Dr. Abdulraheem, your friend and colleague, (is/are) leaving next week.

Gender of a noun: is entirely a matter of sex or the absence of sex.


1) Masculine gender: is a noun that denotes a male; as,
(brother, father, boy, man, uncle, nephew, etc)
2) Feminine gender: a noun that denotes a female; as,
(sister, mother, girl, woman, aunt, niece, etc)
The cost of this book is to pray for me you have the permission to print it or do whatever you want

Get to the heart of English language grammar in easy way

13 of 195

3) Common gender: is a noun that denotes either a male or a


female; as,
(parent, child, student, baby, friend, infant, etc)
4) Neuter gender: is a noun that denotes a thing that is neither a
male nor a female but it is often personified, as if
they are living beings; as, (ship, car, sun, summer,
moon, earth, autumn, nature, liberty, hope,
mercy, justice, peace, charity, etc)
{NOTE} The things that denote power, strength or violence always
represents masculine; as, (sun, summer, winter, death, time,
punishment, etc)
1) The sun is nice. He is good for us.
2) I like the summer and his nice weather.
While objects applied to(remarkable for beauty, gentleness, and gracefulness
represent feminine); as, (the moon, the earth, spring, Autumn, Nature, liberty,
justice, Mercy, peace, hope, charity)
EXAMPLE:
That ship is large. She is the largest ship that I have ever seen.
The moon has hidden her face behind a cloud.
Spring had spread her mantle of green over the earth.
Peace hath her victories no less renowned than war.
The ship lost all her boats in the storm.
Male
Boy
Father
Husband
Actor
Salesman
Great-uncle
Milkman
Grandfather
Hero
Hunter
Founder
Master
Traitor
Conductor
Lion

Female
Girl
Mother
Wife
Actress
Saleswoman
Great-aunt
Milk woman
Grandmother
Heroine
Huntress
Foundress
Madam
Traitress
Conductress
Lioness

Male
Brother
Uncle
Man
Manager
Gentleman
Manservant
Washman
Landlord
Duke
Benefactor
Waiter
Murder
Enchanter
Instructor
Host

Female
Sister
Aunt
Woman
Manageress
Lady
Maidservant
Washwoman
Landlady
Duchess
Benefactress
Waitress
Murderess
Enchantress
Instructress
Hostess

The cost of this book is to pray for me you have the permission to print it or do whatever you want

Get to the heart of English language grammar in easy way

Heir
Poet
Author
Dog

Heiress
Poetess
Authoress
Bitch

Count
Priest
Earl
Bull

14 of 195

Countess
Priestess
Countess
Cow

Nabeel, our teacher, traveled yesterday morning.


We see that Nabeel and our teacher are the one and the same person.
The noun TEACHER follows the noun NABEEL simply to explain
which Nabeel WE referred to.
The article:
0 A & AN:
They are used with singular noun that is countable.
Example:
This is a car.
(Cars)
This is an orange.
(Oranges)
{AN} but not (a) can be used before the sound of the vowel letters / /
Example: An orange. An umbrella. An apple, An hour.
{A} But not (an) can be used with the rest sounds of the consonant letters.
Example:
//
A bag.
A university.
A house.
Study the following:(a or the)+ singular countable : a hat the hat.
(an or the)+singular countable: an orange _ the orange
(the or zero)+plural countable: the hats- the oranges- hats- oranges.
(the or zero)+ uncountable : the water. Water.
My neighbor is an architect.
Our neighbors are architects.
The architects who designed this block won a prize.
Other usage of the articles :
1) To represent number (one); as,
(Not a word was said)
2) In the sense of (any) to represent a whole group from the same
class; as,
(A pupil should obey the rule of school.)
3- To refer to an example of that class; as,
A Shakespeare sonnet.
The cost of this book is to pray for me you have the permission to print it or do whatever you want

Get to the heart of English language grammar in easy way

4567-

15 of 195

With measures; as,


A kilo, an hour, a gallon, a dozen, a day. (= every)
With pairs of nouns; as,
A cup and saucer; a hat and coat; a knife and fork.
With (what) and (such); as,
What a surprise. What an interesting story.
With reference to illness; as,
I have a headache. I catch a cold.

The use of the definite article (The):


When we talk about a particular person or thing or one already referred to:
1) The book you want is out of print.
2) Lets go to the park.
3) He found a book. The book is on Yemen.
When singular represents a whole class; as,
The cow is a useful animal.
The cat loves comfort.
When an adjective represents a whole class, in other word, when the
adjective used as a noun; as,
1) The poor are suffering.
2) The rich are careless.
Before some proper nouns; as,
(Oceans, seas, rivers, canals, desert, group of island, mountain
rages, some name countries which start with republic,
kingdom)
< The Pacific. The Red Sea. The Nile. The Suez Canal. The
Sahara. The West Island. The Alps. The Yemen. The United
Kingdom>
Before names of things unique of their kinds; as,
< The sun. The ocean. The sea. The earth>
With superlative; as,
He is the tallest man I have ever seen.
With ordinals; as,
The first, the second, etc.
Before musical instrument; as,
Can you play the piano?
The cost of this book is to pray for me you have the permission to print it or do whatever you want

Get to the heart of English language grammar in easy way

16 of 195

Omission of the article:


Before names of substances and abstract nouns;
as, (sugar, gold, wisdom, honesty, virtue, .. etc)
Sugar is bad for teeth.
Gold is a precious metal.
Wisdom is the gift of heaven.
Honest is the best policy.
Virtue is its own reward.
**(Note) uncountable nouns take {the} when it is known to the addresser;
as,
Would you please pass me the sugar? (the sugar on the table)
The wisdom of your father is great.
I can not forget the kindness which treated me with.
Before plural countable nouns used in general; as,
(children, computers, ..etc)
a) Children need more care.
b) Computers are used in many offices.
**(note) we can use {the} if we refer to a particular thing or person; as,
where are the children? (= our children)
Before most proper nouns namely, names of continents, countries, cities,
individual mountains, individual islands, lakes, hills, etc.
**(Note) when the article is used before Proper nouns, they become Common
nouns; as,
This man is the second Newton. (= A philosopher as great as Newton)
He was the Napoleon of his age. (= The greatest general of his time.)
He is the Shakespeare of this country. (= The greatest poet)
Before the name of persons; as,
(Ahmed, Amer)
Ahmed is the son of Amer.
Before languages; as,
(Arabic, English, ..etc) They study Arabic.
In certain phrases consisting of a preposition followed by its object; as,
(At home, in hand, in debt, by day, by night, at daybreak, at sunrise, at
noon, at sunset, at night, at anchor, at sight, on demand, at
interest, on earth, by land, by water, by river, by train, by steamer,
by name, on horseback, on foot, on deck, in jest, at dinner, at ease,
under ground, above ground.

The cost of this book is to pray for me you have the permission to print it or do whatever you want

Get to the heart of English language grammar in easy way

17 of 195

In certain phrases consisting of a transitive verb followed by its object; as, (to
catch fire, to take breath, to give battle, to cast anchor, to send word, to
bring word, to give ear, to lay siege, to set sail, to lose heart, to set foot,
to leave home, to strike root, to take offence)
With a unique position held at one time by one person only; as,
He was elected chairman of the board.
He became principal of the collage in 1989.
Before names of relations, like (father, mother, aunt, uncle, and also nurse
and cook); as,
Father has returned.
Aunt wants you to see her.
Cook has given notice.
Before the words (hospital, bed, table, school, church, market, college,
prison, or places are visited or used for primary purpose; as,
I learnt English at school.
I will go church.
My uncle stays in bed in hospital three days.
(Note) {The} is used with these words when we refer to them as a definite place,
building or object than to the normal activity that goes on there; as,
The school is very near to my house.
I met him at the church.
The bed is broken.
I went to the hospital to see my uncle.
Fill in the blanks in (a, an, or the):1) copper is .. useful metal.
2) He is not .. honorable man.
3) able man has not always a distinguished look.
4) reindeer is a native of Norway.
5) Honest men speak .. truth.
6) Do you see . Blue sky?
7) He retuned after hour.
8) school will shortly close for rebuilding.
9) sun shines brightly.
10) I first met himyear ago.
11). Lion is .. king of beast.
12) You are .. fool to say that.
13) French is .. easy language.
14) Who is .. girl sitting over there?
15) Dubai is . Very dear place to live in.
The cost of this book is to pray for me you have the permission to print it or do whatever you want

Get to the heart of English language grammar in easy way

18 of 195

Orders of nouns, in other word, nouns used as adjectives:


1) parts of a whole: from the largest to the smallest one; as,
The house garden gate bell.
2) Something used for especial purpose: the purpose should be in the
beginning and then the material used for; as,
A pan used for frying. It is a Frying pan.
factory built for making Biscuit . biscuit Factory.
Shade made for lamp. Lampshade.
- table used for tea Tea table.
Collage built for Educating student Education collage.
Games made for computers Computer games.
3) If a movable thing consists of many similar parts we should use (OF);
as,
The leg of the table.
The door of the car.
4) The proper noun always comes first; as,
Al-Manal International Institute.
The Alps Mountain.
Al-Horia Street.
Sheba School.
5) The kind of place that you work for always comes at the end; as,
(factory, company, establishment, organization,
office,
school,
institute, etc)
Al-Maram International School. Alameen Trade Company.
6) The purpose that the organization built for can be put at the end or in
the middle of a sentence as,

Al-Mram Company for trading.


Al-Maram trading company.
Al-Mram trade company.
2) Al-Mnal International Corporation for services.
Al-Mnal International Service Corporation.
3) Egyptian International company for foodstuff industries.
Egyptian International foodstuff industries company
7) If the name of place consist of two names of different countries we
should put first the name where the described place located in; as,
The Yemen America institute.
(= The institute is located in Yemen.)
8) If there are a lot of nouns used with (of) to describe a particular persons
position, we should do as the following.
The cost of this book is to pray for me you have the permission to print it or do whatever you want

Get to the heart of English language grammar in easy way

19 of 195

The dean of the collage of languages and arts.


The dean of the languages and arts collage.
Dean,
The collage of languages and arts.
The chairman of general authority of tourism.
The chairman,
General Authority of tourism.
9) Something worked by something: the operator or power is always put first;
as,
Kettle works by Electric Electric kettle.
bomb works by petrol Petrol bomb.
Others: Compound nouns maybe formed from:NOUN +NOUN
Moon light
Chessboard
Armchair
Airman
Manservant
Fire escape
Shoemaker
Ringleader
Screwdriver
Windmill
Cupboard
raincoat
ADJECTIV+NOUN
Sweetheart
Nobleman
Shorthand
Stronghold
Halfpenny
Greenhouse
VERB+NOUN
Spendthrift
Makeshift
Cutthroat
Cutpurse

Break post
Daredevil

GERUND+NOUN
Drawing room
Writing desk
Blotting paper
Spelling book
f) NOUN+GERUND
Horseracing
Horse riding

Outlaw
Downfall
Inside

Postman
Jailbird
Taxpayer
Saucepan

Blackboard
Heavyweight

Telltale
Scarecrow

Walking stick
Stepping stone

Sightseeing

Railway
Horsepower
Teaspoon
Seaside

Quicksilver
Longhand

Pickpocket
Hangman

Looking glass

Sunbathing

ADVERB (or PREOPSITION)+NOUN


Afterthought
Forethought
Foresight
Afternoon
Bypath
Inmate

Picking fruit

Fruit picking

Overcoat
Offshoot

The cost of this book is to pray for me you have the permission to print it or do whatever you want

Get to the heart of English language grammar in easy way

VERB+ADVERB
Drawback
Lockup

Go between

ADVERB+VERB
Outset
Up keep

Out cry

RELATED TO TIME
Afternoon tea
Morning coffee
i) FORMED WITHSELF
Self-confident Self-respect

Die hard

Send off

Income

outcome

Sunday lunch

Self-control

20 of 195

A two-hour work

Self-denial

Self-consciousness

a) Transitive verb+ object: object-verb+ ing; as,


It breaks the heart: heart-breaking.
b) Verb+ adverb of manner: the adverb-p.p.
He behaves well: well behaved.
c) Free from+ noun: noun+ free.
Free from care: carefree.
d) Adjective+ like+ noun: noun-adjective.
It is red like blood: red-blood.
Point out the nouns in the following sentences, and say whether they are
Common, Proper, Collective or abstract: 1) The crowd was very big.
2) Always speak the truth.
3) We all love honest.
4) Our class consists of twenty pupils.
5) The elephant has a great strength.
6) Solomon was famous for his wisdom.
7) We saw a fleet of ships in the harbor.
8) The class is studying grammar.
9) The God vary overflows its banks every year.
10) A committee of five was appointed.
11) The soldiers were rewarded for their bravery.
12) He gave me a bunch of grapes.
13) I recognize your voice at once.
14) Our team is better than theirs.
15) The streets of some of our cities are noted for their crookedness.
16) The room is thirty feet in length.
17) I believe in his innocence.
18) Wisdom is better than strength.
The cost of this book is to pray for me you have the permission to print it or do whatever you want

Get to the heart of English language grammar in easy way

21 of 195

19) He sets a high value on his time.


20) What is your verdict, gentlemen of the jury?
Form Abstract Noun from the following: {Long, young, humble, decent, cruel, bitter, strong, true, short,
prudent, dark, deep, wide, wise, good, vacant, sweet, human, broad, free,
proud, brave, novel, quick, high, poor, just, vain, ignorant, laugh, obey, live,
expect, excel, know, steal, believe, serve, hate, please, act, starve, occupy,
choose, move, conceal, seize, flatter, depart, persevere, succeed, free, see,
judge, pursue, relieve, discover, king, man, thief, woman, bankrupt, infant,
owner, regent, author, hero, beggar, coward, pilgrim, boy, priest, bond,
pirate, friend, captain, rascal, patriot}

PRONOUN

A PRONOUN : it is a word used instead of a noun to a void the repetition of


the noun; as, Ali is absent, because Ali is ill.
Ali is absent, because he is ill.
Osama is a good student. He is diligent.
The rose is nice. It smells sweet
The house is painted white. It is painted white
A PRONOUN : is divided into :
1) Personal pronoun.

2) Impersonal pronoun

The cost of this book is to pray for me you have the permission to print it or do whatever you want

Get to the heart of English language grammar in easy way

3) Reflexive pronoun.

22 of 195

4) Emphatic pronoun

{We, I, you, he, she, it, they} are called personal pronouns because they
stand for the three persons.
(I, we) are the first person(s). (The person speaking )
(You) is the second person(s). (The person spoken to)
(he, she, it, they) are the third person(s). (The person spoken about)
NO
(1)
(2)
(3)
(4)
(5)
(6)
(7)
(8)
(9)

NOUN
The speaker
The speakers
The addresser (person or thing)
The addressers(person or thing)
Nabeel
Manal
Osama and Maram
Cat
Cat and dog

PRONOUN
I
We
You
You
He
She
They
It
They

Impersonal pronoun or empty pronoun; as,


(It) when it is used as,
Emphasizing:
It is me who broke the window.
Standing for no noun:
It rains hard. (= The rain rains)
Referring to weather or time:
It is winter.
It is fine.

It is ten oclock.

The use of {it}


1) Represent things without life; as,
I lost my bag. It is black.
Here is your book; take it away.
2)To identify people,
There is a knock at the door. Who is it?
It is the postman.
3) for a young, unless we clearly wish to refer to the sex; as,
We will have child in May. We will call it Ahmad if it is a boy
but if not we will call it Manal.
The cost of this book is to pray for me you have the permission to print it or do whatever you want

Get to the heart of English language grammar in easy way

23 of 195

4) Animal, unless we wish to speak of them as male and female; as,


He loves his dog and cannot do without it.
The horse fell and broke its leg.
5) To refer to some statement going before; as,
He is telling what is not true; as he knows it.
He deserved his punishment; as he knew it.
6) As a provisional and temporary subject before the verb to be
when the real subject follows; as,
It is easy to find fault. (To find fault is easy)
It is doubtful whether he will come.
It is certain that you are wrong.
To give emphasis to the noun or the pronoun; as,
It was you who began the quarrel.
It was I who was first protest.
It was at Al-markzi that the treaty was made.
It is a silly fish that is caught twice with the same bait.
It is an ill wind that blows nobody good.
With since:
It is three years since I met him.
With said
It is said he is 90 years old.
With takes; as,
It takes us an hour to get to work
As a preparatory subject; as,
It is pleasant to lie in the sun.
It is pleasant lying in the sun.
It is a shame that Tome is not here.
It is easy to make mistake.
It is a pleasure to be here.
It appears that he forgot to sign the letter.
It now looks certain that the fire was caused by cigarette end.
As an indefinite nominative of an impersonal verb; as,
1) It rains
2) It snows 3) It thunders
{Note} The pronoun (it) here seems to stands for no noun whatever, though
this can be readily supplied from the verb. Thus, it rains means
the rain rains.
The cost of this book is to pray for me you have the permission to print it or do whatever you want

Get to the heart of English language grammar in easy way

24 of 195

{Note} Indefinite pronouns: they refer to person or thing in general way,


not in particular; as,
(One, none, they, all, some, a few, a little, somebody, something, everybody, etc);
as,
One hardly knows what to do.
None but fools have ever believed it.
They say he has lost heavily.
All were drowned.
Few escaped unhurt.
Some are burn greatly.
Somebody has stolen my watch.
No body was there to rescue the child.
Many of them were not guilty.
We did not see any of them.
Do good to others.
Did you ask anybody to come?
What is everybodys business is nobodys business.
His story is in everyones mouth.
**(Note) 1- in referring to {anybody, everybody, everyone, anyone, each, etc}
the pronoun {he or she} is used according to the context; as,
I will be glad to help anybody of my boys in his study.
{But now} it is more usual to use a plural pronoun (they, them, their) ;
as,
a) Anybody can do it if they try.
b) Each of them had their share.
**(Note) 2- most of these words may also be used as Adjectives, when they
followed by a noun; as,
I will take you there one day.
Any fool can do that.
He is a man of few words.
Some milk was spilt.
Position of {each}
Each of the men received a reward.
These men received each a reward.
I bought these rewards for $10 each.
If I have a child, I will call it Ali if it is a boy.
Possessive
Pronoun
No
adjective
(1)
I
My
(2)
We
Our
(3)
You
Your
The cost of this book is to pray for me you have the permission to print it or do whatever you want

Get to the heart of English language grammar in easy way

25 of 195

(4)
(5)
(6)

He
His
She
Her
They
Their
This is my car.
These are my cars.
This is his book.
This is her copybook.
This is their house.
This is your pen.
This is our school.
**(Note) we call this kind Possessive Adjective because they are followed by a
noun, and do the work of an adjective. Hence, as they are formed from
pronouns, they are called Pronominal Adjective.
**(Note) the words {his & its} can be used as an adjective and a Pronoun; as,
This is his book. It is his.
This is its food. It is its.
Fill in the blanks with either pronoun or object:
All failed except.(he)
That is a matter between you and.(I /me)
Leave her and ..to toil alone.(I /me)
It is not ..who are to blame.(I /me)
You and ..are invited to tea this evening.(I/me)
Between you and , he drinks heavily.(I /me)
.telephoned yesterday.(she)
We watch . For hours. (he)
Has not arrived yet? (she)
dont understand. (I )
Are you talking to ..? (I)
{Note} It is more polite to say:
(You and I) than (I and you)
(You and he) than (he and you)
(You and Ahmad) than (Ahmad and you)
(Ahmad and I) than (I and Ahmad)
(He and I) than (I and he)
{Note) if we want to use possessive adjective with the above mentioned, we
should know that:
{You and I } = we You and I must mend our way.
{You and he} = your you and he must mend your way.
{He and I}
= we He and I must mend our way.
The cost of this book is to pray for me you have the permission to print it or do whatever you want

Get to the heart of English language grammar in easy way

No
(1)
(2)
(3)
(4)
(5)
(6)

Pronoun
I
We
You
He
She
They

26 of 195

Possessive pronoun
Mine
Ours
Yours
His
Hers
Theirs

This is my car. It is mine.


These are my cars. They are mine.
This is his book. It is his.
This is her copybook. It is hers.
This is their house. It is theirs.
This is your pen. It is yours.
This is our school. It is ours.
Write the correct adjective or pronoun for these sentences:
Whose camera is this? Is it ..?(you)
Excuse me, those are .seats.(we)
Is it ..suitcase or ?(you/he)
Has the dog had . Food? (it)
They are not . Keys- they are (I/she)
I dont think its room; I think it is .(you/ they)
The police asked for .. address.(I)
Have you got .. pen, or would you like to borrow ..?(you/I)
.. garden is bigger than ( they/we)
I think this is book. Oh no, it is .(I/you)
The decision is ..(they)
The cat wants dinner.(it)
You know it is not money. It is (you/I)
Its not .. car, it is. (he/she)
Have you met .? (mother)

In the following sentences point out the pronouns and say for what each stands:Maram was not a bit hurt, and she jumped up on to her feet in a moment.
There were doors all round the hall, but they were all locked.
Manal opened the door and found it led into a small passage.
I wish I had not cried so much, said Zakria.
you are not attending, said the Mouse to Sahar severely. What are you
thinking of?
Come back, the Caterpillar called after her. Mary turned and came back again.
Osama brought his book and laid it on the table.
The cost of this book is to pray for me you have the permission to print it or do whatever you want

Get to the heart of English language grammar in easy way

27 of 195

Hail lost his dog and cannot find it.


Osama is at the head of his class, for he studies hard.
The camel is a beast of burden. It is used to carry goods across the desert.
The female lion is called a lioness. She has no mane.
Train up a child in the way he should go.
In the following sentences use Pronouns in place of nouns wherever you
can:
1) Ameen had taken his watch out of his pocket, and was looking at the
watch uneasily, shaking the watch every now and then, and holding the
watch to his car.
2) The boys went into the garden, where the boys saw a snake.
Very soon the Rabbit noticed Jamal as Jamal went hunting about, and called
out to Jamal in an angry note.
Compound personal pronoun; as,
No
Pronoun
(1)
I
(2)
We
(3)
You
(4)
You
(5)
He
(6)
She
(7)
They

Reflex pronoun
Myself
Ourselves
Yourself
Yourselves
Himself
Herself
Themselves

There are two kinds of Compound personal pronoun:


1) Reflexive pronoun: is used as the object of a verb, and refers to the
same person or the thing mentioned before; as,
I saw myself in the mirror.
He saw himself in the mirror.
She saw herself in the mirror.
They saw themselves in the mirror.
You saw yourself in the mirror.
We saw ourselves in the mirror.
2) Emphatic pronoun: is used for emphasize; as,
I will do it myself.
I myself saw him do it.
He himself said so.
We saw the prime minister himself.
The town itself is not large.
Tell which Pronoun in the following sentences are Reflexive and which
Emphatic:The cost of this book is to pray for me you have the permission to print it or do whatever you want

Get to the heart of English language grammar in easy way

28 of 195

I will go my self.
She hurt herself.
We often deceive ourselves.
I myself heard the remark.
You express yourself very imperfectly.
I wash myself when I get up.
The boys hid themselves.
I absented myself from school.
They love themselves so much that they think of no one else.
The poor widow poisoned herself.
The prisoner hanged himself.
Dont you deceive yourself?
He set himself a hard task.
We exerted ourselves.
The dog choked itself.
They gave themselves a lot of trouble.
We seldom see ourselves as others see us.
A house divided against itself can not stand.
He that wrongs his friend wrongs himself more.
He has landed himself in difficulties.
I was sitting by myself.
I can not bring myself to do it.
Pray dont inconvenience yourself.
And sometimes I do that myself.
Some people are always talking about themselves.

VERB
A verb : is word used to tell or assert something about some person or thing .
A verb often consists of more than one word; as,
1) There are two kinds of verb:
(1)Verb is divided into {Main Verb & auxiliary}
The cost of this book is to pray for me you have the permission to print it or do whatever you want

Get to the heart of English language grammar in easy way

29 of 195

Main Verb = is a word which is used to express an action and it can be


a regular Verb Like Play---Played }
or an irregular Verb Like Know---Knew.}
The use of a verb:
1) To tell us what a person or thing does; as,
He is eating.
The dog is barking.
2) To tell us what is done to a person or thing; as,
The man is hardly beaten.
The window is broken.
3) To tell us what a person or thing is; as,
He is a doctor.
He is clever.
He is a teacher.
4) To tell us where a person or thing is; as,
He is in his room.
Main verb divided into:
1) Transitive verb: is a verb, which is in need of an object; as,
He kicks the ball. (Here we should use an object to let the
sentence have a sense)
2) Intransitive verb: is a verb, which can stand by itself and have a sense; as,
He is sleeping.
Verbs of incomplete prediction usually express the idea of being, become,
seem, appear. The complement usually consists of a Noun called a
(PREDICATIVE NOUN) or an adjective called a (PREDICATIVE
ADJECTIVE). When the complement describes the subject, as in the
following sentences, it is called a (SUBJECTIVE COMPLEMENT).
Tabby is a cat.
Tom became a soldier.
The man seems tired.
The sky grew dark.
Ali became mayor.
You look happy.
2) Certain Transitive Verbs require, besides an object, a complement to
complete their predication and therefore, called an OBJECTIVE
COMPLEMENT; as,
We made him captain.
His parents named him Ahmad.
The cost of this book is to pray for me you have the permission to print it or do whatever you want

Get to the heart of English language grammar in easy way

30 of 195

This made him vain.


The jury found him guilty.
He called his cousin a liar.
I consider the man trustworthy.
We thought him a rascal.
They chose him their leader.
3) There are a few Transitive Verbs which are sometimes used as
Intransitive Verbs; as,
He broke the glass.
The glass broke.
He burnt his fingers.
He burnt with shame.
Stop him from going.
We will stop here for dinner.
Open all the windows.
The show opens at six oclock.
The horse walked.
I walked the horse.
The girl ran down the street.
The girl ran the needle into her finger.
Birds fly.
The boy flies his falcon.
Many verbs can be used as transitive and intransitive verbs; such as,
The ant fought the wasps.
Some ants fight very fiercely.
The shot sank the ship.
The ship sank rapidly.
Ring the bell, Osama.
The bell rang loudly.
The driver stopped the train.
The train stopped suddenly.
He spoke the truth.
He spoke haughtily.
The horse kicked the man.
The horse never kicked.
I feel a severe pain in my head.
How do you feel?
Some transitive verbs may become transitive by having a preposition
added to them; as,
All his friends laughed at him. ( derided)
He will soon run through his torture. (consume)
Please look into the matter carefully. (investigate)
We walked about the affair several times. (discussed)
The cost of this book is to pray for me you have the permission to print it or do whatever you want

Get to the heart of English language grammar in easy way

31 of 195

The police inspector asked his name. (demanded)


Intransitive verbs Sometimes take after them an object similar to the
meaning to the verb. Such an object is called the Cognate Object or
Accusative.
I have fought a good fight.
He laughed a hearty laugh.
I dreamt a strange dream.
He sleeps the sleep of the just.
Let me die the death of the righteous.
She sighed a deep sigh.
She sang a sweet song.
Name the verb in the following sentences and tell if its transitive or not:
The sun shines brightly.
The boy cut his hand with a knife.
The clock stopped this morning.
The police blew his whistle.
The sun rises in the east.
I looked down from my window.
The cat slept on the rug.
The boy easily lifted the weigh.
We eat three times a day.
The dog ran after me.
Verbs of perception:
Certain verbs of perception are followed by either the simple form or the ing
form. There is usually little difference in meaning between them.
Study the following:
1-I saw my friend run down the street.
2-I saw my friend running down the street. (=While)
3-Fathi heard Nader sing at Sabafon.
4-Fathi heard Nader singing at Sabafon.
(= he heard him from the beginning to the end).
Verbs of perception: see, notice, watch, look at, observe, hear, listen to, feel,
and smell.
2.25.1.1. IRREGULAR VERBS; are divided into the following;
1-) All three forms are the same:
Base form

Past tense

Bet
Burst
Cost

Bet
Burst
Cost

Past
participle
Bet
Burst
Cost

The cost of this book is to pray for me you have the permission to print it or do whatever you want

Get to the heart of English language grammar in easy way

Base form

Past tense

Cut
Hit
Hurt
Let
Put

Cut
Hit
Hurt
Let
Put

32 of 195

Past
participle
Cut
Hit
Hurt
Let
Put

2.25.1.2. 2-) Two of the forms are the same:


Base form

Past tense

Beat
Bend
Bleed
Breed
Build
Buy
Catch
Creep
Deal
Dig
Feed
Feel
Fight
Find
Hang
Have
Hear
Hold
Keep
Lay
Lead
Leave
Lend
Light
Lose
Make
Mean

Beat
Bent
Bled
Bred
Built
Bought
Caught
Crept
Dealt
Dug
Fed
Felt
Fought
Found
Hung
Had
Heard
Held
Kept
Laid
Led
Left
Lent
Lit
Lost
Made
Meant

Past
participle
Beat
Bent
Bled
Bred
Built
Bought
Caught
Crept
Dealt
Dug
Fed
Felt
Fought
Found
Hung
Had
Heard
Held
Kept
Laid
Led
Left
Lent
Lit
Lost
Made
Meant

The cost of this book is to pray for me you have the permission to print it or do whatever you want

Get to the heart of English language grammar in easy way

Base form

Past tense

Meet
Pay
Say
Sell
Send
Shine
Shoot
Sit
Sleep
Speed
Spell
Spend
Spit
Stand
Stick
Sweep
Swing
Teach
Tell
Think
Understand
Win
Wind

Met
Paid
Said
Sold
Sent
Shone
Shot
Sat
Slept
Sped
Spelt
Spent
Spat
Stood
Stuck
Swept
Swung
Taught
Told
Thought
Understood
Won
Wound

33 of 195

Past
participle
Met
Paid
Said
Sold
Sent
Shone
Shot
Sat
Slept
Sped
Spelt
Spent
Spat
Stood
Stuck
Swept
Swung
Taught
Told
Thought
Understood
Won
Wound

2.25.1.3.
3-) The base verb and past participle are the same:
Base form

Past tense

Run
Become
Come

Ran
Became
Came

4-) Verbs end in (en):


Base form
Past tense
Bite
Bit
Break
Broke

Past
participle
Run
Become
Come

Past participle
Bitten
Broken

The cost of this book is to pray for me you have the permission to print it or do whatever you want

Get to the heart of English language grammar in easy way

Base form
Drink
Drive
Eat
Fall
Forbid
Forget
Forgive
Freeze
Give
Hide
Mistake
Ride
Shake
Shrink
Sink
Speak
Steal
Strike
Take
Wake
Write

Past tense
Drunk
Drove
Ate
Fell
Forbade
Forgot
Forgave
Froze
Gave
Hid
Mistook
Rode
Shook
Shrunk
Sunk
Spoke
Stole
Struck
Took
Woke
Wrote

34 of 195

Past participle
Drunken
Driven
Eaten
Fallen
Forbidden
Forgotten
Forgiven
Frozen
Given
Hidden
Mistaken
Ridden
Shaken
Shrunken
Sunken
Spoken
Stolen
Stricken
Taken
Woken
Written

2.25.1.4. There are eight verbs with alternative past participle the ones end in
(en) used adjectively:
1) He has drunk liquor.
A drunken man.
2) The iron has melted.
Molten iron.
3) He has proved it.
A proven fact.
4) He has shaved off his beard. A clean-shaven face.
5) The have sheared the sheep. A shorn sheep.
6) The cloth has shrunk.
A shrunken head.
7) The ship has sunk.
A sunken ship.
8) The clock has struck five.
A grief-stricken widow.
2.25.1.5. 5-) The three with different forms:
The cost of this book is to pray for me you have the permission to print it or do whatever you want

Get to the heart of English language grammar in easy way

35 of 195

Base form
Past tense
Past participle
Be
Begin
Began
Blow
Blew
Blown
Choose
Chose
Chose
Draw
Drew
Dawn
Drink
Drank
Drunk
Do
Did
Done
Fly
Flew
Flown
Go
Went
Gone
Grow
Grew
Grown
Know
Knew
Known
Lie
Lay
Lain
Ring
Rang
Rung
Rise
Rose
Risen
See
Saw
Seen
Show
Shown
Showed
Shrink
Shrank
Shrunk
Sing
Sang
Sung
Sink
Sank
Sunk
Spring
Sprang
Sprung
Stink
Stank
Stunk
Swear
Swore
Sworn
Swim
Swam
Swum
Tear
Tore
Torn
Throw
Threw
Thrown
Wear
Wore
Worn
2.25.1.6.
6-) Verbs end with ed in the past and in (en) in the past
participle:
Base form
Past tense
Past participle
Melt
Melted
Molten
Prove
Proved
Proven
Shave
Shaved
Shaven
Complete the following:
Infinitive
Be

Past
simple
Was

Past
participle

Infinitive

Beaten

Eat

Past
simple

Past
participle
Driven

The cost of this book is to pray for me you have the permission to print it or do whatever you want

Get to the heart of English language grammar in easy way

Became

36 of 195

Fall

Begin

Felt
Broke

Forgiven
Bought

Seek

Catch
Dream
Spend
Compound Verb:1) NOUN+VERB
Waylay
Backbite
2) ADJECTIVE+VERB
Safeguard
Whitewash
3) ADJECTIVE+VERB:Overthrow
Overtake
Overheard
Overdo

Cost
Learn
swum

Typewrite

Browbeat

Earmark

Undertake
Outdo

Undergo
upset

Fulfill

Foretell
Outbid

2-1) Auxiliary is divided into (Modals and Helping Verb)


1) Modals ={Will, Would, Shall, Should, Can, Could, May, Might,
Must, need, have to, ought to, used to, dare}
2) Helping Verb ={Is, Am, Are, Was, Were, Have, Has, Had}
The use of be;
Continuous tense: He is playing happily.
With adjective : He is handsome.
With professional: He is a doctor.
With place
: He is in the kitchen.
With comparison : He is taller than his brother.
With Passive
: He is beaten.
To convey order : Be quiet.
To convey construction: No one is to leave this building.
To convey a plan: He is to be married next month.
The prim minister is to make a statement tomorrow.
The cost of this book is to pray for me you have the permission to print it or do whatever you want

Get to the heart of English language grammar in easy way

37 of 195

As main verb to express state: he is hot. He is ill.


To refer to age: He is ten.
Size and weight : I am five feet and six inches. I am 120 pounds.
Distance:
It is 20 k. from here.
Weather :
It is hot/cold/ windy.
Time and date : It is nine oclock. It is May 6.
Price:
How much is it? It is $ 2.
***NOTE> the verb to be is considered main verb if it is not followed by a verb as
the preceding example mentioned above.
2) General conversational use: Auxiliary in short answer
1-2) It is used with the questions answered by yes or no, i.e.,
Do you know him? Yes, I do. (Yes, I know him)
Are there enough people? Yes, there are.
2-2) Agreement and disagreement with remarks.
A) Agreements with affirmative remarks are made with yes, so or of course E.g.
He is driving very fast. Yes, he is.
It is very hot. So it is.
The fire has gone out. So, it has.
He plays well. Yes he does. (Of course he does)
**(Note) we cannot used so instead of yes to express surprise. e.g.
Wow! He got it fast. Yes, He did. (Not so he did)
3-2) Agreement with negative remarks. E.g.
Her husband is not working hard. No, he is not.
She has not finished her work. Oh no, she has not.
**(Note) It is incorrect to begin a sentence with yes if the intention is to express
agreement with a negative remark. e.g.
(Incorrect) your friend has not arrived yet. Yes, he has not.
(Correct) your friend has not arrived yet. No, he has not.
4-2) Disagreement with affirmative remarks is made with no or Oh, no and
auxiliary. E.g.
She is making an error. No, she is not.
I have finished my work. Oh, no. You have not.
5-2) Disagreement with negative remarks made with yes or oh, yes. E.g.
She will not come. Oh, yes she will.
You can not do it. Yes, I can.
6-2) Disagreement with implication of a positive question.
The cost of this book is to pray for me you have the permission to print it or do whatever you want

Get to the heart of English language grammar in easy way

38 of 195

Why did you go there? But I did not.


Why have you been teasing her? But I have not.
7-2) Disagreement with implication of a negative question.
Why did not you apply for that post? But I did.
Why did not they arrive in time? But they did.
8-2) Affirmative addition to affirmative remarks made by using so + auxiliary
(or do/does/did if there is no auxiliary) + the subject. E.g.
khaled can drive fast and Kamal can drive fast, too.
khaled can drive fast and so can Kamal.
Khaled helps others and so does Ali.
9-2) Affirmative additions to negative remarks are made with (but +subject +
auxiliary): e.g.
He will not go but they will.
10-2) Negative additions to negative additions are made with (nor or neither +
auxiliary + subject): e.g.
She did not lose anything and he did not lose anything, either.
She did not lose anything and neither he did.
They can not climb the hill and we can not climb it, either.
They can not climb the hill. Nor can we.
11-2) Negative addition to affirmative is made with (but +subject +auxiliary in
the negative): e.g.
Osama can answer the question but Sami can not.
Do the following example as you learned (agreement and disagreement):
He is a great scholar.
The guests have arrived.
It was Sunday yesterday.
Tomorrow is Tuesday.
He will certainly help you.
The university is not closed these days.
He is not willing to go there.
Her article has been accepted for publication.
Those girls are very clever.
Why did not you go there?
Why did you visit him?
Why he has not completed his work so far?
Why did you write a rude letter to her?
Why was he so upset?
Why does he want to kill himself?
Why does he always neglect his work?
The cost of this book is to pray for me you have the permission to print it or do whatever you want

Get to the heart of English language grammar in easy way

39 of 195

We can not allow him to be so negligent.


We have always helped him.
He is an exceptionally good colleague.
They are not going out on holidays.
This is the second edition of your book.
The price of petroleum products is likely to go up next week.
He did not pay his school fee last month.
His lecture was very illuminating.
Your project has been approved.
Her suggestions were not accepted.
He would not be allowed to leave the headquarters tomorrow.
It did not rain at all Last month.
He has always helped you.
The climate of this place is not good for your health.

USED TO
The use of (used to): It indicates something that happened regularly in the
past but does not happen now:
1) Subject +used to + infinitive:
He used to live in Sanaa, but he does not live now.
He did not use to live in Sanaa, but he lives now.
Where did he use to live?
2)look to the difference between the following;
0 He used to live in Sanaa. (= He does not live now)
1 He is used to living in Sanaa. (= He is accustomed to living in Sanaa.)
2 He got used to living in Sanaa. (=He got accustomed to living in
Sanaa)
Write the correct form of ( USED TO ):
0 I (smoke), but I (give it up) last year.
1 I (not like) him, but then I (change) my mind.
2 He (live) in London before he (go) aboard.
The cost of this book is to pray for me you have the permission to print it or do whatever you want

Get to the heart of English language grammar in easy way

40 of 195

3 (you travel) a lot before you (get) this job.


4 I (drive) a lorry before I (short) this business.
Complete the following:
Tom used to smoke but now .
Ann did not use to be fat but now .
I now he does not play tennis now but did ..?
John is not rich now but
Choose the correct word:
He.. travel a lot.
(used to, is used to)
He .. Getting up early.
(used to, is used to)
He used to .. polite, but he is not now.
(is, was, be)
He did not study every day.
(used to, is used to, use to)

Summary chart of modals and similar expirations:


Auxiliary

May

Might

Uses
1) Polite request
2) Formal
permission
3) Less than 50%
certainty
1) Less than 50%
certainty
2) Polite request
(rare)

Present/future
May I borrow your pen?

Past
XXXXXXXXXX

You may leave the room.

XXXXXXXXXX

1) Advisability

You should study tonight

Should
2) 90% certainty
Ought to

1) Advisability

He maybe at the library


He might be at the
library.
Might I borrow your
pen?

He may have been


at the library.
He might have been
at the library
XXXXXXXXX

You should have


studied last night
She should do well in the
She should have
test. (Future only)
done well in the test.
I ought to study tonight
I ought to have
studied tonight

The cost of this book is to pray for me you have the permission to print it or do whatever you want

Get to the heart of English language grammar in easy way

2) 90% certainty

Had better

Advisability with
threat of bad result

Be supposed
to

Expectation

To be

Strong expectation
1) Strong necessity

Must

Have to
Have got to

Will

You had better be on


time, or we well leave
without you
Class is supposed to
begin at 10.
You are to be here at
nine.
You must go to class

2) Prohibition
(negative)

You must not open that


door
Mary is not in class. She
3) 95% certainty
must be sick.
I have to go to class
1) Necessity
today.
It is silly of you to go out
2) Lack of necessity
if you dont have to.
Necessity
I have got to go to class.
He will be here at six
1) 100% certainty
(Future only)
The phone is ringing. I
2) Willing
will get it.
Will you please pass the
3) Polite request
salt?
He is going to be here at
1) 100% certainty
six. (Future only)

Be going to

Can

She ought to do well on


the test. (Future only)

2) Definite plan

I am going to help him


after I finish my work

1) Ability/
possibility

I can run fast

2) Informal
permission
3) Informal polite
request
4) Impossible
(negative only)

41 of 195

She ought to have


done well on the
test.
XXXXXXXXX
Class was supposed
to begin at ten.
You were to be here
at nine.
I had to go to class
yesterday
XXXXXXXXX
Mary must have
been sick in class.
I had to go to class
yesterday
You did had to go to
class yesterday
I had to go to class
XXXXXXXXX
XXXXXXXXX
XXXXXXXXX
XXXXXXXXX
I was going to paint
my room, but I did
not have time.
I could run fast
when I was child,
but now I can not

You can use my car


tomorrow.

XXXXXXXXX

Can I borrow your pen?

XXXXXXXXX

That can not be true!

That can not have


been true!

The cost of this book is to pray for me you have the permission to print it or do whatever you want

Get to the heart of English language grammar in easy way

1) Past ability
2) Polite request
Could

3) Suggestion
4) Less than 50%
certainty
5) Impossibility
(negative only)

Be able to

Ability
1) Polite request

Would
2) Preference

Used to

Shall

2.25.1.7.

XXXXXXXXXXX
Could I borrow your
pen?
I need help in math.
You could talk to your
teacher
He could be at home
That could not be true!
I am able to help you. I
will be able to help you.
Would you please pass
the salt? Would you
mind helping me?
I would rather go to the
park than stay home.

3) Repeated action
in the past

XXXXXXXXXXX

Repeated action in
the past

XXXXXXXXXXX

1) Polite question
to make a
Shall I open the window?
suggestion
2) = Will with
(I
I shall help him if he
& we)
asked.

42 of 195

I could run fast


when I was child
XXXXXXXXX
You could have talk
to your teacher.
He could have been
at home
That could not have
been true
I was able to help
him
XXXXXXXXX
I would rather have
gone to the park.
When I was a child,
I would visit my
grandparents every
week.
I used to visit my
grandparents every
week
XXXXXXXXX

The degree of certainty:


1-) (In present)
(100%) He is sick.
(95%) He must be sick.
He may be sick.
(50%) He might be sick.
He could be sick.
He can not be sick.
(100%) He is not sick.
(99%) He could be sick.
He can not be sick.

The cost of this book is to pray for me you have the permission to print it or do whatever you want

Get to the heart of English language grammar in easy way

43 of 195

(95%) He must not be sick.


He may not be sick.
(50%) He might not be sick.
2.25.1.8.

2-) (In past)


(100%) He was sick.
(95%) He must have been sick.
He may have been sick.
(50%) He might have been sick.
He could have been sick.
(100%) He was not sick.
(99%) He could not have been sick.
He can not have been sick.
(95%) He must not have been sick.
He may not have been sick.
(50%) He might not have been sick.

(3-) (In future)


(100%) He will help us.
(95%) He should help us.
He ought to help us.
He may help us.
(50%) He might help us.
He could help us.
Use a modal or similar expression with each verb in parentheses. More than
one auxiliary may be possible. Use the one that seems most
appropriate to you.
0 It looks like rain. We (shut) .. the windows.
1 Ameen, (hand, you) me that disk? Thanks.
2 I returned a book to the library. It was two weeks overdue, so I
(pay) .. a fine of $1. I (return) the book
when it was due.
3 Spring break starts on the thirteenth. We (go, not) .. ..
to classes again until the twenty second.
4 You (tell, not) .. Anees about the party. It is a
surprise birthday party for him.
5 A) Mrs. Wilson got a traffic ticket. She did not stop at stop
sign.
B)
That surprising. Usually she is very cautious driver and obeys all
the traffic laws. She (see, not) the sign.
The cost of this book is to pray for me you have the permission to print it or do whatever you want

Get to the heart of English language grammar in easy way

44 of 195

6 A) You are always too tense. It is not good for you. You (learn)
to relax.
B) how?
A) pay attention to your muscles. When they are tight, take a
long, deep breaths. Deep breaths (be) very relaxing.
7 Im sleepy. I (keep, not) .. my eyes open. I (go)
. To bed before I fall asleep right now.
8 In my country, a girl and boy (go, not) out on
a date unless they are accompanied by a chaperone.
9 Sami was serious when he said he wanted to be a doctor when
he grew up. We (encourage) . Him. We (laugh, not)
.. at him.
10
This is none of his business. He (stick, not) .
his nose into other peoples business.
11
My wife and ten children are coming to join me here.
They (live, not) . In my dormitory room.
I (find) .. an apartment.
12
A) (speak, I) to Mr. Yasser Al-Ahdal?
B) He (come, not) .. to the phone right now.
(take) .a message?
13
A) where are you going?
B) I (go) .. to the library. I have to do some
research for my term paper.
14
A) How are you planning to get to the airport?
B) by taxi.
A) you (take) . A shuttle bus instead. It is
cheaper than a taxi. You (get) . One in front of
the hotel. It picks up passengers there on a regular schedule.
15
A) This is a great open-air market. Look at all this
wonderful fresh fish. What kind of fish is this?
B) I am not sure. It (be) .. ocean perch. Lets
ask.
16
A) Did you enjoy the movie last night?
B) it was okay, but I (stay) .. home and (watch)
. TV. There was a good program on , that I
wanted to catch. I only went because my wife wanted to see the
movie.
17
A) Some body called you while you were out, she did not
leave her name.
B) Who did it sound like? Any body you know?
A) Well, it (be) Sami Al-Ahdal, but thats just
a guess. I (ask) . Who was calling, but I did not.
B) thats okay.
18
A) the phone is ringing again. Lets not answer it. Just let
it ring.
The cost of this book is to pray for me you have the permission to print it or do whatever you want

Get to the heart of English language grammar in easy way

45 of 195

B) no, we (answer) .. it. It (be) .


important.
19
The teacher called on Bassim yesterday, but he kept
looking out the window and did not respond. He (daydream)

2.25.1.9.

Fill in the blank space with may or might and indicate the sentences in
which both are possible.
2.25.1.9.1.1.1.
It .. rain; youd better stay at home.
2.25.1.9.1.1.2.
. I borrow your bicycle?
2.25.1.9.1.1.3.
Students. Not talk to each other in the examination.
2.25.1.9.1.1.4.
If you tell him of your need, he help you.
2.25.1.9.1.1.5.
If they arrive their early, they. Catch the train.
2.25.1.9.1.1.6.
. I see your ID card, please?
2.25.1.9.1.1.7.
They arrive next week.
2.25.1.9.1.1.8.
Where. Meet them?
2.25.1.9.1.1.9.
We. Never meet again.
2.25.1.9.1.1.10. He.cheating us.

2.25.1.10.
Fill in the blank space with could or was/were able to and
indicate the sentences in which both are possible.
2.25.1.10.1.1.1. He was very clever; he. Solve any problem.
2.25.1.10.1.1.2. The player was very tired but he.. win the race.
2.25.1.10.1.1.3.
When he was six years old,
he.. speak second language besides his native
one.
2.25.1.10.1.1.4. I.. find the money I had lost.
2.25.1.10.1.1.5. He. Escape when the guards were all asleep .
2.25.1.10.1.1.6.
Adel Omairan
visit his friends whenever his mother let him.
2.25.1.10.1.1.7. He play tennis before he broke his arm.
2.25.1.10.1.1.8. He buy that book because he had no money. (not)
2.25.1.10.1.1.9.
They talk to
teach other because the telephone was out of order.(not)
2.25.1.10.1.1.10. He.. catch the train, because he went to the
station early.
2.25.1.11.
2.25.1.11.1.1.1.
2.25.1.11.1.1.2.
2.25.1.11.1.1.3.

Fill in the blank space with must or appropriate tense of have


to.
He feels sick, he. Visit the doctor.
You answer all the questions.
She felt tired and leave early.

The cost of this book is to pray for me you have the permission to print it or do whatever you want

Get to the heart of English language grammar in easy way

46 of 195

2.25.1.11.1.1.4.
When you go to Russia next month,
you .. put a heavy clothes.
2.25.1.11.1.1.5.
My cars out of fuel. I take a taxi.
2.25.1.11.1.1.6.
I dont have enough money, so I. Pay by check.
2.25.1.11.1.1.7.
Workers get up early.
2.25.1.12.

Fill in the blank space with mustnt or neednt and then change them
into negative using can not & couldnt.
you.. wait any longer; the doctor will not come today.
You.. touch the post; its newly painted.
You.. answer all the question; two will do.
You talk to the students sitting next to you in the
examination.
We.. these vegetables; we can eat them raw.
We.. tell lies; it is immoral to do so.
You. Take these pills; you are quite health now.
You.. drive fast; we still have plenty of time.
You smoke in this crowded place.
You.. drive fast; the road is slipper.
She can not come to the phone because she (wash)
her hair.
(Lock, you, always) .the door to your apartment
when you leave?
Tom is a student, but he (go, not)to school right now
because it is summer. He (attend) ..collage from
September to May every year, but in the summers he (have,
usually)..a job at the post office. In fact, he (work)
.there this summer.
No, I (talk, not).about her. I (mean) .The
woman who ( wear). The blue suit. When I (open)
.. The door, I (find)a surprise.
He went to his friends house, but they (be, not)there.
They (play)..soccer in the vacant lot down the street.
Nowadays I (have). Trouble.
A) Look! It (snow)..

2.25.1.13.

Change the following into negative.


2.25.1.13.1.1.1. I dare tell them the truth.
2.25.1.13.1.1.2. They dared interrupt the judge.
2.25.1.13.1.1.3. We used to spend a few hours their.
2.25.1.13.1.1.4. I need to buy new clothes.
2.25.1.13.1.1.5. We have to abide by these regulations.
2.25.1.13.1.1.6. The patients had to stay in hospital for two more
weeks.
2.25.1.13.1.1.7. You ought to buy a new car.
2.25.1.13.1.1.8. He can do without meat.
The cost of this book is to pray for me you have the permission to print it or do whatever you want

Get to the heart of English language grammar in easy way

47 of 195

2.25.1.13.1.1.9. We must hurry.


2.25.1.13.1.1.10. We have finished discussed the auxiliaries.

MOOD
Mood : is the mode or manner in which the action denoted by the verb
There are three moods in English:
1) Indicative. {Statement of fact or assertion, interrogative, expressing
a supposition)
2) Imperative. {Command, exhortation, and entreaty or prayer}
3) Subjunctive. {Present and past}
When we make a sentence:
We name a person or thing; and
We should say something about that person or thing.
Hence every sentence has two partsThe part, which names the person or thing, we are speaking about.
This is called the Subject.
The part which tells something about the person or thing.
This is called the Predicate.
A) Subject: is the doer who performs the action and start from the beginning of
the sentence until the beginning of the verb like:
A noun like { Osama or Maram}
A pronoun like { he or she}
A noun phrase { the old building}
**(Note) The subject of a sentence can be a hidden subject or one word
or more and usually comes first, but occasionally it is put
after the predicate; as in,
Be calm. (The subject is hidden) =(you be calm)
Ali is a student. (One subject only)
Talal Alkawlani is a friend of mine.
My friend Abdulraheem Al-Ethawi is a kind person.
Here comes the bus.
**(Note) in imperative sentences the subject is left out; as in,
Sit down. Thank him.

The cost of this book is to pray for me you have the permission to print it or do whatever you want

Get to the heart of English language grammar in easy way

48 of 195

A n Object : Is the person or thing that is affected by the action and it


can be:.
Direct object (like, I played basketball.)
Indirect object (like, I bought him a bike)
Object of preposition (like, I live in Sanaa)
Noun phrase (like, I met )
The verb must agree with the subject so the subject dictate the form of
the verb (e.g. I wait, he waits, I am, he is, I have, he has) this
agreement is often called concord.
Sentences Order:
Subject + Verb:
My head aches.
Subject + Verb + complement:
He is happy.
Subject + verb + direct object:
I hate you.
Subject + verb + indirect object + direct object:
He sent his father a letter.
Subject +verb + object +complement:
We made him chair man.
Kinds of sentences according to their mood :
Statements or assertions;
Example:
He is happy.
Interrogative;
Example:
Where are you from?
Expressing a supposition:
If it rain, I will stay at home.
Express request;
Example:
Would you help me, please?
Imperative;(express command) and is used in the second person, since
the person commanded must be the addresser; as,
Be quiet. (= You be quiet)
The cost of this book is to pray for me you have the permission to print it or do whatever you want

Get to the heart of English language grammar in easy way

49 of 195

**(Note) It can be used in the first and third persons; alike sense is
expressed by the use of the Auxiliary Verb let; as,
Let me go.
Let him go.
Let them go. Let us go.

Exhortation; as,
Be steady.
Take care of yourself.
Try to do it better.
Express entreaties or prayer;
Example:
Have mercy upon me.
Exclamation;(express strong feelings)
Example:
How cold the night is!
What a shame!
{Note} if we are going to talk about quantity, we should use (what a lot)
instead of (how) e.g.
what a lot of flowers!
{Subjunctive} it has no present, past, or future form and it is used to
stress importance,
Verbs followed by subjunctive;
Ask (that)
Suggest (that)
It is necessary
(that)
It is imperative
(that)

Request (that)
Recommended (that)
It is important
(that)
Demand (that)

Insist (that)
Propose (that)
It is vital (that)
Advice (that)

(Note) The subjunctive is more common in American English than


British English. In British English, should +simple form is more
usual than the subjunctive: e.g., The teacher insists that we should
be on time.
The teacher demands that we be on time.
I insisted that he pay me the money.
I recommended that she not go to the concert.
It is important that they be told the truth.
They requested that we not work after midnight.
He insisted that the new baby name after his grandfather.
The cost of this book is to pray for me you have the permission to print it or do whatever you want

Get to the heart of English language grammar in easy way

50 of 195

The student requested that the test postpone.


It is vital that no one else know about the secret.

Point out the verbs in the following sentences and name their Moods and
tense:The river flows under the bridge.
I will answer the letter to night.
I new he was there, for I had seen him come.
It has been raining all night.
I had finished when he came.
Be good, sweet maid.
By this time tomorrow I will have reached my home.
It is time we left.
He told me that he had finished the work.
God forgive you!
He is waiting for you in the compound.
The king had never before led his troop in battle.
Complete the following. In many of the sentences there is more than one
possible completion: Mr. Kamal Suraim insists that we . Careful in our
writing.
They requested that we not after midnight.
She demanded that I .. her the truth.
I recommended that Bassim Al-Jaifi to the head of the
department.
I suggest that everyone . A letter to the government.
It is essential that I you tomorrow.
It is important that he the director of the English
program.
It is necessary that everyone .. here on time.
Give the correct form of the verb in the parentheses. Some of the verbs
are passive: Her advisor recommended that she (take) . . Five
courses.
He insisted that the new baby (name) . After his father.
The doctor recommended the she (stay) . In bed for few days.
The students requested that the test (postpone) .. , but
the instructor decided against the postponement.
I requested that I (permit) to change my class.
It is essential that pollution (control) and eventually
(eliminate)
The cost of this book is to pray for me you have the permission to print it or do whatever you want

Get to the heart of English language grammar in easy way

51 of 195

It was such a beautiful day that one of the students suggested we


(have) . Class outside.
The movie director insisted that everything about this productions
(be) . Authentic.
It is important that you (be, not) late.
It is imperative that he (return) . Home immediately.
THE OBJECT

A n Object : Is the person or thing that is affected by the action.


I helped him.
I played basketball.
NO
1
2
3
4
5
6
7

Pronoun
I
You
We
He
She
They
It

Its object
Me
You
Us
Him
Her
Them
It

Object in general:
An object is normally;
1) direct object
: I met Ahmed.
2) indirect object
: I sent him a letter.
3) a phrase
: I found a wonderful watch.
a reflexive pronoun : I saw myself in the mirror.
Object of preposition : I feel sorry for him.
Object after be
: who broke the window? its me or not me
As a subject
: you can tell him me tell him? Not likely!
With comparative
: he is not taller than me. (I am)
In exclamation
: she has been promoted. Lucky her
She is got to repay the money. poor her
Adverbial object or accusative: they are nouns, which denote( time, place, distance,
weight, value etc): as,
I can not wait a moment longer.
He went home.
he weighs 80 K.
He swam a mile.
The watch costs five dollars.
The cost of this book is to pray for me you have the permission to print it or do whatever you want

Get to the heart of English language grammar in easy way

52 of 195

There are two kinds of objects;


Direct object:
{Refers to a person or a thing that effected by the action.}
I kicked him.
I put out the cigarette.
Indirect object:
{ refers to a person who benefits from the action.}
I gave him a gift.
I gave a gift to him.
Indirect object takes variety of forms, some of which;
A noun: we bought a book for Amer.
A pronoun: we bought it for him.
Write the correct form of pronoun in the following:We scored as many goals as ..(they)
Yasser and were absent. (I)
Can you sing as well as ..? (they)
Wait for Sami and (I )
It was that give you the alarm. (I )
Between you and ., I dont believe ..(I, he)
Nobody but was present. (he)
He and were great friends. (I )
Whom I can trust, if not .? (he)
It is not for such as . To dictate to us.(they)
Indirect object comes after direct object with the following verbs:
{ Indirect object+ to +
Announce
Demonstrate
Introduce
Recommend
Bequeath
Mention
Promise
Suggest

Show
Take
Teach
Report
Throw
Write
Entrust
Declare

Sing
Pay
Repeat
Read
Post
Confess
Describe

Send
Leave
Serve
Lend
Play
Offer
Give
Admit
Grant
Bring
Hand
Prove
Explain
Propose
+Direct object.

Owe
Tell
Sell
Say
Pass
State
Confide

The cost of this book is to pray for me you have the permission to print it or do whatever you want

Get to the heart of English language grammar in easy way

53 of 195

We admitted to him.
I confessed to the lawyer.
I mentioned it to you.
I explained the lesson to you.
Indirect object comes after direct object with the following verbs:
{ Indirect object+ for +
Buy
Do
Fix
Get
Save

Call
Cut
Find
Sing

Catch
Change
Make
Bring
Order
Reach
Reserve
Leave
+ Direct object

Cook
Keep
Fetch
Build

He brought a bag for me.


He fetched a Pepsi for me.
He did it for me.
He fixed a cup of tea for me.
5-3) The following verbs can be used in two ways; {Buy/ brought/give}
He bought me a present.
He bought a present for me.
He gave me a present.
He gave a present to me.
He brought a present to me.
He brought me a present.
6-3) The following verbs can be used without (To/For)
Ask
Grant
Show

Bet
Owe
Pay

Teach
Tell
Write

Forgive
Promise

He taught us English.
He showed us a picture.
He asked me a question.
The cost of this book is to pray for me you have the permission to print it or do whatever you want

Get to the heart of English language grammar in easy way

54 of 195

He forgave me what I said.


7-3) The following verbs can be used as the following:
Brin
g
Buy
Fetch

Give

Send

Hand
Pass

Show
Teach

Give me it.
Give me them.
Show me it.
Fetch me them.

PRESENT COUNTINUOUS
It talks about an action that is happening at the
Same time of speaking.
The present continuous is used
For an action going on at the same time of speaking; as,
He is eating his breakfast now.
For a temporary action which may not actually happening at the same time
of speaking; as,
I am reading David Copper field
(but I am not reading it at this moment)
As advice, surprising, annoyance or warning, it can be used with (always,
continually and constantly); as,
My dog is very silly; he is always running
out into the road.
You are always putting your shoes in my room.
With a situation or an action that takes a limited time; as,
The price of oil is growing these days.
I am studying English nowadays.
0

To suggest a gradual development; as,


It is getting dark.
He is becoming more and more depressed.
1) The structure of the tense;
I

+ am
He, she, it

+ is
They, we, you
+ are

+ verb + ing.

The cost of this book is to pray for me you have the permission to print it or do whatever you want

Get to the heart of English language grammar in easy way

55 of 195

Example:
I am helping my father at the moment.
He is playing football now.
They are fighting each other at the present.
2) How to change the sentence into negative
Subject+ be+ verb+ ing
Subject +be +not +verb+ ing
Example:
He is eating.
He is not eating
Choose the correct answer;
I (answer, answering, am answering) the question.
At the present time my father (help, is help, is helping) me.
You (is welcoming, am welcoming, are welcoming) the new teacher now.
She (not is cleaning, is cleaning not, is not cleaning) her room.
3) HOW TO MAKE QUESTIONS
(note) to change a sentence into a question, you should first study the
following:
No

Word

Ask about
Name of
person
Something
Selection
Name of
place

1)

Who

2)
3)

What
Which

4)

Where

5)

Why

Reason

6)
7)

When
Whose

Time/date
The owner

8)

How

Health

9)

How

The way

10)

How
many

Number

11)

How
much

Quantity

Example
Who is that man standing at the
window? Its Ahmed)
What hit you? It is Ahmads car
Which book do you want? (That one)
Where are you? I am in the house
Why did not you help him? I was
busy.
When did you come? now
Whose book is this? it is mine.
How are you?
I am fine
How do you find it?
It is nice.
How many friends do you have?
I have a lot of friends.
How much did you pay for this
coat?
I paid five dollars.

The cost of this book is to pray for me you have the permission to print it or do whatever you want

Get to the heart of English language grammar in easy way

No

Word

Ask about

12)

How old

Age

How far

Distance

How
long
How
high

Length of
things
Height of
things
Height of
people

How tall
How
deep
How
wide
How
often

56 of 195

Example
How old are you?
I am 28.
How far is it from Sana'a to
Aden?
It is about 360 km.
How long is the rope?
It is too long.
How high is the wall?
It is about five matters.
How tall is he?
He is five feet.

Depth

How deep is the river?


It is too deep.

Width

How wide is the street?


It is ten matters.

Times

How often do you go there?


Twice a week.

In some expressions as what are you? the word what does not refer to
the person, but to his employment or sex.
What are you? (I am an engineer.)
What are you? (I am a man.)
1-3)Yes/ no questions;
Subject +be + verb + ing
Be +subject +verb + ing?
Example:
He is eating.
Is he not eating?
Is, Am, Are + subject +verb + ing?
He is drinking his juice.
Is he drinking his juice?
Are they smoking?
Am I explaining the lesson?
2-3) Wh/ questions;
Wh/questions ={what, where, why, when, who}
(1)
(2)
(3)

(4)

The cost of this book is to pray for me you have the permission to print it or do whatever you want

Get to the heart of English language grammar in easy way

57 of 195

Who +is, am, are +verb +ing?


What is he do ing?
He is running.
Where am I going?

You are going home


Who is eating?

Maram is eating.
Why are you crying?

Because I am sad.
When is he writing?

He is writing now
Change the sentence into question:
I am building a house.
He is shouting.
We are fighting each other.
She is not swimming.
You are always staying up late.
It is eating its food.
We are not helping each other.
Ali and Fatima are shouting at each other.
Khaled and his dog are running.
Aiman and Anees are fighting each other.
{NOTE} To answer a question starts with (yes/no question) be careful to the
following in any sentence.
If the question has the pronoun (you) in answer it must be changed into( I /
we) depending in the intended meaning if singular we will use (I ) but
if plural we will use ( we ) and conversely.
Example:
Are you studying?
Yes, I am.
Are you studying?
Yes, we are.
I .?
You? We.? You.
Am I studying?
Yes, you are.
Are we studying?
Yes, you are.
{Note} we cannot contract the helping verb in short answer; as,

Yes, he is. But not (yes, hes.)


4) How to add ing
0 Verbs end in (e)
The cost of this book is to pray for me you have the permission to print it or do whatever you want

Get to the heart of English language grammar in easy way

1
2
3
4
5

58 of 195

Come
coming
Verbs end in two consonant letters;
Work
working
Verbs end in (ee)
See
seeing
Verbs end in (ie)
lie
lying
Verbs end in one vowel + one consonant letter;
Run
running
Verbs end in (y)
play
playing
(NOTE) We can not double the last letter if the word ends in
Blowing. Fixing. Enjoying.

(w, x, y, or z)

{NOTE} we can use this tense with adjective:


You are being foolish. =(Only in this situation)
You are foolish. =(It is one of your character.)
{NOTE} We can use always with this tense.
You are always coming late. =(I dont like your coming late)
You always come late. = (We are not surprised for this)

The cost of this book is to pray for me you have the permission to print it or do whatever you want

Get to the heart of English language grammar in easy way

59 of 195

PAST CONTINUOUS
IT TALKS ABOUT AN ACTION THAT CONTINUED FOR
SOME TIME IN THE PAST

He was reading his book.

THE USES OF IT
1
Two actions happened in the past at the same time but the one in the past
continuous happened earlier.
0 Example;
I was walking down the street when the rain started.
When the rain started, I was walking down the street.
2
3

To suggest a gradual development; as,


It was getting dark.
He was becoming more and more depressed.
With stories description past continuous used for description and past simple
with narrative; as,
I was sitting in my chair. My son was snoring in the bed in the adjacent room.
Suddenly some one knocked at the door. I opened the door and found to my
surprise Mr. Nabeel Amer standing there.
4 To indicate a repeated action causing annoyance, e.g.
--> He was always writing rude letters to me.
5 Two actions in progress simultaneously in the past.
--> While I was walking, it was raining.
--> It was raining while I was walking.
0

THE STRUCTURE OF IT
I, he, she, it
You, they, we

was
+
were

+ Verb +ing.

The cost of this book is to pray for me you have the permission to print it or do whatever you want

Get to the heart of English language grammar in easy way

0
0
1

60 of 195

Example;
He was eating.
They were fighting each other.
How to change the sentence into negative :
0
Subject +be+ verb+ ing

Subject +be +not +verb+ ing


1
Example:
He was eating.
He was not eating
Choose the correct answer;
I (was answering, answer, were answering,) the question.
My father (was help, were helping, was helping) me.
You (are welcomed, were welcoming, are welcoming) the new teacher.
She (not was cleaning, is not cleaning , were not cleaning, was not
cleaning) her room.
How to change the sentence into question:
2
3

1
0
0
1

Subject +be + verb


Be +subject +verb
Example:
He was eating.
Was he not eating?

Yes/ no questions;
Was, Were + subject +verb + ing?
Was he drinking his juice?
Were they smoking?
0
1
2
3
4
5
1

Wh/ questions;
Wh +was, were +verb +ing?
What was he doing?
He was reading.
Where was I going?
You were going to the movie.
When were you writing?
I was writing when you wear phoning.
Why were they laughing?
Because you were crying.
Who was dancing?
The model was dancing.

The cost of this book is to pray for me you have the permission to print it or do whatever you want

Get to the heart of English language grammar in easy way

61 of 195

Change the sentence into question:


I was building a house.
He was shouting.
We were fighting each other.
Add an ( ing) to the following:
Agree .. hope.. run. Study. Prefer.

PRESENT SIMPLE
It talks about everything in general
Example;
6 STATEMENT FACTS:
Water consists of hydrogen and oxygen.
7 HABIT:
I always drink tea after lunch.
8

EVERYDAY ACTIVITY:
He usually plays tennis.

To refer to time of speaking; as,


He wants you to come now.
It is hot here.
He is in the kitchen.
You are clever.

10 With official programs:


The collage opens on June 23rd.
The film starts at 5:00.
11 For future in clauses with if; as,
If I met him, I will give him a lesson.
12

To refer to future time in subordinate clause, e.g.


we will start when you are ready.

13
14 With sentences begin with (here/there); as,
Here he comes.
Here comes the boss.
There he goes.
The cost of this book is to pray for me you have the permission to print it or do whatever you want

Get to the heart of English language grammar in easy way

62 of 195

There goes the train.

No
(1)

(2)

(3)

(4)

15 STATE VERBS: are those without action like;


Mental
Existing
Emotional
Possession Sense
Know,
Consist Of,
Realize,
Love, Like,
Contain,
Possess
Hear
Understand,
Appreciate
Include
Recognize
Believe, Feel,
Suppose,
Be, Exist
Hate, Dislike
Belong
Feel
Think
Imagine,
Doubt,
Owe, Cost,
Taste,
Fear, Envy
Have
Remember,
Weigh
Smell
Forget
Want, Need,
Seem, Look,
Mind, Care
Own,
See
Prefer, Mean
Appear
The former verbs cannot be used in the continuous tense if they mean
state verbs, but if they have another meaning, they can be used in
present continuous.

Here are some adverbs used with this tense;


{Always, everyday/month/year, Often, Usually, Sometimes, Rarely, Never}
1
The structure of it
i. He, she, it
(adverb) + verb + s + ...
ii. I, You, they, we (adverb) + verb +..
Example;
0
He (always) eats his lunch.
2) They (never) fight each other.
How to change the sentence into negative

He/she/it +does +not +verb


We/you/I/they+ do+ not+ verb.
(note) in negative we should not add (S) to the (Verb)
Example:
He eats his breakfast.
The cost of this book is to pray for me you have the permission to print it or do whatever you want

Get to the heart of English language grammar in easy way

63 of 195

He does not eat his breakfast.


They eat their breakfast.
They dont eat their breakfast.

0
1

Choose the correct answer;


I (answer not, not answering, dont answer)
the questions.
My father (not help, do not help, does not help) me.
You never (welcoming, welcome, are welcoming) the new teacher.
She (cleaning, is cleaning, cleans) her room everyday.
He (is caning, cans, can) lift that box.
She does not( had, has, have) a car.
HOW TO MAKE QUESTIONS:
Yes/ no questions;
If you want to make question in present simple Add (do/does) to the
beginning of the question if there is no auxiliary and we
must not add (S) to the (VERB);
Do +I, you, they, we +verb +?
Does +he, she, it + verb +?
He always helps his family.
Does he always help his family?
2

You must not add (s) or any change to the verb and use (have)
instead of (has) .see examples below;
He has a car.
Does he has a car? (incorrect)
Does he have a car? (correct)
He runs fast.
Does he runs fast? (incorrect)
Does he run fast? (correct)

Example;
Do you play tennis?
Yes, I do. Or ( yes I play tennis.)
Does he play tennis?

Yes, he does. Or yes, he plays tennis.

Yes, I have.

Do you have a car?


The cost of this book is to pray for me you have the permission to print it or do whatever you want

Get to the heart of English language grammar in easy way

64 of 195

Does he have a car?

Yes, he has.

Yes, I can.

Can you eat this meat?


Can he eat this meat?
0

Yes, he can.

Change the sentence into question:


o I build a house.
o He shouts.
o We fight each other.
2) Wh/ questions;
0 Wh +do/ does +verb +?
What does he do?
He is a teacher.
Where does he live?
He lives in Al-Horia Street.
When do you go there?
I go there every other day.
Why does she come to school?
Because she wants to study.
{NOTE} in this tense (who) is used as the following;
Either
who + verb + s +object?
Example:
who plays football?
Osama plays football.
Or
who+ do/does +subject +verb+ preposition?
Example:
Who do you talk to?
I talk to Manal.

The cost of this book is to pray for me you have the permission to print it or do whatever you want

Get to the heart of English language grammar in easy way

65 of 195

SIMPLE PAST
IT TALKS ABOUT AN ACTION THAT HAPPENED
IN
0

A PARTICULAR TIME IN THE PAST

0 HERE ARE SOME ADVERBS show that the sentence in the past simple:

{Yesterday, last week, last month, last year, ago, as, when, as soon as}
It is used :
1 When the questioner wants to find out the time of the action; as,
*** When did you meet him last?
2 When the action took place at a definite time but this time is not
mentioned; as,
*** I bought this jacket from London.
*** The plane arrived four minutes later.
3 When the time of an action becomes known through the use of the
present perfect tense; as,
*** Where have you been? I have been to Nabeels house. I
went there to convey your message to him. did you meet
him there? -no, I did not. He was not at home.
4

Sometimes the adverbs are not used if the time here might be implied or
indicated by the context; as,
I learnt English in America.
I am so tired I could not sleep well.

Verbs in the past are two kinds;


0
Regular verbs: those end in (ed).
Example; {play
played}
1

Irregular verbs: those have different shape from the present


Example; {know
knew}
OTHE STRUCTURE OF IT:

The cost of this book is to pray for me you have the permission to print it or do whatever you want

Get to the heart of English language grammar in easy way

Subject + verb in the past + ...

Example;

He ate his lunch two hours ago.


2) They helped each other last week.
3) I visited him yesterday morning.

HOW TO MAKE QUESTIONS


1) Yes/ no questions; {if you want to make question in past
simple, add (did) to the beginning of the question if there is
no helping verb;
Did
----He helped his family.
Did he help his family?

4
2
5
3

66 of 195

You must not add (ed) or any change to the verb and use (have)
instead of (had) . see examples below;
He had a car.
Did he had a car? (incorrect)
Did he have a car? (correct)
He played soccer.
Did he played soccer? (incorrect)
Did he play soccer? (correct)
2) Wh/ questions;
0 Wh +did +verb +?

Examples;
What did he write?

He wrote a story.

She went to the supermarket.

He got up at 7:00.

Because they saw a snake.

Where did she go?


When did you get up?
Why did they shout?
{NOTE} in this tense (who) is used as the following;
Either
who + verb in the past + object?
Example: who played football?
The cost of this book is to pray for me you have the permission to print it or do whatever you want

Get to the heart of English language grammar in easy way

67 of 195

Manal played football.


Or
who+ did+ subject +verb+ preposition?
Example: Whom did you talk to?
I talked to Ali.

0
0

1
2
3
4
5
6

How to add {ed} to the verb


Verbs end in (e)
Hope
hoping
Verbs end in two consonant letters;
Work
worked
Verbs end in (ee)
agree
agreed
Verbs end in (ie)
tie
tied
Verbs end in one vowel + one consonant letter;
Stop
stopped
Verbs end in (y)
play
played
Verb end in consonant + (y)
Reply
replied

(NOTE) We can not double the last letter if the word ends in (w, x, y,or z)
Plowed. Fixed. Enjoyed.

The cost of this book is to pray for me you have the permission to print it or do whatever you want

Get to the heart of English language grammar in easy way

68 of 195

PRESENT PERFECT
It talks about an action or situation that occurred for many
times in the past and it might be happened again, or it has never
happened so far.
0
0 An action happened more than one time in the past and it might occurred
again; as,
I have been to London twice.
0

An action that never happened so far;(not, never); as,


1 I have not eaten my breakfast.
(It means that there is still time to have it)
2 I have never visited him.
An action which we think of its effect; as,
He has eaten my breakfast.
I have cut my finger. It is bleeding.
I have finished my homework.

1 With the newspaper reports and news broadcasts the present perfect is often
used in the first sentence and the past simple in the following
sentences; as,
Twenty valuable bronze statues have been stolen from
the National Museum. The thief entered the Museum
through the back door, killed one of the night guards and
then disappeared with the statues.
2

To denote an action that happened at some time in the past and


continuing up to the present moment (for/since) is used; as,
I have known him for a long time.
I have been ill since last week.

Adverbs used with it; {already, for, since, just, yet, never, ever, so far, till
now, today, this week/month or year}; as,
He has already gone. (It means that the listener did not expect that)

The cost of this book is to pray for me you have the permission to print it or do whatever you want

Get to the heart of English language grammar in easy way

69 of 195

He has just gone.( completed action in the immediate past)


Have you ever eaten snails?
no, and I hope never will?

1 The structure of it:


0
He, She, It
+has + P.P.
1
I, We, They, You + have + P.P.
Example:
1) He has played tennis.
2) I have played tennis.
3) She has come for three days.
How to change the sentence into negative:
0 Subject +have/has +not +P.P
Example:
He has eaten.
He has not eaten.
2
Choose the correct word;
I (answer, has answered, have answered) the phone.
My father (had helped, has help, have helped) me.
You (has welcome, have welcome, have welcoming) the new teacher.
She (not has cleaning, has not clean, has not cleaned) her room.
HOW TO MAKE QUESTIONS;
Yes/ no questions; {if you want to make question in Present perfect,
you must do the following:
0
Have/has +subject +P.P. +?
Example;
They have played tennis.
Have they played tennis?
He has played tennis.
Has he played tennis?
2) Wh/ questions;
0
Wh +have/has +P.P. +?
Examples;
1.
2.
3.
4.
5.

What has he done?


Where have I gone?
When have you written?
Why have they cried?
Who played football?

The cost of this book is to pray for me you have the permission to print it or do whatever you want

Get to the heart of English language grammar in easy way

70 of 195

0 (Yet): comes at the end of a question or with negative sentence:


0
Have you written the letter yet?
1
I have not written the letter yet.
(Already): comes in the middle of the sentence to talk about unexpected
news;
a) Do your homework.
b) But I have already done it.
0 {Note} the use of (for & since): they express a situation that happened
in the past and continues to the present;
0
{Since + a particular time}.(or) with time which give
illogical meaning if we add (s) to the end of it.
Example:
I have been here since seven oclocks. (incorrect)
I have been here since seven oclock. (correct)

{For+ a duration of time}. (or)with time which give logical meaning


if we add (s) to the end of it.
Example:
I have been here for seven hours.
I have been here for one hour.

Add an (ed) to the following:


2
Agree .. hope.. play. Study. Prefer.
2.25.1.14. Fill in the blank space with since or for.
we have been walking one hour.
They have taught there. A long time.
I have stopped smoking I got sick.
She has been waiting noon.
Jamal has been in hospital.. the accident.
That old building has been there.. 300 years.
We have had very nice weather.. the last week.
He has disappeared.. yesterday morning.
I have been looking for my book. Ages.
. When has she been making such nice hat?

The cost of this book is to pray for me you have the permission to print it or do whatever you want

Get to the heart of English language grammar in easy way

71 of 195

PRESENT PERFECT CONTINUOUS


IT TALKS ABOUT AN ACTION THAT
HAPPENED IN THE PAST AND MAY BE
FINISHED OR NOT.

the signs of the action is still clear to see at the present time; as,
1) He has been playing football.
(Thats why his clothes are dirty)

Happened in the past, continued up to now and it may be take time in


future.
Example:
I have been studying English for six years.
=(I still study)
An action that still Changing or developing;
Example:
The weather has been getting warmer recently.
i.

How to change the sentence into negative:


Subject +have/has +not +been+ P.P
Example:
He has been eating.
He has not been eating.
Choose the correct word;
I (have answering , have answered, have
been answering) the phone.
My father (have been helping, has helping,
has been helped) me.
You (have welcoming, have welcomed,
have been welcoming) the new teacher.
She (has not been cleaned, has not cleaning,
has not been cleaned) her room.

The cost of this book is to pray for me you have the permission to print it or do whatever you want

Get to the heart of English language grammar in easy way

72 of 195

HOW TO MAKE QUESTIONS:


Yes/ no questions; {if you want to make question in present perfect
continuous, you must do the following;
4

Have/has +subject +been+ verb +ing +?


Example;
Have you been playing tennis?
Has he been playing tennis?
2) Wh/ questions;
1
(Wh) +have/has +been +verb +ing +?
2
Examples;
1. What has he been doing?
2. Where have I been going?
3. When have you been writing?
4. Why have they been crying?
5. Who has been playing football?

The cost of this book is to pray for me you have the permission to print it or do whatever you want

Get to the heart of English language grammar in easy way

73 of 195

THE PAST PERFECT


IT EXPRESSES AN ACTIVITY THAT WAS COMPLETED
BEFORE ANOTHER ACTIVITY IN THE PAST
3
i.
ii.

The action in the past perfect always happened first


and the second is in the past simple.
iii.
Example:
1) He had eaten the food before I came in.
2) After I came in, he had eaten the food.

{NOTE} When we use (before/after) it is preferred not to use the past


perfect, but we should use the simple past instead}
Example:
He ate the food before I came in.
4
The structure of it
1

Subject + had + P.P.


Example:
1) He had played tennis.
2) I had played tennis.
3) She had come for three days.

HOW TO MAKE
QUESTIONS
Yes/ no questions; {if you want to make question in
past perfect, you must do the following;
5
0
1
2
3

Had +subject +P.P. +?


Example;
You had played tennis.
Had you played tennis?
He had played tennis.
He had played tennis?
2) Wh/ questions;
0 Wh +had +P.P. +?
Examples;
i. What had he done?
ii. Where had I gone?

The cost of this book is to pray for me you have the permission to print it or do whatever you want

Get to the heart of English language grammar in easy way

74 of 195

iii. When had you written?


iv. Why had they cried?
v. Who had played football?

The future simple


It talks about an action that will take time in particular time
in the future
5
Here are some adverbs indicate the future;
{Tomorrow, next week, next month, next year, soon, in the days to come}
1 It talks about things which we cannot control:
I will be twenty-nine next month.
We will know our exam results in May.
2 We use it when we decide to do something at the same time of speaking.
*** It is raining. I will take the umbrella.
*** Mr. Smith is busy at the moment. All right. I will wait.
*** We use it with (I believe, think, expect, probably or I am sure)
*** I think we will win.
*** I am sure he will get the first class.
6

The structure of it;


Subject + will + verb +
Example:
I will answer the phone.

Subject + be + going + verb +..


Example:
I will answer the phone if it rings.

Subject + be + motional verb+ ing +..


Example:
I am traveling to Aden next week.
i. Subject + be + to + verb+ ..
Example:
The president is to meet the ambassador
tomorrow morning.
The cost of this book is to pray for me you have the permission to print it or do whatever you want

Get to the heart of English language grammar in easy way

75 of 195

HOW TO MAKE
QUESTIONS
Yes/ no questions; {if you want to make question in future
simple, you must do the following;
6
7

Will +subject + infinitive +?


Be + subject + going to + verb +?
Example;
Will you play tennis?
Is he going to play tennis?

2) Wh/ questions;
3 Wh +will +infinitive +?
4 Wh + be + going to + infinitive?
Examples;
1. What will he do?
2. Where am I going?
3. When will you write a letter to your family?
4. Why will they fight each other?
5. Who is going to play football?
6.
(Study ) the use of {will, be going to, be to, be }
0
WILL & BE GOING TO: are used to make a prediction
Example:
It will rain tomorrow.
It is going to rain tomorrow.

4
5

1 WILL but not (going to)to talk something without prior plan.
Example :
The phone is ringing. I will get it.
2
BE GOING TO but not (will) to talk something with
prior plan.
Example :
If the phone rings, I am going to get it.
3 BE TO : to talk about future plans or forecasts especially in
formal way like in newspaper.
Example :
The president is to pay visits to many Government circles.
The thick fog is to clear this afternoon.
BE ABOUT TO: used for the immediate future.
Lets get onto the train. It is about to leave.

The cost of this book is to pray for me you have the permission to print it or do whatever you want

Get to the heart of English language grammar in easy way

76 of 195

Dont go out now. We are about to have lunch.

5
MOTIONAL VERBS: when we want to talk
about future with verbs describe motion such as {go, walk, run,
travel, come, ..Etc} we are not need to use (going to/ will).
Example :
He is coming from school after two minutes.
(NOTE) Sometimes we use a past simple to talk about future; as,
I wish I could help you.

The cost of this book is to pray for me you have the permission to print it or do whatever you want

Get to the heart of English language grammar in easy way

77 of 195

Future continuous
IT TALKS ABOUT AN ACTION THAT IS TAKING TIME
IN PARTICULAR TIME IN THE FUTURE

We use it to talk about an action which will be in progress at a time in the future; as,
I suppose it will be raining when we start.
This time tomorrow I will be sitting on the beach in
Singapore.
Can I see you at 5 oclock?-Please dont come then.
I will be watching the tennis match on the TV.
i. With actions those already planned or expected; as,
The postman will be coming soon.
He will be meeting us next year.
ii. Here are some adverbs indicate the future;
{When, as}
When I get there, I will contact you
8
The structure of it;

Subject + will + be + verb +ing


Example:
When you come at midnight, I will be sleeping.

HOW TO MAKE
QUESTIONS
1
Yes/ no questions; {if you want to make
question in future continuous, you must do the
following;
Will +subject +be + verb +ing..?
Example;
Will you be play tennis at 10:00?
2) Wh/ questions;
0
Wh +will +be +verb +ing..?
Examples;
1. What will he be doing at seven oclock?
2. Where will he be going then?
3. When will you be writing a letter to your family after dinner?

The cost of this book is to pray for me you have the permission to print it or do whatever you want

Get to the heart of English language grammar in easy way

78 of 195

Future perfect

IT TALKS ABOUT AN ACTION THAT WILL BE


COMPLETED BEFORE ANOTHER ACTION IN THE
FUTURE
9
i. It is used to talk about an action that will be completed by a certain future
time; as,
ii. I will have written my exercise by then.
iii. He will have left before you go to see her.
By the end of this month I will have worked here for five years.
iv. Here are some adverbs indicate the future;{BY, WHEN}
10

9
4

The structure of it;


i. Subject + will + HAVE + p.p+
Example:
When you come at midnight, I will have
finished reading.
HOW TO MAKE QUESTIONS
Yes/ no questions; {if you want to make question in future
perfect, you must do the following;
Will +subject +P.P +..?
Example;
Will you have played tennis by the time I get there?
Wh/ questions;
1 Wh +will +Subject+ have + P.P+ ..?
Examples;
What will he have done by seven oclock?

The cost of this book is to pray for me you have the permission to print it or do whatever you want

Get to the heart of English language grammar in easy way

79 of 195

PAST PERFECT CONTINUOUS


IT TALKS ABOUT AN ACTION THAT BEGAN BEFORE A CERTAIN

11 POINT IN THE PAST AND CONTINUED UP TO THAT TIME.

Example:
At the time he had been writing a novel for two months.
When Mr. Smith came to school in 1995, Mr. Amer had already
been teaching there for five years.
0 USE THE CORRECT TENSE:
It is beautiful! This is the first time I have ever seen snow. It
(snow, not) ..in my country.
I have this book (for/ since)..May 1.
I (own, not) ..an umbrella. I (wear) .a
waterproof hat on rainy days.
Right now I (look) around the classroom. Ali (write)
..in her book. Aml (bit) her pencil. Adel Said
(scratch) ..his head. Adel Hussain (stare) out
the window. He (seem) .to be daydreaming, but
perhaps he (think) .hard about verb tenses. What (think,
you) . Sameer Bakri (do) ?
Barbara (tutor, often) .other students in her Math
class. This afternoon she (help) Homadi with his math
assignment because he (understand, not) .the
material they (work).on in their class this week.
I (call) .. Osama Al-Madani at nine last night, but he (be,
not) .at home. He (study) at the library.
I really (enjoy) ..my vocation last January. While it
(snow) the sun (shine) ..in Florida.
While you (shovel) .snow in Iowa, I (lie) ..on
the beach in Florida.
It was my first day of class. I (find, finally) the right
room. The room (be, already) ..full of students. On
one side of the room, students (talk, busily) to each
other in Spanish. Other students (speak) .
Japanese. And some (converse) in Arabic. It
(sound) . Like the United Nations. Some students, however,
The cost of this book is to pray for me you have the permission to print it or do whatever you want

Get to the heart of English language grammar in easy way

80 of 195

(sit, just) quietly by themselves. I ( choose)


.. an empty seat in the last row and (sit) .. down.
In a few minutes, the teacher (walk) into the room
and all the multilingual conversation (stop) ..
What (learn, you) ..since you (come) ..
here? How many new friends (make, you) .. I hope
you (meet, already) .. a lot of interesting people.
Libraries today are different from those in the 1800s. for example, the
contents of libraries (change) greatly through the years.
In the 1800s,libraries (be) simply collections of books.
However, today most libraries (become) multimedia
centers that contain tapes, computers, disks, films, magazines, music,
and paintings. The role of the library in society (change, also)
in the 1800s, libraries (be) .. open only to
certain people, such as scholars or the wealthy. Today libraries serve
every one.
A) Do you do several traveling?
B) yes. I like to
travel.
A) what countries(visit, you) ?
B) well, I (be)
. in India, Turkey, Afghanistan, and Nepal, among others.
I (be, never) .. in any of those countries. When (be,
you) . in India?
b) two years ago. I (visit, also) many of countries in
Central America. I (take) . A tour of Central
America about six years ago.
I have this book (for/ since)..one year.
The telephone (ring) four times in the last hour, and
each time it has been for my roommate.
The telephone (ring) for almost a minute. Why dose
not some one answering it?
We (have) three major snowstorms so far this winter.
I wonder how many more we will have.
Whats the matter? Your eyes red and puffy. (Cry, you) ....?
He (be) . A newspaper reporter before he (become)
.. a businessman.
I was late. The teacher (give, already) a quiz when I
(get) . To class.
I almost missed the plane. All of the other passengers (board,
already) .. by the time I (get) . There.
Yesterday at a restaurant, I (see) .. Abdullah AlMotawakl, an old friend of mine. I (see, not) him in
years. At first, I (recognize, not) . Him because he
(lose) . at least ten pounds.
It was midnight. I (study) . For five straight hours.
No wonder I was getting tired.
Wake up! You (sleep) . Long enough. It is time to get up.
The cost of this book is to pray for me you have the permission to print it or do whatever you want

Get to the heart of English language grammar in easy way

81 of 195

The damage we do to our environment today (affect) .


The quality of life of future generations.
A) Mr. Nabeel Amer (be, not) here next term. He
has resigned. Who (be) . The new teacher? Do you
know?
b) yes. Khaled Alfath. Mr. Alfath (teach) the same
courses Mr. Amer taught: English, and Geometry. I (be) ...
in his algebra class. Do you know which algebra class you (be)
in next term?
She (have) a new car.
She (not/have) .a new car.
When you come, I (sleep)
The planeat 3:03.(arrive)
I will phone you when heback.(come)
When I get home my dogat the gate waiting for me(sit)
I ..him this evening.(visit)
Hehis work this day.(do)
Unless we . Now we can not be on time.
She .unconscious since for oclock.
She jumped off the bus while it.(move)he thanked me for
what I .(do)
It started to rain while wetennis. (play)
A) this light (work, not) the bulb is probably burned
out. Where are the new light bulb?
b) I (get) . One for you.
I dont understand how those marathon runners do it! The race
began over an hour ago. By the time they reach the finish line,
they (run) .. steadily for more than two hours. I dont
think that I can run more than two minutes.
What? He got married again? At this rate, he (have)
. A dozen wives by the time he (die).
Since the beginning of the semester, Anwar (read) .. three
novels. Right now he (read) a farewell to Arms. He
(read) .. that novel for the past three days. He (intend)
. To finish it next week. In this lifetime, he (read)
... many novels, but this is the first Hemingway novel
he (read, ever) .
I have a long trip ahead of me tomorrow, so I think Id better go
to bed. But let me say goodbye now because I wont see you in the
morning. I (leave, already) by the time you ( get)
.. up.
Right now we have a heat wave. The temperature (be) ..
in the upper 90s for the last six days.
The weather has been terrible lately. It (rain) .. off and
on for two days, and the temperature (drop) ... at least
The cost of this book is to pray for me you have the permission to print it or do whatever you want

Get to the heart of English language grammar in easy way

82 of 195

twenty degrees. It (be) in the low 40s right now. Just


three days ago, the sun (shine) . And the weather was
pleasant. The weather certainly (change) . Quickly
here. I never know what to expect. Who knows? When I (wake)
. Up tomorrow morning, maybe it (snow)

a) Tom. Who turn up the air conditioner? It is really cold in here.


My nose is cold and my fingers are cold.
b) I (make) .. you a hot cup of tea.
thanks. That sounds good.
I think it (will/ is going) rain.
Dont worry. I (am going to/will) help you.
We (will/ are going to) travel next week.
Ahmad and Ali (is/are) at home.
He (can/ cans) saw this wood.
He (dont must/ must not) do bad things.
Each one (is/are) studying hard.
Every two (study/studies) hard.
Ali and I (am/ are) drinking.
You but not me (is/are) drinking.
Neither you nor me (is/are) drinking.
Either you or him (is/are) drinking.
One of my friends (is/are) drinking.
He does not (has/have) a book.
What did he (did/ do/ does) yesterday?
He had (had/have/ has) a car.
She always (come/came/ comes) early.
Does he(reads/read/reading) the newspaper?
I (think/ am thinking) he(is good/ is a good) man.
I (think/ am thinking) about you.
The chef (is tasting/ tastes) the sauce.
The food (tastes/is tasting) good.
The piano is heavy. It (weighs/ is weighing) a lot.
The grocer (is weighing/ weighs) the panama now.
Look to those clouds. It (is raining/ is going to rain).
When I ( had gone/ went) to bed, I slept soundly.
1 AFTER YOU HAVE FINISHED CHOOSING THE CORRECT
TENSE PUT THEM IN NEGATIVE AND INTEROGATIVE.

PASSIVE TENSE

The cost of this book is to pray for me you have the permission to print it or do whatever you want

Get to the heart of English language grammar in easy way

83 of 195

HOW TO CHANGE A SENTENCE FROM


ACTIVE TO PASSIVE

1)

The Passive: Passive sentences always describe what happens to


people or things, as a result of action by other people
or things. On other hand, they indicate that what
happens is more important than who is responsible.
2)
To change the sentences from Active to Passive you should know the
following at first;
No
The subject
Its object
Its verb to be
1I
Me
Is, was
2He
Him
Is, was
3She
Her
Is ,was
4They
Them
Are, were
5You
You
Are, were
6We
Us
Are, were
7It
It
Is, was
o
To change a sentence from active into passive. the active must
have (S+V+O)
o
We must add verb to be in the passive;
0
(Verb to be) is one of the following:
o
{Be, Is, Am, Are, Was, Were, Being, Been}
o
Now to change it
1) Omit the subject.
Example:
S
V
O

He helped me .
. helped me .
2) Transfer the object to the place of subject with changing
its shape from (O to S)
Example:

Subject +verb +

object .

The cost of this book is to pray for me you have the permission to print it or do whatever you want

Get to the heart of English language grammar in easy way

..

me

me .
3) Add the verb to be to the verb

helped

84 of 195

was

helped

We are going to start step by step with every tense using verb to be with
every tense;
Tense
Present
simple
Past simple
Present
continuous
Past
continuous
Present
perfect

Active
Take(s)
Took
Is /am/are
Taking
Was/were
taking
Have/has
taken
Had
taken
Will take

Past perfect
Future

Modal+
take

Modals
0

In Passive Come with


Is/am are+ PP(taken)
Was/were+ PP(taken)
(Is/am/are)+being+ PP(taken)
(was/were)+being+ PP(taken)
(have/has)+been+ PP(taken)
(had) +been+ PP(taken)
(Will/shall/be going to)+be+
PP(taken)
Modal+ be+ PP(taken)

Present simple;
{Active} Subject + verb + object.
(1) {Passive} object + is, am, are + P.P.

: Examples;
1-

{Active} He helps me.


{Passive} I am helped.

{Active} I help him.


1{Passive} He is helped.
30

{Active} She helps us.


{Passive} We are helped.

The cost of this book is to pray for me you have the permission to print it or do whatever you want

Get to the heart of English language grammar in easy way

85 of 195

(Note) It is not essential to repeat the auxiliary verb (to be) in a list of
processes; as,
1) The cars are washed, cleaned, checked and driven to the ports.
0

0
1
0
0

Present continuous;
a. {Active} Subject + is , am, are + V + ing + O
{Passive} O + is, am, are +being + P.P.
: Examples;
{Active} I am helping my father.
{Passive} My father is being helped.

{Active} I am helping them.


{Passive} They are being helped.

{Active} He is helping me.


{Passive} I am being helped.

Past simple;
{Active} Subject + past verb + object.
{Passive} Object +was, were + P.P.

: Examples;

{Active} He helped me.


{Passive} I was helped.

{Active} I helped them.


{Passive} They were helped.

Past continuous;
{Active} Subject +was, were + V + ing + O
{Passive} Object + was, were + being +P.P

: Examples;

{Active} He was helping me.


0
{Passive} I was being helped.
1
Present perfect;

The cost of this book is to pray for me you have the permission to print it or do whatever you want

Get to the heart of English language grammar in easy way

86 of 195

o {Active} Subject +Have/ Has +P.P. +O.


{Passive} Subject +Have/Has +Been +P.P.
2

Examples;

{Active} He has helped us.


{Passive} We have been helped.

{Active} they have helped her.


{Passive} She has been helped.

Past perfect;
a. {Active} Subject +Had +P.P. + O.
{Passive} Object +Had +Been + P.P.
Examples;
{Active} He had helped him.
{Passive} He had been helped.

a.

The following modals are


always followed by {Be} when change the sentence from
Active to Passive.
Will, Would, Shall, Should,
Can,
+ be +PP
Could, May, Might, Must
: Examples;
{Active} I will help her.
{Passive} She will be helped.

{Active} He can help us.


{Passive} We can be helped.
{Active} She may help them.
{Passive} They may be helped.
{Active} I must help him.
0
{Passive} He must be helped.

The cost of this book is to pray for me you have the permission to print it or do whatever you want

Get to the heart of English language grammar in easy way

87 of 195

{Active}They will surprise him greatly if you tell him.


1
{Passive} He will be greatly surprised if
he is told.
{Active}One should keep ones promises.
2 {Passive} Promises should be kept.
0

Change the following into Passive:


0 We elected him captain.
1 I have already seen him.
2 He sends us a letter every other day.
3 She was watching T.V.
4 They had fought after they shouted at each other.
5 The legend told us how they saved the castle.
6 Let him do it.
7 Dont argue with me.
8 Can you lift this for me, please?
9 Have you found him?
10 You will lose him if you are late.
11 Why should you suspect me?
12 He sent for you.
13 I always avoid reading such news.
14 I am looking forward to marrying her.

Passive with direct and indirect object:


1) He gave the ball to me. Direct

indirect

I was given the ball.


The ball was given to me.

2) He sent me a gift.

I was given a gift.

A gift was given to me.

To have something done: here the object goes before the


participle; as,
1) I am having my house painted.
1 Ive had the car fixed.
2 I will have your coat cleaned for you.
3 He has his car cleaned.
4 She is going to get her house repaired by the council.

The cost of this book is to pray for me you have the permission to print it or do whatever you want

Get to the heart of English language grammar in easy way

88 of 195

5 I had my bag stolen.


0
Rewri
te the following sentences using (have + object +past participle )
changing some words if necessary:0 Someone delivers the newspaper. (We)
We have the newspaper delivered.
1 Someone cleaned the carpets every year. (I )
2 Their house needed painting.(they are going)
3 We ask someone to check the accounts every month.(we)
4 My camera is being repaired at the moment.(Im)
5 I think it is time to service the car.(it is time to )
6 The computer is no good: we are changing it.(we)
7 I dont like the office curtains. Its time to change them.(I
think I will)
8 We could not to the Jacks flat: it was being painted.(we
could not go Jacks flat: he)
9 Theres something wrong with the typewriter. It needs
repairing.(I think I will)
1
0

Passive with more than one main verb:


1) I wanted her to give me a pen.
She was wanted to give me a pen.

2) I can not wait seeing her.


Seeing her can not be waited.

0
1
2

Passive with command:


1) Write the lesson.
Add (let) +object + (be) +PP.
Let the lesson be written.
2) Do your homework.
Let your homework be done.
3) Dont play football.
Let not football be played.

Passive with questions:


At first Change the question into sentence;
Does he write the lesson?
The cost of this book is to pray for me you have the permission to print it or do whatever you want

Get to the heart of English language grammar in easy way

89 of 195

He writes the lesson.


2) Then change it into Passive with yes/no question.
The lesson is written.
Is the lesson written?
Did he write the lesson?
The lesson was written.
Was the lesson written?
Who did this?
By whom was this done?

Adverb with Passive (change the position of the adverb):

I fought them bravely.


They were bravely fought.

Verbs used as adjective in the shape of Passive:


He is married.
He got married to Jane.
I am finished with my work.
I am done with my work.
I am lost.
I am interested in Arab civilization.
I am satisfied with my job.
The passive with GET:
I stopped working because I got tired.
They are getting married next month.
I get worried because he was two hours late.
Present participle convey an active meaning, while Past participle convey a
Passive meaning; as,
1) The problem confuses the students.
The students are confused.
They are confusing problems.
2) The story amuses the Children.
The children are amused.
It is amusing story.
Stative Passive: here the passive form may be used to describe an existing
situation or state; as,
The door is opened. (no action is taking place)
The window is broken. (no action is taking place)
I am interested in Yemen civilization. (Often Stative verb followed by
preposition.)
The cost of this book is to pray for me you have the permission to print it or do whatever you want

Get to the heart of English language grammar in easy way

90 of 195

I am lost. I am married. I am don with my work. (Such idiomatic usage of


passive has no equivalent active sentences)
Complete the following with sentences with the present or the past
participle of the verbs in parentheses:
10 The (steal) jewelry was recovered.
11 Success in ones work is a (satisfy) . Experience.
12 The dragon was a (terrify) sight for the villagers.
13 The (terrify) ...villagers ran for their lives.
14 I found myself in an (embarrass) ...situation last
night.
Do the following exercise:
I read him a story.
I asked them a question.
I gave her an examination.
Dont eat the orange.
Help your brother.
They grew rice in India.
She called him stupid.
We have changed our mind.
You will be reading the story.
Does he hit him?
Why did she break the window?
Is he drinking the milk?
Shakespeare wrote that play.
I will invite you to the party.
Ann is preparing that report.
The teacher is going to explain the lesson.
Osma Nabeel has suggested a new idea.
Amar developed that theory.
The solution to my mathematics problem appeared to me in the dream.
Does your father teach you how to use this machine?
My sweater is made in France.
The world cup soccer games are being televised all over the world.
An accident happened at the corner of fifth and Alzobairy.
When is someone going to announce the result of the contest?
If you expose a film to light while you are developing it, you will
ruin the negative.
My fathers plain will arrive at 10:00.
The government requires each international student to have a
visa.

The cost of this book is to pray for me you have the permission to print it or do whatever you want

Get to the heart of English language grammar in easy way

91 of 195

ADJECTIV
E

IS A WORD USED TO ADD SOMETHING TO THE


0
MEANING OF THE NOUN
0
1
2
3 THE STRUCTURE:
O Subject +be +adjective.
Example:
She is beautiful.
0
Subject +be +(a)+ adjective +noun.
Example:
She is a beautiful girl.
They are beautiful girls.
1

THE ADJECTIVE DEPENDS ON the modifier itself. If it is Singular,


it is singular. If it is plural it is plural. If its MALE it is male. If it is
FEMALE, it is female in its meaning. So we mustnt change it. In other
words, its singular, plural, male, and female.
0
Example:
0 She is clever.
1 He is clever
2 They are clever. Not (clevers)

We can talk about a whole class by using (the + adjective)


Example: the poor always suffer.
Kinds of adjective:
1) Adjective of quantity:
they answer the question (how much?)
Examples:
I ate some rice.
Children need much patience.
He has little intelligence.
I have had enough exercise.
He has lost all his wealth.
You have no sense.
I did not eat any rice. Etc.
2)Adjective of number: They answer the question (how many)
The cost of this book is to pray for me you have the permission to print it or do whatever you want

Get to the heart of English language grammar in easy way

92 of 195

i. Example:
ii. The hand has five fingers.
iii. Few cats like cold water.
iv. There are no pictures in this book.
v. I have taught you many things.
vi. All men must die.
vii. Here are some oranges.
viii. Most boys like cricket.
ix. There are several mistakes in your exercise.
3) Demonstrative adjective: they answer the question (which)
x. Example:
xi. This boy stronger than me.
xii. That boy is industrious.
xiii. These mangoes are sour.
xiv. Those rascals must be punished.
xv. I hate such thing.
xvi. Dont be such a hurry.
xvii. Yonder fort once belongs to Shivaji.
xviii.
4) Adjective can be used as if they were Noun after (a / an) and sometimes can have a
plural.
The listener mentally supplies the missing noun:
I have my medical on Monday (= medical examination)
Do not be such a silly! (= A silly fool)
The meter with the electrics in my car. (The electrical system)
5) The + Adj: adjectives after THE can be used to represent a group as a whole
The rich are not always happy
Andrew was sent to a special school for the deaf
6) Nouns that behave like adjectives, names of materials, substances etc
Its a cotton dress a summer dress.
(Note) Here are some materials have adjectival form such as; gold, golden, lead, leaden,
silk, silken, silky, stone, and stony. but the adjectival form generally has a metaphorical
meaning like:
- I have a gold watch. (= A watch made of gold)
I have a golden watch (= A watch is like gold in color),
7) present and past participle used as adjectives:
Breaking glass, frightening story
Broken heart, a frozen lake, locked door, an aged parent, a crooked-path, a naked
man
Amazed / ing / annoyed / ing excited / ing impressed / ing.
8) Adjective used in measurements:
(Deep-long-wideetc)
How deep is that pool? It is five meter deep
How old are you? I am five years old.
Jim is six foot tall = Jim is six feet tall = Jim is a six-foot man
The cost of this book is to pray for me you have the permission to print it or do whatever you want

Get to the heart of English language grammar in easy way

93 of 195

9) Adverb that can function as attributive Adj, especially in fixed phrase.e.g.


The above statement, the undersign, the inside cover- etc.
10) Size generally precedes age and shape:
A large table, a large old table, a large old round table.

(NOTE) the use of (this/ that/ these and those):


This and its plural these refer to what is close at hand, and nearest to the
thought of the speaker; as,
This is better than that.
That and those refer to what is farther away and more remote; as,
I dont like those boys.
That, with its plural those, is used to avoid the repetition of a preceding
Noun; as,
The climate of Taiz is like that of Aden.
The streets of this city are worse than those of Hadah.
When we are talking about two things, first take this and the second take
that; as,
Alcohol and tobacco are both injurious; this perhaps, less
than that.
**(Note) the word {this, that, these, those} can be used as a pronoun and an
adjective; as,
This is my book. Those are your books.
b. Formation of the possessive case:
1-With singular add (s); as,
The boy has a car. The boys car.
1 With plural add (s); as,
The boys have a car. The boys car.
2 With irregular singular add (s); as,
The man has a car. The mans car.
3 With irregular plural noun add (s); as,
The men have a car. The mens car.
4 With a singular noun ends in (consonant+ y) add (s)
The company has two branches.
The companys two branches.
5 With a plural noun ends in (consonant+ y) omit
The <y> and add (ies)
The cost of this book is to pray for me you have the permission to print it or do whatever you want

Get to the heart of English language grammar in easy way

94 of 195

The companies have two branches.


The companies two branches.
6 If the noun ends in <s> we add only <>; as,
Anees has a car. It is Anees car.
7 If something belongs to two persons, the possessive is added
to the latter; as,
Anwar and Aiman have office
Anwar and Aimans office. (= One office)
8 If some things belong to two persons separately, the possessive
is added to each one; as,
Anwar has an office and Aiman has an office
Anwars and Aimans offices.
(= Two offices or more)
9

If both the speaker and the listener know the place, We add
only (s) and omit the name after it.
Met him in my uncles. (= Uncles house)
I bought it from the bakers.

Other possessives:
1- use (OF)
The end of the road.
The smell of cooking.
The leader of the party.
2- for common nouns like(house, car, school, table, etc) of is not
necessary, and the word position changes:
car keys
The kitchen door.
A bus driver.
3- always use of with the words front, top, bottom, back, end.
The bottom of the garden. ( not the garden bottom)
The front of the house.( Not the house front)
4) For expressions of time s or s is used:
A fortnights holiday.
Two weeks rest.
Use the correct possessive to complete the sentences. If two answers are
possible, write the more likely one:
0 I opened the..(door/car)
1 My keys are in the .(pocket/my suit)
The cost of this book is to pray for me you have the permission to print it or do whatever you want

Get to the heart of English language grammar in easy way

95 of 195

2 I opened the ..(door/kitchen)


3 She put her suitcase in the .(car/boot)
4 My room is at the ...(front/hotel)
5 I am afraid I have broken the (leg/chair)
6 Would you turn on the ?( light/ kitchen)
7 We sat on the . ( carpet/ sitting-room)
8 Someone has damaged the (front/my car)
9 This is Mr. Smith, the .(manager/cinema)
10 The cat walked along the (wall/garden)
11 The doctor told me to have a (week/rest)
12 You will feel better after a
(fortnight/holiday)
13 After a few . The plane took off.(hours/delay)
14 Would someone open the ?
(window/bathroom)
1 Position of an adjective:
A single adjective is used attributively and generally used immediately before
the noun (and is therefore it is noun phrase); as,
0 A clever boy is always successful in his study.
1 I met a young shop assistant.
We say that an adjective used Predictively when it comes directly after ( be,
seem, look, appear, etc) and it can be used on its own as a
complement; as,
0 He is clever.
1 The young shop assistant met me.
2 He seems angry today.
(NOTE) there are some adjectives used only predictively; as,
0 Those describing health; as,
He is ill.
He is sick.
He feels faint.
I am well.
1 Those start with (a)
The vessel is afloat
The children are afraid.
The buildings are alight
Everything that is alive.
The children are asleep.
2 Those describe feeling:
The cost of this book is to pray for me you have the permission to print it or do whatever you want

Get to the heart of English language grammar in easy way

96 of 195

I am sorry for him.


I am glad to meet you.
You look upset.
I am content with it.
In poetry, however the adjective frequently placed after the noun; as,
1) Children dear, was it yesterday.
2 ADJECTIVES USED AS A NOUN:
0 As Plural Nouns denoting a class of person; as,
The rich know no how the poor live.
1 Those driven from proper noun; as,
Australians, Americans, Canadians, Italians.
2 Some denoting persons; as,

Juniors, seniors, mortals, inferiors, superiors, nobles,


criminals, elders, minors.
3 Some adjectives used as noun only in the plural; as,
Sweets, bitters, valuables, eatables.
4 In certain phrases; as,
In future I will charge you for medical advice.
In short we know nothing.
The negotiations were carried on in secret.
I will see you before long.
Before long, he will be appointed to a higher post.
At present, he is in pecuniary difficulties.
I dont want any more in present.
He has left India for good.
At best we shall get no more dividend than five cents in a dollar

At best he is a clever versifier; but a poet he is certainly not.


It must be said to his credit that he stood by his friend
through thick and thin.
I must have your terms down in black and white.
Right or wrong, my country.
I am afraid the young man is from bad to worse.
The long and short of it, is that I distrust you.

0
maybe formed from:-

Compound adjective

The cost of this book is to pray for me you have the permission to print it or do whatever you want

Get to the heart of English language grammar in easy way

1) NOUN+ADJECTIVE (or participle) :Blood red


Sky blue
Snow white
Skin deep
Purse proud
Life long
Home sick
Stone blind
Stone blind
Heartrending Ear piercing
Timeserving
Bedridden
Handmade
Sea girl
2) ADJECTIVE+ADJECTIVE
Red-hot
Blue black
White-hot
3) ADVERB+PARTICIPLE:Longsuffering
Everlasting
Outspoken
Downhearted

Pitch dark
World wide
Seasick
Moth-eaten
Lovelier

Dull gray

Thoroughbred
Far seen

97 of 195

Breast high
Head strong
Noteworthy
Heartbroken

Lukewarm

well dressed
Inborn

5
Write the opposite of the following and put them in
sentences:
{Happy, sad, industrious, lazy, big, small, soft, harsh, hard, polite,
rude, wise, foolish, rich, poor, young, new, old, long, short, quick,
slow, strong, weak, handsome, ugly, clever, dull, kind, cruel,
healthy, dutiful, distant, certain}
6 Use the following adjective in sentences:
{Happy, sad, industrious, lazy, big, small, soft, harsh, hard,
polite, rude, wise, foolish, rich, poor, young, new, old, long,
short, quick, slow, strong, weak, handsome, ugly, clever, dull,
kind, cruel, healthy, dutiful, distant, certain}
7

Use suitable adjective with the following:


{Storm, siege, sleep, victory, advice, blow, silence, hands, water,
servant, flower, city, artist, dealer, voice, husband, subject,
child, king, dog}

2.25.1.15. 24) Adjectives follow (get)


{GET +
Angry
Busy
Empty
Hungry
Old
Tall

Anxious
Chilly
Fat
Late
Rich
Thirsty

Bald
Cold
Full
Light
Sick
Well

Better
Dark
Good
Hot
Sleepy
Wet

Big
Dizzy
Heavy
Mad
Nervous
Worse

I am getting hungry.
The cost of this book is to pray for me you have the permission to print it or do whatever you want

Get to the heart of English language grammar in easy way

98 of 195

You should not eat so much. You will get fat.


2.25.1.16.
Complete the sentences by using an appropriate form of get and the
given verbs: (tired) I think I will stop working. I
(hurt) there was an accident, but no body .
(lose) we did not have a nap, so we
(dress) we can leave as soon as you
(marry) when . You ?
(accustom) how long did it take you to . . Living
here?
(bare) I , so I did not stay for the end of the movie.
(pay) I on Fridays. I will give you the money I owe
you next Friday. Okay?
(disgust) I . And left because the things they were
saying at the meeting were ridiculous.
(engage) First, they
(marry) then, they
(divorce) Later, they
.

ADJECTIVE ORDER

The cost of this book is to pray for me you have the permission to print it or do whatever you want

Get to the heart of English language grammar in easy way

99 of 195

0
OPINION
: New, old, good, bad, beautiful, ugly, nice, wonderful,
Etc.
1
SIZE
: Big, Large. Small; Wide; Narrow; Thick; fat, heavy;
Etc.
2
AGE:
Little. Young Old, A Two- Year
Old..ETC.
3
SHAPE: Round, Circular, Curved, Straight, Triangular, Rectangular.
Etc.
4
Colour
: White, Red, Yellow, Blue, Gray, Green, Black, Pink,.
Etc.
NATIONALITY: Yemeni, American, British, Iraqi, Japanese, ..Etc.
5
MATERIAL
Wool, Cotton, Silk, Iron, Wood, Plastic, Silver,
Gold, .Etc.
2.25.1.17. Reorder the following adjective:0
We bout some glasses German old lovely.
1
Have you seen his sports Italian car new nice?
2
He lived in a wooden old hat dirty.
3
She was wearing a beautiful shirt silk black and white.
4
We stood under statue an wooden old enormous.
5
They gave him clock a silver beautiful.
6
I bought a table cloth large striped red and white.
7
It was old lovely scarf a woolen green.
8
The sent him plate blue and white China expensive very some.
9

10
11
12

2.25.1.18. Reorder the following sentences;


Plastic , black , big , bag .
English, tall ,

nice ,

girl .

Yemeni , young , famous , doctor.


Red , old , large , car .

13

Lovely , big , brown , eyes .

14

Bag , paper , blue , small , strong .

The cost of this book is to pray for me you have the permission to print it or do whatever you want

Get to the heart of English language grammar in easy way

15
16
17
18

100 of 195

Long , pretty , dark , hair .


Cotton , large , white , coat .
Silk , black , large , jacket .
American, leather, new, brown, shoes.
----------------------------------------------------------------------------0{NOTE} Sometimes the order of adjective depends on the translation
itself so we can say either
American academic institute
or Academic American institute
1{NOTE} if the material adjective has more than one adjective in one
sentence, we should start with origin and its parts; as,
Natural Mineral Water. =(Water consists of many mineral,
And these mineral are taken from nature)

The cost of this book is to pray for me you have the permission to print it or do whatever you want

Get to the heart of English language grammar in easy way

101 of 195

COMPARISON OF ADJECTIVE
2
There are three kinds of comparison;
(He is tall.)
1) Positive:
a) As + adjective + as
He is as tall as his brother.
b) Nearly as + adjective+ as
He is nearly tall as his brother.
c) Twice/three times +as adjective+ as
This chair is twice times as cheap as that one.
2) Comparative:
a) adjective +er +than
He is taller than his brother.
b) More + adjective +than
He is more courageous than his brother.
3) Superlative:
a) The + adjective +est
He is the tallest in the family.
b) the + most + adjective..
He is the most courageous between us.

0
1
2
3
4
5

c) by far/easily+ the most +adjective


It is by far the most expensive car in the word.
It is easily the most expensive of all the cars I
have seen.
0 The negative of the former;
He is not as tall as his brother.
He is not so tall as his brother
He is not taller than his brother.
He is not the tallest of the class.
He is not more courageous than me.
He is not the courageous between us.

The cost of this book is to pray for me you have the permission to print it or do whatever you want

Get to the heart of English language grammar in easy way

4
0
1
2

How to add (er):


No
Adjective
1)
Happy
2)
Brave
3)
Red

Consonant + Y
(Y)= (ier)
(e)=(er)
(v+c)= (dder)

102 of 195

Comparative
Happier
Braver
Redder

Superlatives are used to compare one thing with several others:


1) They are used with (the ..in), or (the ..of) sometimes they
are used with only (the)
This is the longest river in the world.
This is the most expensive car of them all.
This is the most expensive car here.
0
Write
the superlatives of the words given, using (in) or (of) where necessary:
0
This is
..the world. (big building)
1
This is
. Here.
2
He
bought . The shop. (comfortable chair)
3
I think
she is ..the group. (good singer)
4
He is
the company.(careful driver)
5
Who is
the class? (old student)
6
It is
. I have ever seen. (bad film)
7
She is
.. all the assistants. (helpful)
8
He is
... his class.(young)
9
She
is I have ever met.(strange person)
0
Compl
ete these sentences:
0
The
weather is much it usually is at the same time.
(pleasant)
1
Going
by car took twice going by train. (long)

The cost of this book is to pray for me you have the permission to print it or do whatever you want

Get to the heart of English language grammar in easy way

103 of 195

2
It was
by far time of my life. (bad)
3
The
trains a lot all the other ways off getting there. (fast)
4
This
exam was a bit ..all the other test.(good)
5
Flying
s a lot . Going by car.(quick)
6
The
food is not nearly. It has been in the past.(good)
7
She is
a bit .. her brother.(sensitive)
8
First
class is much second.( expensive)
9
The
film was not.. I had thought it would be. (good)
5NOTE {more /most} come with adjective composed of two syllable
more like (beautiful) = (beau ti ful)
6
NOTE (there are irregular adjective):
No
Positive
Comparative
1)
Good/well
Better
2)
Bad/evil/ill
Worse
3)
Many
More
4)
Much
More
5)
Little
Less/er
6)
Few
Less
7)
Far
Farther
8)
Far
Further
9)
Late
Later/latter
10)
Old
Older/elder
11)
Nigh
Higher
12)
Fore
Former
13)
In
Inner
14)
Up
Upper
15)
Out
Outer

or

Superlative
Best
Worst
Most
Most
Least
Least
Farthest
Furthest
Latest/last
Oldest/eldest
Next
Foremost/first
Inmost/innermost
Uppermost
Utmost

{Note} To emphasize the comparative or superlative, you can add the


following before them (far, much, great, less, no, lots, a lot); as,
The cost of this book is to pray for me you have the permission to print it or do whatever you want

Get to the heart of English language grammar in easy way

104 of 195

He is better than his brother.


He is a lot better than his brother.
He is much better than his brother.
He is less clever than his brother.
NOTE: (later &latest) refer to time.
Example:
1) He is later than expected.
1 I have not heard the latest news.
NOTE: (latter &last) refer to position.
Example:
3) The latter chapters are lacking in interest.
4) The last chapter is carelessly written
5) Ours is the last house in the street.
NOTE: (older/oldest) used of person and thing:
Example:
6) He is older than his sister.
7) It is the oldest temple in the town.
NOTE: (ELDER &ELDEST) used with persons of the family only
(elder) does not used with than.
Example:
8) Osama is my elder son.
9) Manal is my eldest daughter.
NOTE: (farther &further) express distance:
Example:
10) Sana'a is farther/further than Taiz to Aden.
(Further) but not (farther) is used to mean addition.
19 After this he made no further mistakes.
NOTE: (nearest & next) nearest means the shortest distance.
Example:
12) Hodaidah is the seaport nearest to Khokha region.
(Next) refers to one of the sequence thing.

13) He lives in the next house.


{Note} certain English comparative have lost their comparative meaning
and are used as positive and cannot be followed by than; as,
Former
Hinder

Posterior
Ulterior

Neither
Elder

Inner
Outer

The cost of this book is to pray for me you have the permission to print it or do whatever you want

Get to the heart of English language grammar in easy way

Exterior
Utter
Major
Prior

Interior
Anterior
Minor

Upper
Latter
Inferior

105 of 195

Senior
Junior
Superior

Examples:
1> the inner meaning of this letter is not clear.
2> the soldiers ran to defend the outer wall.
3> the elder brother is a doctor.
4> I have no ulterior motive in offering you help.
5> Ali is inferior to Ahmed in intelligence.
6> Ahmeds intelligence is superior to Alis.

Do the following exercise:


The majority accepted the..proposal.(later/latter)
The .. part of the book shows signs of hurry.(later/latter)
At a .date, he was placed in charge of the whole taluka.
(later/latter)
I prefer the ..proposition to the former.(later/latter)
Is there no news than last weeks?
I have an.sister.(older/elder)
Ibraheem is .than Nabeel.(older/elder)
His brother is a doctor.(older/elder)
She is the ..than the two sisters.(older/elder)
I can not walk any.(farther/further)
No .reasons were given.(farther/further)
He walked off without .. ceremony. (farther/further)
The..news from China is very disquieting. (last/latest)
The ..time I saw him, he was in high spirit. (last/latest)
Today is the.to receiving the tenders.(last/latest)
We expect to get the .news in a few hours. (last/latest)
This hotel is .. hotel in the country.(big)
Nabeel is as as Osama.(happy)
Manal is . Maram.(old)
Manal is the student.(clever)
How is your brother today? Is he ..? (good)
The cost of this book is to pray for me you have the permission to print it or do whatever you want

Get to the heart of English language grammar in easy way

106 of 195

Her doll is than yours.(pretty)


Your knife is sharp, but mine is .
Prevention is .. than cure.
Who is the boy in the class?
This is the of my two sons.
This is the .I can do.
Shakespeare is the.than any other English poet.
Change the degree of Comparison, without changing the meaning:Malacca is the oldest town in Malaysia.
Soya beans are at least as nutritious as meat.
No other planets is so big as Jupiter.
Very few boys are as industrious as Latif.
He would sooner die than tell a lie.
India is the largest democracy in the world.
Shakespeare is the greater than any other English poet.
Lead is heavier than any other metal.
The tiger is the most ferocious of all animals.
Australia is the largest island in the world.
Some people have more money than brains.
A wise enemy is better than a foolish friend.
The Marwaries are not less enterprising than any other community in India.
I know him quite as well as you do.
No other man as strong as I am.
Mount Everest is the highest peak of the Himalayas.
Very few animals are as useful as the cow.
America is the richest country in the world.
The pen is mightier than the sword.
Open rebuke is better than secret love.

The cost of this book is to pray for me you have the permission to print it or do whatever you want

Get to the heart of English language grammar in easy way

107 of 195

ADVERB
Adverb: The word adverb (ad-verb) suggests the idea of adding to the meaning of
a verb. This is what many adverbs do. They can tell something about the action.
Example:
Hashim played football well. (How did he play?)
However, adverbs can also modifier:
Adjective
: he is very good.
.Other adverb
: I will do it very soon.
.Prepositional phrase: you are entirely in the wrong.
Complete sentences : strangely enough, I won the prize.
Nouns
: the man over there is a doctor.
(Note) adverb sometimes effects the meaning of a sentences:
a) He has left.
He has just left
b) He has finished work. He has already finished work.
Kinds of adverb:
Adverbs of time which shows when; as,
(before, now, lately, daily, already, formerly, since, ago, yesterday,
tomorrow, etc)
Have not I seen you before?
John came on Sunday, and I arrived the day after.
I had a headache this morning, but I am all right now.
I have not been feeling very well lately.
She is paid on a daily basis.
The cost of this book is to pray for me you have the permission to print it or do whatever you want

Get to the heart of English language grammar in easy way

108 of 195

It is too late to give him any advice, he is already made up his mind
Formerly he worked in a factory, but now he is a teacher.
I saw him Sunday, but I have not spoken to him since.
Adverbs of frequency which show how often; as,
(always,
rarely, usually, often, seldom, twice, again, frequently, etc)
I have been to Marb twice.
The sun never shines at night.
I rarely work at weekend.
I hardly ever do it.
I occasionally work late on Friday
I frequently eat out at the restaurant.
I nearly always come to work by train.
Adverbs of manner which answer how?; as,
(clearly, well, soundly, sadly, so, only, hard, etc)
He speaks clearly
He is sleeping soundly
He walked sadly away
I ate so much food that I was almost sick
I saw him only yesterday.
I tried so hard to please her
{Note} there are adjective end in (ly) like adverb such as, (friendly,
cowardly, deadly, lively) to use them as adverb put them in
the middle of (in.way); as,
He gave me a friendly smile.
He is smiling at me in friendly way.
He did it in a good way.
Adverbs of place which answer where?; as,
(up, down, within, backward, in, out, etc)
My pen fell on the floor, and I pick it up
She was jumping up and down
He will arrive within an hour
Can you say the alphabet backwards?
Open the box and put the money in it
Open the box and take the money in out
Adverbs of degree or quantity, which answer how much/many?; as, (too,
almost, any, quite, rather, partly, as..as, altogether, no better,
prettily, very, etc)
Its too hot here
I almost dropped the plate
They nearly always have coffee for breakfast
I cannot stay any longer
The cost of this book is to pray for me you have the permission to print it or do whatever you want

Get to the heart of English language grammar in easy way

109 of 195

Im not quite ready to go


Its rather cold
What you say is partly true?
He is an altogether different matter
Adverb of reason or negative; as,
(surely, certainly, not, etc)
Surely you remember him
He is certainly good
Adverbs of reason; as,
(hence, therefore, etc)
The town was built near a bridge on the river: Hence the name Cambridge
The item you requested is no longer available, therefore we are returning your
check
Adverbial phrase: Adverbial phrase of manner, place and time and others
are often formed with a Preposition + noun: in a hurry at all, very
much in dead as a matter of fact, in that case.
.Adverb particles: Certain words as, in, off, up, function either as
prepositions or as adverb particles when such words followed by an
object the function are preposition when there is no object, they are
adverb; as,
The children are in the house. (Preposition)
The children have just gone in. (adverb)
He climbed up the hill. (Preposition)
I picked my pen up. (adverb)
Good-bye! I am off now.
get off my foot.
Interrogative adverbs; as,
(where, when, what, why, who, which, whose, how)

other usage:
1) The use of (THE) with comparative; if two things of exactly the same kind are being
compared, we can use THE before a comparative in formal style: if we dont mention
the second item;
Which is (the) longer?
The gray coat is (the) longer.
2- Comparative with (er) and (er) this kind can convey the idea of general increase:
Saly is growing fast. She is getting taller and taller.
Comparative came more and more complicated
Holiday flights are getting less and less expensive.
3- The + comparative + the; this kind show that when one charge is made, another
follows;
The more money you make, the more you spend.
The more expensive petrol become, the less people drives.
The cost of this book is to pray for me you have the permission to print it or do whatever you want

Get to the heart of English language grammar in easy way

110 of 195

3- Intensities used with comparative.


A bit (very) much far even hardly a lot, lots, a little, no, rather, somewhat.
It is a little colder to day.
Houses are much more expensive these days
5- degree of similarity are; almost, exactly, just, nearly + as + adj.
6- adverbs used with comparison.
Siham sings as sweetly as her sister.
I can not swim so well as you can.
The rain cleared more quickly than I expected.
The faster I type, the more mistakes I make
It rained more and more heavily
I drive faster than anyone I know.

Spelling and form of adverbs ending in (ly) :


Ad (ly) to the adjective
badly
(Y) Becomes (ily)
happy
happily
Deleted (e) and add (ly)
hobble
nobly
Adjectives ending in (ic) add (ally) fantastic

fantastically

Comparison of adverbs; only adverbs of manner, degree, and time are


admit of comparison:
There are some adverbs compared like adjective by ending (er), and
the superlative by adding (est); as, (fast faster fastest. Hard
harder hardest.)
There are some adverbs ending in (ly) we add more & most except
(early); as, (swift swifter--. Most swifter)
Some very common irregular adverbs in comparative:
POSITIVE
Ill /badly
Well
Much
Little
Near
Far
Late

Comparative
Worse
Better
More
Less
Nearer
Farther/further
Later

Superlative
Worst
Best
Most
Least
Next
Farthest
Last

formation of adverb:
Adjective + ly :
(wisely, cleverly, beautifully, happily, singly)
Proposition + noun:
(be-times, beside, to-day, to-morrow)
The cost of this book is to pray for me you have the permission to print it or do whatever you want

Get to the heart of English language grammar in easy way

111 of 195

a +adjective:
(Abroad, along, aloud, anew, behind, below, beyond)
Proposition +adverb:
(Within, without, before, beneath)
Adverb +proposition:
(Thereby, wherefrom, therein, thereof, thereon, thereto, therewith,
wherefore, wherein, whereon, whereof, hitherto, thenceforth,
thenceforward, henceforth, henceforward)
2.25.1.19. Adverb +and + adverb:
Again and again (= more than once, repeatedly)
By and by (=before long, presently, after a time)
Far and near (in all direction)
Far and away (by a great deal, decidedly, beyond all comparison)
First and foremost (=first of all)
Now and then (=from time to time, occasionally)
Now and again (= at intervals, intermittently)
Once and again (=on more than one occasion, repeatedly)
Out and away (= beyond comparison, by far)
Out and out (= decidedly, beyond all comparison)
Over and above (many times, frequently, repeatedly)
Through and through (=thoroughly, completely)
Thus and thus (=in such and such a way)
Example:
Good book should be read again and again.
I warned him again and again.
By and by the tumult will subside.
His fame has spread far and near.
As a statesman he saw far and wide.
This is far and away the best course.
He is far and away the best bowler in our eleven.
He now and then writes on fiscal questions.
I write to them now and then.
He worked ten years, off and on, on his Pali Dictionary.
I have told you once and again that you must not read such rash.
This is out and away the best work on Astronomy.
He gains over and above this, the goodwill of all people.
Over and above being hard-working he is thoroughly honest.
He reads all the novels of Scott over and over.
I believe Sami is out and out the best Hindu bowler.
He has read Milton through and through.
Thus and thus only we will succeed.
The cost of this book is to pray for me you have the permission to print it or do whatever you want

Get to the heart of English language grammar in easy way

112 of 195

Position of adverbs:
Verb + preposition + adverb; as,
Nabeel is in his room .
Preposition +adverb+ Verb; as,
Nabeel is in his room studying.
Verb+ adverb of manner; as,
It
rains heavily.
Verb + object+ adverb of manner.
He speaks English well.
Verb+ adverb of place ( up/ down/ here/ there); as,
He is sitting over there.
He is up.
Please, sit down.
Verb+ adverb of degree+ adverb of manner; as,
He is very well.
Verb+ adverb of manner+ place; as,
He went quickly to school.
Verb+ adverb+ adjective; as,
It is terribly cold.
Verb+ adverb+ adverbial phrase; as,
He was madly in love with her.
Adverb+ noun+ adverb; as,
Quickly he went to school.
Verb+ adverb+ adverb; as,
He went home quickly.
Time+ noun+ verb+ adverb; as,
Yesterday he went home quickly.
Short time+ longer + longest; as,
I got up at 6:00 early in the morning.
Adverb of Place+ place+ place; as,
He is sitting on the chair under the tree in the park.
Adverb of manner + place + time; as
He spoke loudly at the meeting last night.
The cost of this book is to pray for me you have the permission to print it or do whatever you want

Get to the heart of English language grammar in easy way

113 of 195

Adverb of frequency come after (to be/ have) or immediately after


the pronoun if there is not; as,
I always help others.
He is always helping others.
I have always sent him letters.

Adverb with Passive:

I fought them bravely.


They were bravely fought.

In the following sentences (1) pick out the adverbs and tell what each
modifies; (2) tell whether the modified word is a Verb, an adjective, or
an adverb; (3) classify each Adverb as an adverb of time, place,
manner, degree, etc:He was ill pleased.
Try again.
He is too shy.
We rose very early.
I am so glad to hear it.
Cut it lengthwise.
Too many cooks spoil the broth.
Are you quite sure?
That is well said.
Once or twice we have met alone.
The railway station is far off.
I have heard this before.
Father is somewhat better.
I am much relieved to hear it.
The walk was rather long.
The patient is much worse today.
She arrived a few minutes ago.
Ambition urges me forward.
She was dressed all in black.
We were very kindly received.
Her son is out in Iran.
I surely expect him tomorrow.
He could not speak, he was so angry.

The cost of this book is to pray for me you have the permission to print it or do whatever you want

Get to the heart of English language grammar in easy way

114 of 195

You are far too hasty.


The secret is out.
He is old enough to know better.
I would much rather not go.
You need not roar.
Your watch is five minutes too fast.
He went off on Monday.
His health is no worse than before.
Wisdom is too high for a fool.
There is a screw loose somewhere.
I see things differently now.
Rome was not built in a day.
The door burst open and they came in.
We have scotched the snake, not killed it.
Dont crowd your work so closely together.
The patient is no better today.
Do not walk so far.
Put not your trust in princes.
Order the carriage round.
He has been shamefully treated.
I wonder you never told me.
0 Insert the given adverbs (or adverb phrase) in their normal position:
He invited me to visit him. (often)
I am determined to yield this point.(never)
The letter was read. (Carelessly)
We have seen her.(just, in the square)
I have to reach the office.(by 9:30, usually)
Will he be (there, still).
I will meet you(this evening, in the park).
The train has left (just).
Can you park your car near the shops?-yes, I can(usually).
You have to check your oil before starting 9always).
He is in time for meal (never).
We should come (here, one morning).
He has recovered from his illness (quite).
She goes to the cinema (seldom)
That is not good (enough).
You must say such a thing (never, again).
Osama Amer arrives (always, at 9 oclock, at the office, in the
morning)
He played the violin (last night, brilliantly in the concert).
1
Form sentences to illustrate the use of the following words as adjective
and as adverb:
{Very, near, ill, only, clean, long, early, late, fast and friendly}
The cost of this book is to pray for me you have the permission to print it or do whatever you want

Get to the heart of English language grammar in easy way

115 of 195

2.25.1.20. Write the correct formof adverbs for these sentences:o She works .. than all the others. (hard)
o Of all the machines, this one works the .(good)
o Could not you drive a bit ?(careful)
o I can not understand: would you ask him to speak.?(clear)
o They all behaved badly, but Pat behaved the.(bad)
o John was shouting than everybody else.(loud)
o I think I understand than the others.(good)
o Susan climbed ..than the rest of us.(fast)
o She gets up . Than everybody else in the house.(early)
o Do you think they have acted ..?(stupid)
Complete these sentences with an Adjectives and an adverbs: check
1
It was
. Driving I have ever seen. (good)
2
Peter
sang .. than all the others. (loud)
3
The holiday was not ..as the one we had
last year. (expensive)
4
Shes a good student: she works ..than the
others.(careful)
5
Would you play please? I am trying to
sleep.(quiet)
6
Of all the people in the factory, Nansi works
..(efficient)
7
The weather is not .as I had expected.
(bad)
8
This is the company in the world.(big)
9
She plays the piano..than anyone else in her
class.(beautiful)
10
Mr. Smith is person in the village.(old)
11
Mark hit the ball very .(hard)
12
She runs .than anyone else in the team.
(fast)
13
Do you think older people drive .than young
people.(slow)
14
They all dance well, but John dances
(good)
15
I think they both behaved very .(rude)

The cost of this book is to pray for me you have the permission to print it or do whatever you want

.25.1.21.

Get to the heart of English language grammar in easy way

116 of 195

INFINITIVE
18) Infinitive used like (a noun)
1-18) As the subject of a verb:
To find fault is easy.
To err is human.
To reign is worth ambition.
2-18) As the object of a verb:
I dont mean to read.
I like to play cards.
3-18) As the complement of a verb:
Her greatest pleasure is to sign.
His custom is to ride daily.
4-18) As the object of a preposition:
The speaker is about to begin.
He had no choice but to obey.
5-18) As an object complement:
I saw him go.
The cost of this book is to pray for me you have the permission to print it or do whatever you want

Get to the heart of English language grammar in easy way

117 of 195

2.25.1.22. Other uses of the infinitive:


2.25.1.22.1.1.
To express a purpose: He went to America to study English.
2.25.1.22.1.2.
After only to express a disappointing sequel: - e.g.
She hurried to the house only to see that it was on fire.
He came back only to find that his son had a car accident.
2.25.1.22.1.3.

It can be used as a link without only and without any idea of


misfortune: - e.g.
He went to his office to hear that he had got a promotion.

2.25.1.22.1.4. After verb to be to express an unfulfilled plan or arrangement: - e.g.


She was to have finished this report earlier. (But she did not)

2.25.1.23. 22) Verbs with infinitives:


Afford
Beg
Demand
Hesitate
Need
Promise
Struggle
Finish

Agree
Care
Deserve
Hope
Offer
Refuse
Threaten
-----------

Appear
Claim
Expect
Learn
Plan
Regret
Volunteer
------------

Arrange
Consent
Fail
Manage
Prepare
Remember
Wait
------------

Ask
Decide
Forget
Mean
Pretend
Seem
Want
----------

I can not afford to buy it.


They agree to help us.
She appears to be tired.
I will arrange to meet you at the airport.
He asked to come with us.
He begged to come with us.
I dont care to see that show.
She claims to know a famous movie star.
She finally consented to marry him.
I have decided to leave on Monday.
He demanded to know who is responsible.
She deserves to win the prize.
The cost of this book is to pray for me you have the permission to print it or do whatever you want

Get to the heart of English language grammar in easy way

118 of 195

I expect to enter a graduate school in the fall.


She failed to return the book on time.
I forgot to mail the letter.
Dont hesitate to ask for my help.
He hopes to arrive on Monday.
He learned to play the piano.
She managed to finish her work early.
I did not mean to hurt your feeling.
I need to have your opinion.
They offered to help us.
I am planning to have a party.
We prepared to welcome them.
He pretends not to understand.
I promise not to be late.
I refuse to believe his story.
I regret to tell you that you failed.
I remember to look the door.
That cat seems to be friendly.
I straggled to stay awake.
She swore to tell the truth.
She threatened to tell my parents.
He volunteered to help us.
I will wait to hear from you.
I want to tell you something.
She wishes to come with us.
2.25.1.24. ( Verbs followed by a (pronoun +an infinitive):
Advise
Challenge
Forbid
Need
Require
Warn
Watch

Allow
Convince
Force
Order
Teach
See
Let

Ask
Dare
Hire
Permit
Tell
Hear

Beg
Encourage
Instruct
Persuade
Urge
Feel

Cause
Expect
Invite
Remind
Want
Make

She advised me to wait till tomorrow.


He allows me to use his car.
I asked him to help me.
They begged us to come.
Her laziness caused her to fail.
She challenged me to race her to the corner.
I could not convince him to come to our party.
He dared me to do better than he ha had done.
He encouraged me to try again.
The cost of this book is to pray for me you have the permission to print it or do whatever you want

Get to the heart of English language grammar in easy way

119 of 195

I expect you to be on time.


I forbid you to tell him.
They forced him to tell the truth.
She hired a boy to mow the lawn.
They instructed him to be careful.
Tom invited the Johnsons to come to his party.
I want you to help us figure out the solution.
The judge ordered me to pay a fine.
He permitted the child to stay up late.
I persuaded him to come for a visit.
She reminded me to look the door behind me.
Our teacher required us to be on time.
My brother taught me to swim.
The doctor told me to take these pills.
I urged her to apply for the job.
I want you to be happy.
I warned you not to drive too fast.
I saw him steal the purse.
I made her buy the car.
I helped him to do the packing.
You dare not talk to him.
You would rather tell your father the truth.
You had better tell your father the truth.
Rather than leave him suffering, they took him to the hospital.
Sooner than see her broke, I lent her some money.
2.25.1.25.

After the following adjective or too+ adj:

Happy
Astonished
Disgusted
Sad
Young

Glad
Amazed
Disappointed
Lovely
Early

Sorry
Surprised
Pleased
Possible

Relieved
Horrified
Angry
Late

After enough; as,


He is fit enough to join the army.
He is intelligent enough to pass the exam.
He speaks clearly enough to be understood.
Example:
I pleased to introduce you to my family.
Our enemy was horrified to meet our brave soldiers.
She is very sorry to hear about her fathers death.
He was relived to find his stolen car.
It is lovely to see you all working together.
It was impossible to ask the boss for a leave.
It is too late to call him.
The cost of this book is to pray for me you have the permission to print it or do whatever you want

Get to the heart of English language grammar in easy way

120 of 195

It is too early to call him.


2.25.1.26.

State how the infinitive is used in the following sentences: There was nothing for it to fight.
Lets pray.
The mango is fit to eat.
I heard her sing.
I have come to see you.
The order to advance was given.
Men must work and women must weep.
I am sorry to hear this.
He is slow to forgive.
To retreat was difficult; to advance was impossible.
Everybody wishes to enjoy life.
My desire is to see you again.
There was not a moment to be lost.
The counsel rose to address the court.
My right there is none to dispute.
The ability to laugh is peculiar to mankind.
He has the power to concentrate his thoughts.
He was quick to see the point.
I am not afraid to speak the truth.
It is a panel offence to bribe a public servant.

2.25.1.27. Combine together the following pairs of sentences by using


infinitive: He did not have even a cent with him. He could not by a loaf of bread.
Every cricket team has a captain. He directs the other player.
You must part with your purse. On this condition only you can save
your money.
He went to Marib. He went to visit the damp.
The robber took out a knife. He intended to frighten the old man.
I speak the truth. I am not afraid of it.
The insolvents property was sold by the official Assignee. The
insolvents creditors had to paid.
He wants to earn his livelihood. He works hard for that reason.
The strikers held a meeting. They wished to discuss the terms of the
employers.
He has five children. He must provide for them.
The old man has now little energy left. He cannot take his morning
constitutional exercises.
The Rajah allowed no cows to be slaughtered in his territory. It was
his custom.
He formed a resolution. It was to the effect that he would not
speculate any more.
Every one should do his duty. India expects this of every man.
The cost of this book is to pray for me you have the permission to print it or do whatever you want

Get to the heart of English language grammar in easy way

121 of 195

She visits the poor. She is anxious to relieve them of their sufferings.
He collects old stamps even at great expense. This is his hobby.
He must apologize for his misconduct. It is the only way to escape
punishment.
I have no aptitude for business. I must speak it out frankly.
He was desirous of impressing his host. So he was on his behavior in
his presence.
That young man has squandered away all his patrimony. He must
have been very foolish.
He has risen to eminence from poverty and obscurity. It is highly
creditable.

GERUND
2.25.1.28. Gerunds: is the (ing) form of the verb. (e.g., playing)
Study the following:
V
N
V
1) Playing tennis is fun. =(It is the subject of the sentence.)
S
V
V
N
2) We enjoy playing tennis. =(It is used as the object of the verb enjoy)
V
PREP
V
N
3) He is excited about playing tennis. (It is used as the object of the
preposition about)
2.25.1.29. 20) Gerund used like (a noun)
1-20) As the subject of a verb:
Seeing is believing.
Hunting deer is not allowed in the country.
2-20) As the object of a verb:
Stop playing.
Children love making mud castles.
He contemplated marrying his cousin.
3-20) As the object of a preposition:
The cost of this book is to pray for me you have the permission to print it or do whatever you want

Get to the heart of English language grammar in easy way

122 of 195

I am tired of waiting here.


He is fond of swimming.
He was punished for telling a lie.
We were prevented from seeing the prisoner.
I have an aversion to fishing.
4-20) As a complement of a verb:
Seeing is believing.
What I most detest is smoking.
2.25.1.30. 5-20) After the following expressions: 2.25.1.30.1.1.1.1.1. It is no use/ good.
2.25.1.30.1.1.1.1.2. Can not stand/ help.
2.25.1.30.1.1.1.1.3. Would/ dont mind.
It is no use waiting for him; he is not coming.
Would you mind opening that window?
I can not stand walking in the sun.

2.25.1.31. 24) Have+{fun/a good time/trouble/ difficult time/ a hard time}+


verb+ ing:
a) I have fun playing tennis.
b) I had a hard time finding your home.
c) I had a difficult time finishing this work.
d) I had a good time chatting with you.
2.25.1.32. 26) The perfect gerund; as,
1) Having finished my work, I went home.
2) He was accused of having killed an old lady.
2.25.1.33. 27) The Passive gerund (being + pp) or (having+ been+ pp); as,
2.25.1.33.1.1.1. I remembered being knocked down by the crowd.
2.25.1.33.1.1.2. The remaining of the building shows no sign of having been bombed.
2.25.1.34. 28) {Spend/ waste/ sit/ stand/ lie/ find/ watch} + phrase +ing:
a) I spent most of time studying.
b) I lay most of the time thinking.
c) I stood there wondering what to do next.
d) I found her using my phone.
i) Wasted my time looking for him.
j) I watched him dancing happily.
k) I found him cheating.
The cost of this book is to pray for me you have the permission to print it or do whatever you want

Get to the heart of English language grammar in easy way

123 of 195

l) I sat there reading.


2.25.1.35.

21) Verbs followed by (gerund)

Consider

Anticipa
te
Delay

Appreciat
e
Deny

Dislike

Enjoy

Finish

Forget

Keep

Mention

Mind

Miss

Practice

Quit

Recall

Recollect

Regret

Rememb
er

Resent

Resist

Stop

Suggest

Tolerate

Prevent
Can not
stand

Forgive
It is no
use

Excuse
It is no
good

Admit

Advise

Complete

Understan
d
Give up
Dont
mind

Imagine
Discuss
Can not
help
Postpone
Recommen
d
Risk
Avoid
Put off
Would
mind

He admitted stealing the money.


He advised waiting till tomorrow
He anticipated having a good time on vacation.
He appreciated hearing from them.
He avoided answering my question.
I finally completed writing my term paper.
I will consider going with you.
He delayed leaving from school.
She denied committing the crime.
The discussed opening a new business.
I dislike driving a long distances.
We enjoyed visiting them.
She finished studying about ten.
I will never forget visiting the prophet mosque.
I can not help worrying about it.
I keep hoping he will come.
She mentioned going to a movie.
Would you mind helping me with this?
I miss being with my family.
Let's postpone leaving until tomorrow.
The athlete practice throwing the ball.
He quit trying to solve the problem.
I dont recall meeting him before.
She recommended seeing the show.
The cost of this book is to pray for me you have the permission to print it or do whatever you want

Get to the heart of English language grammar in easy way

124 of 195

I regret telling him my secret.


I can remember meeting him when I was child.
I resent her interfering in my business.
I could not resist eating the dessert.
She risks losing all of her money.
She stopped going to classes when she got sick.
She suggested going to a movie.
She will not tolerate cheating during an examination.
I dont understand his leaving school.
I can not stand walking in the sun.
No smoking.
It is no use waiting for him.

PARTICIPLE
2.25.1.36. 25) Participle: is a word, which is partly a verb and partly an
adjective. It consists of four:
1-25) Present participle: the words of participle here end in (ing) to
represent an action on going or not complete; as,
He met a girl carrying a basket of flowers.
I heard him singing in his room before he slept.
Past participle: The words of participle here end in
(ed/d/t/en/n) to represent a complete action; as,
{play/move/lose/drive/write}
I found him killed at my way home.
3-25) Perfect participle: The words of participle here end in
(ed/d/t/en/n) to represent a complete action at some past time; as,
Having rested, we continued our journey.
4-25) Participle adjective: The words of participle here end in
(ed/d/t/en/n/ing) here the past participle is used adjectively
as Passive in meaning but the present participle used as
Active in meaning; as,
A rolling
stone gather no moss.
The cost of this book is to pray for me you have the permission to print it or do whatever you want

Get to the heart of English language grammar in easy way

125 of 195

We had a drink of the sparkling water.


A lying witness ought to be punished.
He played a losing game.
A burnt child dreads the fire.
5-25) Use of participle:
1) Attributively; as,
A rolling stone gather no moss.
A burnt child dreads the fire.
2.25.2.

2.25.3.

2.25.4.
2.25.5.
2.25.6.

2) Predicatively; as,
The man seems worried. (Modify the subject)
He kept me waiting. (Modify the object)
3) To form the continuous tense; as,
She is reading in the library.
They have been working all day today.

4) After verb of sensation (see, hear, taste, smell, and feel); as,
She saw him waiting for the bus.
5) They can feel her shaking of cold.
I smell something burning.
6) When to action occur simultaneously, one of them participle; as,
He went to Sanaa cycling.
She stood watching the bus.
Ali came running.

2.25.7.
7) Two sentences with the same subject, the first is present
participle; as,
Finishing his work, he left for Paris.
Seeing the accident, he called the police.
2.25.8. 8) The second sentence is participle if it is part of the first sentence
or a result of it; as,
He fired, killing one of the deer.
Ali ran into a post, smashing the headlamp.
2.25.9. 9) To form perfect or Passive; as,
She has broken her leg.
Her leg has been broken.
The cost of this book is to pray for me you have the permission to print it or do whatever you want

Get to the heart of English language grammar in easy way

126 of 195

Having broken her leg, she went to the hospital.


2.25.10. 10) To imply a Passive voice; as,
The postman came in covered with snow.
She walked a long the street followed by a group of children.
2.25.11. 11) Absolutely phrase; as,
The weather being fine, I went out.
Many having arrived, we were freed from anxiety.
The wind having failed, the crew set to work with a will.
The master being absent, the business was neglected.
The wind being favorable, they embarked.
2.25.11.1.

Joining sentences with participle:


1-) Simple sentence:
He walked out of the room. He slammed the door behind him.
2-) Compound sentences:
He walked out of the room and slammed the door behind him.

3) Complex sentences:
When he walked out of the room, he slammed the door
behind him.
4-) Participle construction:
He walked out of the room, slamming the door behind him.
5-) We can make more than one participle:
After I looked the number up in the phonebook, I
made sure that I had got it right I phoned again.
After looking up their number up in the phone book
and making sure I had it right, I phoned again.
6-) Participle construction in place of Co-ordinate clause:
She lay awake all night. She recalled the events of the day.
She lay awake all night, and recalled the events of the day.
She lay awake all night recalling the events of the day.
7-) Participle construction in place of adverb of time:
{since, after, before, when, while}
Since I phoned you this morning, I have changed my plans.
Since phoning you this morning, I have changed my plan.
When I found the door opened, I became suspicious.

The cost of this book is to pray for me you have the permission to print it or do whatever you want

Get to the heart of English language grammar in easy way

127 of 195

On finding the door opened, I became suspicious.


During the time when I was trying to open the can, I cut
my hand.
In trying to open the can, I cut my hand.
8-) Participle construction with (being) and (having been)
He is so ill; he can not go back to school yet.
Being so ill, he can not go back to school yet.
He was so ill, so he could not go back to school yet.
Being so ill, he could not go back to school yet.
He has been ill for a while, so he needs more time to
recover before he can go back to school. He can not
go back to school yet.
Having been ill for a while, so he needs more time to
recover before he can go back to school. He can not
go back to school yet.
He had been ill for a very long time, so he needed more
time to recover before he could go back to school.
Before he could go back to school yet.
Having been ill for a long time, so he needed more time
to recover before he could back to school. He could not
go back to school yet.
2.25.11.2. To join two sentences with participle at the beginning, the subject
in the second sentence must be the subject of the first one,
I found him eating. I left him alone.
Finding him eating, I left him alone.
1) With (there & it)
As it was a bank holiday, all the shops are shut.
It being a bank holiday, all the shops are shut.
As there is no further business, I declared the meeting closed.
There being no further business, I declared the meeting
closed.
2.25.11.3. Pick out the participle in each of the following, and tell what kind
it is: General speaking, we receive what we deserve.
Having gained the truth, keep truth.
The cost of this book is to pray for me you have the permission to print it or do whatever you want

Get to the heart of English language grammar in easy way

128 of 195

I saw the storm approaching.


Hearing a noise, I turned round.
Considering the fact, he received scan not justice.
The enemy, beaten at every point, fled from the field.
Being dissatisfied, he resigned his position.
The rain came pouring down in torrents.
Having elected him president, the people gave him their loyal support.
The traveler, being weary, sat by the Woodside to rest.
The fat of the body is fuel laid away for use.
Being occupied with important matter, he had no leisure to see us.
The children coming home from school look in at the open door.
Michael, bereft of his son Luke, died of a broken heart.
Lessons learned easily are soon forgotten.
A work fitly spoken is like apples of gold in picture of silver.
Seeing the sunshine, I threw open the window.
Seizing him by the arm, his friend led him away.
Encouraged by his wife, he preserved.
Overcome by remorse, he determined to atone for his crime by
liberty to the church.

2.25.11.4. Point out the gerund and participle and tell what kind it is:
He was found fighting desperately for his life.
He has ruined his sight by reading small print.
Hearing the noise, he ran to the window.
We saw a clown standing on his head.
Asking questions is easier than answering them.
Waving their hats and handkerchiefs, the people cheered the king.
Walking on the grass is forbidden.
Jumping over the fence, the thief escaped.
The miser spends his time in hoarding money.
Much depends Ramas returning before noon.
Amassing wealth ruins health.
I was surprised at Nabeel being polite.
We spent the after noon in playing cards.
The miser hated spending money.
She was angry with her boyfriend trying to lie to her.
Praising all alike is praising none.
Are you afraid of his hearing you?
I determine to increase my salary by managing a little farm.
The year was spent in visiting our rich neighbors.
Singing to herself was her chief delight.
He preferred playing football to studying his lesson.
I can not go on doing anything.
The cost of this book is to pray for me you have the permission to print it or do whatever you want

Get to the heart of English language grammar in easy way

129 of 195

2.25.11.5. Combine the following pairs of sentences by making use of


participle:
e.g. The magician took pity on the mouse.
He turns it into a
cat. Taking pity on the mouse, the magician turned it into a cat.
e.g. The train was ready to leave the station. The people had taken
their seats. The people having taken their seats, the train was
ready to leave.
2.25.11.5.1.1.1.
The porter opened the gate. We entered.
2.25.11.5.1.1.2.
We started early. We arrived at noon.
2.25.11.5.1.1.3.
We met a man. He was carrying a load of wood.
2.25.11.5.1.1.4.
The stable door was open. The horse was stolen.
2.25.11.5.1.1.5.
He seized his stick. He rushed the to the door.
2.25.11.5.1.1.6.
The hunter took up his gun. He went out to shot the lion.
2.25.11.5.1.1.7.
A crow stole a piece of cheese. She flew to her nest to enjoy the
tasty meal.
2.25.11.5.1.1.8.
The wolf wished to pick a quarrel with the lamb. He said,
How dare you make the water muddy?
2.25.11.5.1.1.9.
A passenger alighted from the train. He fell over a bag on the
platform.
2.25.11.5.1.1.10. Ali met his brother in the street. He asked him where he was going.
2.25.11.5.1.1.11.
My wife was charmed with the silk. She
bought ten yards.
2.25.11.5.1.1.12.
The steamer was delayed by a storm. She came into
port a day late.
2.25.11.5.1.1.13.
He had resolved on a certain course. He acted
with vigor.
2.25.11.5.1.1.14.
He staggered back. He sank to the ground.
2.25.11.5.1.1.15.
The letter was badly written. I had great
difficulty in making out its content.
2.25.11.5.1.1.16.
They had no fodder. They could give the cow
nothing to eat.
2.25.11.5.1.1.17.
A hungry fox saw some bunch of grapes. They
were hanging from a vine.
2.25.11.5.1.1.18.
Cinderella hurried away with much haste. She
dropped one of her little glass slippers.

The cost of this book is to pray for me you have the permission to print it or do whatever you want

Get to the heart of English language grammar in easy way

130 of 195

DIRECT AND INDIRECT QUESTION

2
0
1

Direct questions start with either yes/no question or wh/question.


1) Where is he?
2) Is he at home?
Indirect question starts with (a yes/no question) or (a sentence).
(Do you know /can you tell me / would you tell me).
(I dont know / I want to know)

To change a question from DIRECT into INDIRECT do the following:


2 Change the question from question into a sentence.
Where is he ?
Where he is .
Is he at home?
He is at home.
3 Add to the beginning of indirect question one of the following (do you

know /can you tell me / would you tell me) or (I dont know/ I want to
know).
4
(?).

If the indirect question starts with (a yes/no question) put at

the end

Do you know where he is?


5

But if it starts with a sentence add to the end (.)


I want to know where he is .
6
If the
direct question starts with ( a yes/no question) when you change it add
(if) before it.
Is he at home?
(2) I dont know if he is at home.
(3) Do you know if he is at home?

The cost of this book is to pray for me you have the permission to print it or do whatever you want

Get to the heart of English language grammar in easy way

131 of 195

Example:
1) Where do you go?
2) Where you go.
I dont know where you go.
1) Does he go to school?
2) He goes to school.
I dont know if he goes to school.
1) When can I find him?
2) When I can find him. Do you know when I can find him?
1) Why will you go there?
2) Why you will go there. I dont know why you will go there.

Do the following exercise:


1. What are you doing?
2. When will you come back home?
3. Can you help me?
4. Where does he work?
5. Why did he scream?
(note) for more information see noun clause.

The cost of this book is to pray for me you have the permission to print it or do whatever you want

Get to the heart of English language grammar in easy way

132 of 195

Direct and indirect speech


3
4

There are four kinds;


1) Statement
3) Command
8-

2) question
4) exclamation

Statement:
8.6) He said to me, I will see you tomorrow
Change the following into
No
1)
2)
3)
4)
5)
6)
7)
8)
9)
10)
11)
12)
13)
14)
15)
16)

Change from
Now
This
These
Here
Today
Tomorrow
Yesterday
Ago
Last week
Thus
Future
Present
Past
Say to
Says to
Said to

To
Then
That
Those
There
That day
The next day
The day before
Before
The week before
So
Past
Past
Past perfect
Tell
Tells
Told

The cost of this book is to pray for me you have the permission to print it or do whatever you want

Get to the heart of English language grammar in easy way

133 of 195

17)
Saying to
Telling
18)
Has said
Has told
19)
Come
Go
20)
I
He
21)
Ask
Require
Note: (with order we use ordered, told, commanded, urge)

5 How to form it:


My brother is a student.
... She told me that my brother was a student.
1) She said, My brother is a student .
2) My brother is a student . she said .
3) My brother she said is a student .
My brother is a student. He is attending a university.
They said to us that her brother was a student and that he was
attending a university.
1) She said , My brother is a student . He is attending a
university.
2) She said , My brother is a student He is attending a
university she added .
When will you be here?
She asked when would you be here .
1) She asked , When will you be here?
1

10

Direct speech

No
1)

She said, I am watching TV.

2)

She said, I watch TV.

3)

She said, I will watch TV.

4)

She said, I am going to watch


TV.

Indirect speech
She said that she was
watched TV.
She said that she watched
TV.
She said that she would
watch TV.
She said that she was going
to watch TV.

The cost of this book is to pray for me you have the permission to print it or do whatever you want

Get to the heart of English language grammar in easy way

5)

She said, I may watch TV.

6)

She said, I have to watch TV.

7)

She said, I must watch TV.

8)

She said, I should watch TV.

9)

She said, I watched TV.

10)

She said, Write your


homework.

12)

She said, When do you watch


TV?
She said, Do you watch TV?

13)

He says, I am glad to be here.

11)

14)
15)
16)
17)
18)
19)

He said to me, What are you


doing?.
Where do you live? asked the
stranger.
He said, Will you listen to such
a man?
The manager said to him, Go
away.
He said to him, please wait
there till I return.
Call the first witness, said the
judge.

20)

He shouted, Let me go.

21)

He said, Be quiet and listen to


my words.

22)

He said, Alas! I am undone.

23)
24)

Abduladif said, How clever I


am!
He said, Bravo! You have done
well.

134 of 195

She said that she might


watch TV.
She said that she had to
watch TV.
She said that she had watch
TV. (Or) she must have
watched TV
She said that she should
watch TV.
She said that she had
watched TV.
She told me to write my
homework.
(Or) order me
She asked when I watched
TV.
She asked if I watched TV.
He says that he was glad to
be there.
He asked me what I was
doing.
The stranger enquired
where I lived.
He asked them whether
they would listen to such a
man.
The teacher ordered him to
go away.
He requested him to wait
there till he return.
The judge commanded them
to call the first witness.
He shouted to them to let
him go.
He urged them to be quiet
and listen to his words.
He exclaimed sadly that he
was undone.
Abduladif exclaimed that he
was very clever.
He applauded him, saying
that he had done well.

The cost of this book is to pray for me you have the permission to print it or do whatever you want

Get to the heart of English language grammar in easy way

25)
26)
27)
28)

29)

So help me, Heaven! he cried,


I will never steal again.
He said to me, I dont believe
you.
He said to him, I dont believe
you.
I said to you, I dont believe
you.
Curse it! exclaimed the driver,
who could have foreseen such
ill-luck? But for accident we
should have caught the train
easily.

135 of 195

He called upon Heaven to


witness his resolve never to
steal again.
He said he did not believe
me
He said to him I dont
believe him.
I said I did not believe him.
The driver exclaimed with
an oath that nobody could
have foreseen such ill-luck.
But for the accident they
would have caught the train
easily.

6 {NOTE} with (truth, immediate report, or after says/has said/will say /past of
modals) there is no change in the indirect speech:
he said, the earth is round.
He said that the earth is round.
He says, He is busy.
He says that he is busy.
He has said, He is busy.
He has said that he is busy.
He will say, I will watch TV.
He will say that he watches TV.
I did not here him. What did he just say?
He said he wants us to write after him.
Do the following exercise:
He said, Where are you?
He said, I must go.
He said, I am happy.
He said, I run faster than you.
He said, You came late.
He said, You ought go home early.
He says, He is absent.
He said, Water boils at 100o .
He said to me, I have often told you not to play with fire.
You have done very badly! remarked the teacher.
They wrote, It is time to settling with this matter.
The examiners orders were no one is to bring books into the
room nor ask me questions about what I have told you to do.
The cost of this book is to pray for me you have the permission to print it or do whatever you want

Get to the heart of English language grammar in easy way

136 of 195

The dwarf said to her, promise me that when you are queen you
will give me your first born child.
That is my hors, he said, And if I dont prove it in a few
minutes I will give up my claim.]
I will avenge your wrongs, he cried, I will not enter Athens
until I have punished the king who had so cruelly treated me.
He wrote and said, I am unable to come just now because I am
ill, but I will certainly start as soon as I am well enough to do so.
One day he sent for Nabeel and said to him, you are now old
enough to earn your living, so you must set off, and make your
own way in the world.
What do you want? he said to her.
Are you coming home with me? he asked.
He enquired, When do you intend to pay me?
The poor man exclaimed, Will none of you help me?
Bring me a drink of milk. Said the swami to the villagers.
Sit down, boys said the teacher.
Halt! shouted the officer to his men.
Run away, children said their mother.
What a rare article milk is, to be sure, in London! Said Mr. Anwar
Amer with sigh.
He said, My god! I am ruined.
He said, Alas! Our foes are to strong.
Ah me! exclaimed the queen, What a rash and bloody deed
you have done!
Put the following in direct speech:
He asked Adel to go with him.
Adel replied that he could not do so.
Mr. Osama Almadani asked me what had become of Khaled.
The master requested that they would attend carefully to what he
was saying.
I told them to be quiet.
An old mouse asked who would bell the cat.
I asked Mary if she would lend me a pencil.
He said that though he had come, it was against his will.
He asked him to leave the room, and forbade him to return.
Osama asked Amar if he had read the drama.
The magistrate asked the prisoner what he was doing with his hand
in the gentlemans pocket.
He advised his sons not to quarrel among themselves, when he was
dead but to remain united.
The line told the fox that he was very weak, that his teeth fallen out,
and that he had no appetite.
He replied that he had promised to reward his soldiers and that he
had kept his word.
The cost of this book is to pray for me you have the permission to print it or do whatever you want

Get to the heart of English language grammar in easy way

137 of 195

The speaker said that it gave him great pleasure to be there that evening.
{NOTE} (Say/ Said + Subject). But (Said To/ Told + Object.)
He said he would come the next day.
He said to me he would come the next day.
He told me he would come the next day.
TAG QUESTION
7
8 QUESTION TAG : ask about conformation.
He is a student, is not he?

1) HOW TO FORM IT:


we form it by adding (be, do, have or modals + pronoun).

If the sentence affirmative, the tag will be negative.


He is at home, is not he?
9- If the sentence is a negative, the tag will be affirmative
He is not at home, is he?
3) If there is auxiliary in the first part, it must put it in the later
part at first+(not) subject?
We are friends, are not we ?
4) But if there is no auxiliary we should add (do/does) in present as,
9.6) He enjoys himself, does not he?
and (did) in the past as,
9.7) {He enjoyed himself, did not he?}
5) If the sentence starts with (I am) the tag should be (are not I) as,
9.8)
I am happy, are not I?
9.9)
(But look ) I am not a teacher, am I?
9

{The tag question must always be a pronoun not a noun as,


9.10)
Osama is clever, is not he?

The cost of this book is to pray for me you have the permission to print it or do whatever you want

Get to the heart of English language grammar in easy way

138 of 195

9.10.1.1.1. Look at these exceptions:


No
1)

Starts with
I am
Some of, none
of, Anyone,
anybody,
Somebody,
someone,
everyone, each
one, anyone,
no one,
nobody
Anything,
Something,
every thing,
nothing

Change to
Are not I?

Example
I am a teacher, are not I?

They

Someone is calling, are not


they?

It

Everything is wrong, is not


it?

4)

Suggestions

Shall

5)

Orders/
Requests

6)

This/ that

It

7)

These/those

They

8)

Used to

Used to/
did

2)

3)

1)

2)

Will/
Can/wont

Lets play billiards, shall


we?
Pass me the salt please, will
you?
Dont play here, will you?
This house is wonderful, is
not it?
These are good books, are
not they?
They used to get up early,
used not/ did not they?

{Note} if there are more than one helping verb we should


use the first; as,
He will have finished by ten, wont he?
The answer to such questions is as the following;
1) If the first part is affirmative, the expected answer is affirmative
as, {He is handsome, is not he? Yes he is.}
2) If the first part is negative, the answer expected is negative as, {He
is not handsome, is he? No he is not.}

The cost of this book is to pray for me you have the permission to print it or do whatever you want

Get to the heart of English language grammar in easy way

139 of 195

{NOTE} Sometimes we follow affirmative with affirmative tag question to


show interest in something or just to be polite as, {you are a
teacher, are you?} {Your shoes are tight, are they?}

Fill in the planks with suitable question tag:


He will help me, ?
She takes care of her young brother, ?
Each one should do his duties, ..?
Every thing is going right,.?
Do your homework, .?
Let's call he after five minutes, ..?
I am not sad, .?
Anwar is really is a great scholar, ?
She is not very intelligent, ..?
Those officers were not very helpful, ?
We had completed the work before he came, ?
We must go now, ?
I shall expect a letter tomorrow, .?
He speaks English fluently, ?
You used to visit her every other day, .?
He never say a word, ..?
Please give me a glass of water, ..?
Kindly do me a favour, ..?
She dared me to do that, ..?
You dare not jump from that height, .?
You need some money today, .?
You need to see a doctor, ?
You dont need to buy this book, ?
Let me help you, .?
Lets get down to our work now, ..?
Please do let us eat before we go out, .?
A few people will certainly help her, ..?
Few people here would agree to his proposal, ?
I was willing to help him, .?
you should have been there, ..?
nobody here can do it for him,.?
Someone must take it up now, ?
One of you should start it now, ?
None of those boys were honest, ..?
Some of them have agreed to your proposal, .?
Dont be late tomorrow, .?
Something can do, .?
Someone can help, .?
No one can do it, ?
The cost of this book is to pray for me you have the permission to print it or do whatever you want

Get to the heart of English language grammar in easy way

140 of 195

Nothing can satisfy her, ..?


Make an answer to the former tag questions.

0
1

{CONDITIONAL CLAUSE} OR {IF CLUES}


There are three kinds of it;
1) Likely
2) Unlikely

3) Impossible

Likely: it is in present tense, which can be true someday.


The structure of it:
a) If +subject + present subject + will + verb.
If I have money, I will buy this car.
Unlikely: it is in present but we dont think if it comes true or not.
The structure:
b) If +subject + past subject + would + verb.
If I had money, I would buy this car.
Impossible: it is in the present but it can not be true at all.
The structure of it:
c) If +subject + past perfect subject + would+ have +P.P.
If I had had wings, I would have flew.
True in the present or future; as,
It is raining right now, so I will not go for a walk.
If it is not raining now, I will go for a walk.
o
Untrue in the present or future; as,
It is raining right now, so I will not go for a walk.
if it were not raining now, I would go for a walk.
o
Untrue in the past; as,
I did not have a gun, so I did not kill him.
If I had had a gun, I would have killed him.
o
Omitting if; as,
Had I had a gun, I would have killed him.
Were I you, I would not do that.
Should any one call, please take a message.
Had not seen him with my two eyes, I would not have believed it.
Implied condition; as,
I would have gone with you, but I had to study. (= If I had not had to study)
o

The cost of this book is to pray for me you have the permission to print it or do whatever you want

Get to the heart of English language grammar in easy way

141 of 195

I would not have succeeded without your help. ( = if you had not helped
me)
He ran, otherwise, he missed his bus.
2-18) Wish : It can be followed by several different tenses such as:A) Wish+ past simple: to express an unrealistic desire for the present situation
to be different (note) the pronoun used here is the same person with ;
as,
Example:I wish I were rich. (or I was rich)
He wish he were rich.
I wish I lived in America.
Do you wish we lived nearer the school?
Rewrite the following sentences using ( wish + past simple )
Id love to speak more languages.
I hate having to go to school on Sunday.
Why don we have a bigger house?
Why is the school so expensive?
B) Wish+ past perfect: to express an unrealistic desire for an action or
situation in the past wish is impossible (note) the pronoun used here is
the same person with ; as,
Example:Kamal did not come.
I wish Kamal had come.
Adel could not come
I wish Adel could not have come.
Rewrite the following sentences using ( wish + past perfect )
I met Ali.
I went to the party.
I decided to work in London.
We put our money into a grocery shop.
We did not realize that a supermarket was opening nearly.
C) Wish+ would+ infinitive: to express annoyance with a person or situation
and a desire for a situation to be changed by someone either in the
present or the future (note) the pronouns used here are different with;
as,
Example:I wish he would give up smoking.
I wish it would stop raining.
Rewrite the following sentences using ( wish + past simple )
The cost of this book is to pray for me you have the permission to print it or do whatever you want

Get to the heart of English language grammar in easy way

142 of 195

Peter does not wash very often.


Sami takes too money days off work.
John does not wear a tie very often.

(NOTE) If only can be used instead of wish in all three constructions


described above and its stronger and more unrealistic than wish:Example:If only I was rich.
If only I had not stolen money.
If only it stop raining.
A) In the future:
She will not tell me.
I wish (that) she would not tell me.
He is not going to be there.
I wish he were not going to be there.
She can not come tomorrow.
I wish she could come tomorrow.
B) In the present:
I dont know French.
I wish I knew French.
It is raining right now.
I wish it were not raining right now.
I can not speak English.
I wish I could speak English.
D) Untrue idea with as if/ though:
1.2.1.1.
He is not a child, but he talked as if he were a
child.
He did not take a shower, but when he came in from the rainstorm,
He looked as though he had taken a shower with his clothes on.
2.25.11.6.
Use if instead of unless but for otherwise.
Unless you practice what you learn, you will soon forget it.
Unless we eat, we could not live.
But for the money he sent me, I would not have traveled.
But for the fine weather, we can not travel.
You must answer the first question; otherwise, you will lose 20 marks.
I always arrive on time; otherwise, the teacher wouldnt allow me to enter
the class.
I had written to him first, he wouldnt have sent me a letter.
The cost of this book is to pray for me you have the permission to print it or do whatever you want

Get to the heart of English language grammar in easy way

143 of 195

Put on your coat; otherwise you will catch a cold.


Dont touch that bomb; otherwise it may explode.
But for the air we breathe, life would be impossible.
Unless you reserve a seat soon, you will not travel tomorrow.
Unless he were here, I couldnt speak to him.
But for his bad temper, he would have lived happily.
He was very careful; otherwise a terrible accident would have taken place.
But for him, we would all be miserable.
Had not taken the injection, he would have died.
Were he a little wiser, he would not answer his father back.
2.25.11.7.
Complete the following sentences with a main clause from your
own.
If he drive too fast,.
If it begins raining,..
If people fell sick,.
Water evaporates if..
I will help you if
If had taken the pill,..
If you eat too much,..
If I did not find my glasses,
Jasmine could lose weight if..
If he had treated me well,

PHRASE
Adjective phrase: is a group of words does the work of an adjective; as,
The vizier was a wealthy man.
The vizier was a man of great wealth.
The cost of this book is to pray for me you have the permission to print it or do whatever you want

Get to the heart of English language grammar in easy way

144 of 195

The magistrate was a kind man.


The magistrate was a man with a kindly nature.
The chief lives in a stone house.
The chief lives in a house built of a stone
I like to see a smiling face.
I like to see a face with a smile on it.
Pick out the adjective phrase, (2) and then replace the adjective phrase by an
adjective, (3) change each sentence into adjective clause:
A man in great difficulties came to me for help.
He is a person of very considerable renown.
Wild beast in small cages are a sorry sight.
A man without an enemy is a man with a few friends.
He tells a tale with a ring of truth in it.
A friend in need is a friend indeed.
A bird in a hand is worth two in the bush.
In a low voice he told the tale of his cruel wrong.
The police arrested a man of one of the criminal tribes.
He was a lad of great promise.
He wore a turban made of silk.
He has done a deed of shame.
He led a life devoid of blame.
He is a man without a friend.
They came to a path cover with mud.
He carried a sword stained with blood.
I met a little girl from a cottage.
Sadam was a man with plenty of impudence.
From this village in the mountains came a chieftain of great fame.
Kamal Suraim was a soldier full of hope and free from fear.
Nobody likes a person with a bad temper.
I admit that Abdulraheem Al-Ethawi is a man of sense.
The top of the mountains is covered with snow.
Osama Nabeel is an author of great versatility.
It is of no use.
Adverb phrase: is a group of words does the work of adverb; as,
Bassim Ran quickly.
Bassim ran with great speed.
He answered rudely.
He answered in a very rude manner.
He does his work carelessly.
The cost of this book is to pray for me you have the permission to print it or do whatever you want

Get to the heart of English language grammar in easy way

145 of 195

He does his work without any care.


He is coming now.
He is coming at this very moment.
No such diseases were known then.
No such diseases were known at those days.
The arrow fell here.
The arrow fell on this spot.
You can buy it everywhere.
You can buy it in all places.
He fell down.
He fell to the ground.
No

Adverb

1)

Bravely

2)

Unwisely

3)

Beautifully

4)

Formerly

5)
6)
7)
8)
9)

Recently
Soon
There
Away
Abroad

Adverb phrase
In a brave manner, or with
bravely
In an unwise manner, or
without wisdom
In a beautiful style.
In former time, or once upon a
time
Just now, or at a recent date
Before very long, or at early date
At that place.
To another place
In a foreign country

In each of the following sentences replace the adverb phrase by an adverb:-

The bodies were mangled in a impudent


way.

Let us cease work from this very moment.


It was just in this spot that he died.

The cost of this book is to pray for me you have the permission to print it or do whatever you want

Get to the heart of English language grammar in easy way

146 of 195

The children replied with perfect


truthfulness.

He arrived at that moment.


He that he will come at a very early date.
He seems to have acted with great promptitude.
No one would dare to answer him in and terrible manner.
I accept your statement without reserve.
He succeeded in the long run.
He is ignorant to a proverb.
The post boy drove with fierce carrier.
He has been painted in his proper colours.
The wind blew with great violence.
He has proved his case to my satisfaction.
Replace the following adverbs by an adverb phrase:10 The pigeon flies swiftly.
11 Did Rama behave well?
12 Go away.
13 The dying man replied feebly.
14 Gently fell the rain.
15 We will pitch the tents just here.
16 He expects to get promotion soon.
17 He built his house yonder.
18 They have only recently arrived.
19 Although hungry, the soldiers worked cheerfully.
20 He spoke eloquently.
21 Soon the sun will set.
22 Do your homework thoroughly.
23 They were hurrying homeward.
24 Formerly he worked at the school of economic.
25 He tried hard.
26 The door was suspiciously open.
The cost of this book is to pray for me you have the permission to print it or do whatever you want

Get to the heart of English language grammar in easy way

147 of 195

Which of the following adverb phrase and which are adjective phrase:Have you heard of the man in the moon?
How could be a man in the moon?
They live on an island.
A house on an island was washed away.
Awful is the gloom beneath her.
Is this the train to Cairo?
It usually goes to Cairo, sir.
Use the following phrases in sentences:{ in a loud voice; without further delay; with one voice; for certain;
just in time; up in arms; of no consequence; out of fashion; with great
satisfaction; in the twinkling of an eye; on either side of the street; in a
shady nook; to the last man; with a smile; at sexes and sevens; at the
eleventh hour; on the top of the hill; in future; at nine oclock}
NOUN PHRASE: is a group of words does the work of a noun; as,
Early to bed is a good maxim.
He hopes to win the first prize.
The boy wants to go home.
He enjoyed playing football.
To win a prize is my ambition.
Playing tennis is good for muscles.
Pick out the noun phrase in the following sentences:
His father wished to speak to the headmaster.
The wicked vizier loves getting people into trouble.
The poor debtor intended to pay back every Penny of the money.
He dislikes having to punish his servant.
Horses prefer living in dark stables.
I should hate to do such a thing.
Have you ever tried to climb a cocoanut palm?
Thinking good thoughts precedes good actions.
He refuses to answer the question.
To write such rubbish is disgraceful.
Promise to come again.
Why do you like visiting such a man?
Traveling in a hot dusty train gives me no pleasure.
He denies stealing the money.
Your doing such a thing surprises me.
Pick out the phrase and say whether they are adjective phrases, adverb
phrases or noun phrases:
He speaks like a born orator.
It grieves me to her of your illness.
Beyond a doubt this man is honest.
The cost of this book is to pray for me you have the permission to print it or do whatever you want

Get to the heart of English language grammar in easy way

148 of 195

He failed in spite of his best efforts.


He won the prize by means of trickery.
Dont talk like that.
I have forgotten how to play this game.
He gains their affection in spite of my faults.
I dont expect such treatment at your hands.
He speaks to fast to be understood.
I dont know what to do.
I dont understand how to solve this problem.
He persevered amidst many difficulties.
He succeeded in the long run.
Birds of a feather flock together.
This is a matter of no importance.
The train is behind time.
He is a man of means
It lies near his heart.
He keeps the necklace under luck and key.
He is a person of no importance.
I want to go the cinema today.
I love to hear the watchdogs honest bark.
I did it of my own free will.
Show me how to do it.
His car ran over a dog.
Things are in a bad way.
She is a woman of a wonderful patience.
I have found the key to his secret.
The plan has the virtue of committing us to nothing.
I dont see the point of the story.
How to find the way to the ruin is the question.
Osama Nabeel was a man of might.
He did it against his will.
I have no time to waste on trifles.
Dont do things by halves.
Supply a noun phrase:
I want
..delights me.
We all hope
Pretended .
..seems dishonest.
.surprised my father.
Do you wish.?
..Gives me no pleasure.
is not easy.
I dont intend ..
The cost of this book is to pray for me you have the permission to print it or do whatever you want

Get to the heart of English language grammar in easy way

149 of 195

clause
The cost of this book is to pray for me you have the permission to print it or do whatever you want

Get to the heart of English language grammar in easy way

150 of 195

Adverb clause: is a group of words which contains a subject and a predicate of


its own, and does the work of an adverb; as,
He has a chain of gold. ( phrase)
He has a chain which is made of gold.
( which) is subject. (is made of gold) is a predicate. But
(which is made of gold) is a Clause.
** study the following example:You may sit wherever you like.
He fled where his pursuers could not follow.
He behaves as one expected him to do.
Because you have done this I will punish you.
As he was not there, I spoke to his brother.
If you eat too much you will be ill.
He finished first though he began late.
Will you wait till I return?
Just as he entered the room, the clock struck.
They went where living was cheaper.
He does not always speak as he thinks.
Take a lamp because the night is dark.
If I make a promise I keep it.
He advanced as far as he dared.
I forgive you since you repent.
In each sentence replace the adverb phrase by adverb clause:On his return we asked him many questions.
Do it to the best of your ability.
The price was met on his arrival by his secretary.
In spite of poverty he became distinguished.
Upon seeing the signal the troop set out.
Nobody must expect to become rich without hard work.
They were too grateful to him for kindness.
In comparison with air water is heavy.
The weather is too bright to last.
My heart is too full for words.
The work is too much for any man to do single-handed.
With a view to early retirement he saved his money
In the event of the presidents death the vice president succeeds him.
He always carries out his duties according to instructions.
The price is high for an old car.
He ran with all his might.
After such a hard work, he requires a long rest.
He was base enough to accept the dishonorable terms.
Many ships were so shattered as to be wholly unmanageable.
A rose by any other name would smell as a sweet
The cost of this book is to pray for me you have the permission to print it or do whatever you want

Get to the heart of English language grammar in easy way

151 of 195

In each of the following sentences replace each adverb clause by adverb or


adverb phrase:I have not been well since I returned from Egypt.
When the sunset he returned home.
They fought as heroes do.
When the righteous rule, the people rejoice.
Though I am poor, yet am I contented.
We have come that we may help you.
When he entered the room he saw the vase broken.
The thief crept as a jackal does.
I am glad that he recovered from his illness.
He works hard so that he may become rich.
He worked so hard that he may succeed.
As soon as I saw the cobra, I ran away.
We ran so that we may arrive in time.
He jested even as he lay dying.
No man can become a great artist unless he applies himself continually
to his art.
There was nothing he would not do if only he might make profit.
He was not so rich that he could buy a motorcar.
When he had uttered these words he sat down.
The news is so good that it can not be true.
I did not pay him, as I had no money with me.
He lived carefully so that he might live long.
He may go home after his work is finished.
As he was sick. He remained at home.
He was punished, as he was deserved.
This sum is right so far as the working is concerned.
It rained so hard that the streets were flooded.
I took him because you recommended him.
My parents were poor though they were of noble birth.
He refuses to work whatever I may say.
We will do the work as well as we can.
Robinson Crusoe was puzzled when he discovered the print of a foot
on the sand.
He was passing rich though his income was only $40 annually.
We are kind to you because you are kind to us.
The passage is so difficult that I can not comprehend it.
Adjective clause or relative clause: is a group of words which contains a subject
and a predicate of its own (verb) and does the work of an adjective;
as,
The umbrella, which has a broken handle, is mine.
The letter brought money, which was badly needed.
The house that I live in belongs to my father.
I am monarch of all I survey.
I have a little shadow, which goes in and out with me.
The cost of this book is to pray for me you have the permission to print it or do whatever you want

Get to the heart of English language grammar in easy way

152 of 195

The dog that bites does not bark.


He tells a tale that sounds untrue.

(Note) we can change the adjective clause into


adjective phrase by omitting {who, which, or that} as,
Clause The girl who is sitting next to me is Mary.
Phrase The girl sitting next to me is Mary.
Clause Arabic has an alphabet that consists of 26 letters.
Phrase Arabic has an alphabet consisting of 26 letters.
In the following sentences replace each adjective
phrase by adjective clause: A man of industrious habits is sure to succeed.
He told us the time of his arrival.
The time for departing has now arrived.
Do you know the road leading to the temple?
I have a box, filled with almonds.
We all admire a man of courage.
A city on a hill cannot be hid.
The people in the gallery could not hear.
You can have anything of your liking.
The houses of the Barmans are generally built of bamboo.

In the following sentences replace adjective clause by


adjective or adjective phrase: Do you know the woman who is wearing a blue sari?
The boy who sits near me is my cousin.
That was the reason why he came late.
The reason, why he failed, is obvious.
The workers, who were weary with their exertion, lay down to rest.
The sun, which at midday was hot, made the traveler thirsty.
Which is the road that leads most quickly to the station?
People who eat too much die early.
Many men who have not been trained to write become journalists.
This is the place where our forefathers landed.
The explanation he gave was not satisfactory.
Such men as you cannot be easily disheartened.
This boy, who has been industrious, has earned a prize, which he has
well deserved.
A belief, which is generally held, is not necessarily one, which is true.
An author who was famous during the freedom struggle lived in that
cottage. Which overlooks the lake.

The cost of this book is to pray for me you have the permission to print it or do whatever you want

Get to the heart of English language grammar in easy way

153 of 195

NOUN CLAUSE: is a group of words, which


contains a subject, and a predicate of its own, and does the work of a
noun. It can be used as the subject of a sentence or the object of it; as,
What he said is completely wrong.
I dont believe what he said.
Here are words to introduce noun clause (who,
whom, that, which, whose, when, where, why, how)
I dont know where she went.
I could not hear what he said.
Do you know when they arrive?
What they should do is obvious.
I dont know whether she will come.
I wonder if she will come or not.
I dont know what I should do.
I dont know what to do.
I dont know whether to go or(to) stay.
He told us where to find it.
I think that he is a good actor.
I expect that I will get a prize
I often wonder how you are getting on.
I fear that I will fail.
He replied that he would come.
Do you deny that you steal the money?
No one knows who he is.
He saw that the clock had stopped.
I went to see what had happened.
I dont understand how it all happened.
He begged that his life might be spared.
That the world is round is a fact.
It is fact that the world is round.
Replace each noun clause by a noun or noun phrase:
I hope that I will be there in time.
He is sure that we will win the match.
I remarked that it was a fine day.
They dont know where he is concealed.
The police must know where he is living.
I believe what he says.
Tell me what you think about this.
The doctor is hopeful that she will soon recover.
It is to be regretted that he retired from the world so early in life.
I dont believe what he says.
He showed how the problem was done.
It seems that he is sharper.
The cost of this book is to pray for me you have the permission to print it or do whatever you want

Get to the heart of English language grammar in easy way

154 of 195

He does not know where I live.


Tell me why you did this.
It is not known who has written this book.
The law will punish whosoever is guilty.
Pick out the clause in each of the following sentences and what kind of clause it is:
Come when you like.
I know the man who is here.
He says that he met your brother.
The hand that rocks the cradle rules the world.
Before I die I intend to see Venice.
Let us rejoice as we go forward.
I was reading a book, which I had read before.
Perhaps he thinks that I am a fool.
As I drew near I saw a very curios sight.
Where are the friends whom I knew?
Can a man live whose Saul is dead?
I think that he will die.
I want to find the man who did this.
He made a vow that he would fast for a week.
It was not the vizier whom the king suspected.
He admitted that he wrote the letter.
That he will do it, I have no doubt.
I know the place, which you mention.
That such a thing, I could not believe it.
When he heard this he turned very pale.
Have you seen the horse that he has bought?
My father hopes that you will visit us.
He behaved, as a brave man should do.
I will wait till the next train comes.
The general feared that he would be surrounded.
He ate when he was hungry.
Since he has been in hospital, he has improved greatly.
As I was going in, my father came out.
There came a time when he was tired of waiting.
He spoke of a time when war should cease.
They live where the climate is good.
I know a place where roses grow.
They have gone to a land whither few traveler go.
They returned whence they had came.
Let no follow me to the retreat whither I now depart.
Sadly they returned to the prison whence they had so hopefully
set forth that morning.
You will always regret the day when you did this.

The cost of this book is to pray for me you have the permission to print it or do whatever you want

Get to the heart of English language grammar in easy way

155 of 195

Replace the words in italic by suitable noun phrase:


The time of his coming no one can guess.
I heard of his success.
The reason of his failure will never be known.
I predict a change in the weather.
The jury believed the man guilty.
We expect an improvement in business.
Who can doubt the truth of his statement?
I know him to be trustworthy.
I dont believe the account given by him.
He confessed his guilty.
He described the attack on the camp.
The place of their meeting was known to the police.
The soldier told us the direction of the wind.
His arrival was quite unexpected.
His friends hoped for his success.
I know your regard for him.

JOINING WORDS

The cost of this book is to pray for me you have the permission to print it or do whatever you want

Get to the heart of English language grammar in easy way

156 of 195

RELATIVE PRONOUN

9.10.2.

A) {CLAUSE} is a group of
words containing a subject + verb.

The Clause : is divided into:


1-A) INDEPENDENT CLAUSE OR MAIN CLAUSE WHICH can stand by
itself (She helped me.)
2-A) DEPENDENT CLAUSE OR SUBORDINATE CLAUSE, WHICH IS NOT
A COMPLETE SENTENCE AND IT, needs to be connected to a main
clause to make sense. (I thanked the woman)

If the repeated in the second sentence is

Pronoun

Nominative

WHO

Who

Whose

Who/whom

Which

Of Which/
whose

Which

WHICH

THAT

That

Genitive

Accusative

That

Used with
Singular,
plural,
male,
female
Singular
&plural.
Used for
things and
animals
Singular ,
plural,
male &
female. It
can be used
instead of
on which,
in which, at
which.

The cost of this book is to pray for me you have the permission to print it or do whatever you want

Get to the heart of English language grammar in easy way

157 of 195

Other words or subordinating conjunctions


used to introduce adverb clause:

NO

KIND

1)

TIME

2)

CAUSE &EFFECT

3)

OPPOSITION

4)

CONDITION

USED WORDS
After, before, when, while, as, by the
time, since, until, as soon as, once, as/so
long as, whenever, every time (that), at
first time (that), at last time (that) the
next time.
Because, since, now that, as, as/so long
as, inasmuch as, so that, in order that
Even though, although, though,
whereas, while
If, unless, only if, whether or not, even
if, providing that, provided that, in
case that, in the event that.

The moment, which is lost, is lost forever.


The man who is honest is trust.
He who hesitates is lost.
Blessed is he who has found his work.
They never fail who die in great cause.
The horse, which I recently bought, is an Arab.
I remember the day on which he came.
I remember the day that he came.

1) To add relative CLAUSE do the

following;
The two sentences must have identical element; as,
The man is a doctor. He is standing over there.
The man is rude. I hit him yesterday.
The house is big. I live in it.
Omit the repeated pronoun.
The man is a doctor. He is standing over there.
Write the subject of the first sentence.
The man
Write after the subject the suitable relative pronoun.
The man who

The cost of this book is to pray for me you have the permission to print it or do whatever you want

Get to the heart of English language grammar in easy way

158 of 195

Write after the relative pronoun the rest of the second sentence.
The man who is standing over there
O
O
o Now write the rest of the first sentence.
The man who is standing over there is a doctor.
The man is a doctor. He is standing over there.
(1)
(2)
(3)
The man who is standing over there is a doctor.
2.26) EXAMPLE:
The woman is an engineer. She is laughing.
The woman who is laughing is an engineer.
The car is white. I bought it.
The car, which I bought, is white.
The boy is crying. I beat him.
The boy whom I beat is crying.
The boy is crying. His bike is stolen.
The boy whose bike is stolen is crying.
{Note} (Whose) must be between two nouns.
The man whose bike is stolen is crying.
{NOTE} If the subject is proper noun, we should put the second sentence
between commas; as,
Mr. Smith is a doctor. He is sitting over there.
Mr. Smith, who is sitting over there, is a doctor.
{Note} if there are a person and an animal in the same sentence we must use
(that); as,
The man and his dog were turned out. They had trespassed on the
club premises.
The man and his dog that had trespassed on the club premises were turned
out.
JOIN THE FOLLOWING SENTENCES WITH A SUITABLE RELATIVE
PRONOUN:
The ox was very fast. We could not catch him.
You must not eat the fruit. The fruit is unripe.
The girl is my friend. She came here.
My friend swims well. He lives here.
The man is very rich. I bought his car.
I live in a house. A garden surrounds the house.
The pen is very good. You gave me yesterday.
Monday is the day. We will come then (on that day).
The cost of this book is to pray for me you have the permission to print it or do whatever you want

Get to the heart of English language grammar in easy way

159 of 195

7:05 is the time. My plane arrives then (at that time).


1960 is the year. The revolution took place then.
I shouted at a man. He did not come back.
I dropped a television. It never worked again.
I hired a machine. It was broken.
She bought some clothes. They were beautiful.
They built a wall. It fell down after three week.
I asked a policeman. He was not very helpful.
There is a lady. Her dog was killed.
They are the people. Their shop burned down last week.
I am the person. You stay in my flat.
Change the following sentences into their original shape:
The answer, which you gave, is not right.
I know the woman whose child is hurt.
Bring me the letters, which the postman left.
We met the soldiers whose their ship wrecked.
Here is the book that you lent me.
Dogs soon know the persons by whom they are kindly treated.
They that seek wisdom will be wise.
I hate children who are cruel.
Show me the knife that you have bought.
We always like boys who speak the truth.
We saw the dog that worried the cat.
{Note} (That) can be also used after interrogative pronoun (who & what); as,
Who that saw her did not pity her.
Who am I that I should object?
What is it that troubles you so much?
What is there that I dont know?
{Note} After all, same, any, none, nothing, the only; as,
All is not gold that glitter.
He is the same man that he has been.
It is only donkeys that bray.
It was not for nothing that he studied philosophy
Man is the only animal that can talk.
{Note} After the superlative Degree; as,
He was the most eloquent speaker that I ever heard.
The wisest man that ever lived made mistake.
This is the best that I can I do.

The cost of this book is to pray for me you have the permission to print it or do whatever you want

Get to the heart of English language grammar in easy way

160 of 195

{Note} if the person in the first sentence is not the same person in the second
sentence, we cannot combine them; as,
=
Ahmed is a doctor. Amer is sitting over there.
SEPARATE THE FOLLOWING INTO THEIR ORIGINAL:
The doctor who examined me was very good.
The people I was waiting for were late.
The man whose opinions I respect most is my friend.
Did I tell you about the woman I met last night?
The woman I was dancing with stepped on my toe.
The doctor who examined the sick child was very gentle.
The people I was waiting for was late.
The term paper David is writing must be finished by Friday.
The man whose opinions I respect most is my father.
Did I tell you about the woman I met last night?

2)

We can change the place of the preposition before the


relative pronoun but we must change the relative from subject to object {who
whom, which which}

EXAMPLE:
1011121314-

1) The person is an engineer. I talked to him.


The person to whom I talked is an engineer.
The house is large. I live in it.
The house, which I live in, is large.
The house in which I live is large.
The house I live in is large.

THE OMISSION OF THE RELATIVE PRONOUN:


IF THEY ARE OBJECT.
If they are not preceded by a preposition.
If they are not preceded by commas.
The man whom I met yesterday was my brother.
The man I met yesterday was my brother.
{Note} (Which) is a subject if it is followed by a verb and object if it is
followed by a pronoun.
The car, which is white, is mine.
The cost of this book is to pray for me you have the permission to print it or do whatever you want

Get to the heart of English language grammar in easy way

161 of 195

The car, which I bought, is white.


We can use (that) instead of (who, whom, which) if they;
Are not preceded by a preposition.
Are not between commas.
The man whom I met yesterday is my brother.
The man that I met yesterday is my brother.
We can not omit (whose) or use a preposition before it or but it between
commas; as,
2.26.1. The boy is happy. His bike is found.
2.26.2. The boy whose bike is found is happy.
using adjective clause to modify pronouns; as,
2.26.2.1.1.1.1. Tom was the only one I knew at the party.
2.26.2.1.1.1.2.
Scholarships are available for those who need financial
assistance.
2.26.2.1.1.1.3.
There is someone whom I wanted to meet.
2.26.2.1.1.1.4.
Everything he said was pure nonsense.
2.26.2.1.1.1.5.
Anybody who wants to come is welcome.
2.26.2.1.1.1.6.
It is I who am responsible.
2.26.2.1.1.1.7.
He who laughs last laughs best.
(note) an adjective clause with {which} can be used to modify the pronoun
{that}; as,
The bread my mother makes is much better than that which
the baker makes.
2.26.2.2.
we use of {whom, which, whose} to express quantity in adjective
clause with expressions like: {some of; many of; most of; none of; two
of; half of; etc}
I have ten friends. Most of them are from Sana'a. = Some of whom
are from Sana'a.
I have ten books; most of them are on history. = Most of which on
history.
I have ten books; one of my books is on history. = One of whose
books on history.
Other usage of (why, when, where, which)
Do you know the time when he came?
This is the reason why he hates me.
2.26.3.
Do you know the town
where he went to?
2.26.4.
Tom was late. That
surprised me.
The cost of this book is to pray for me you have the permission to print it or do whatever you want

Get to the heart of English language grammar in easy way

2.26.5.
surprised me.
2.26.6.
This is too bad.
2.26.7.
which is too bad.

162 of 195

Tom was late which


The elevator is out of order.
The elevator is out of order,

2.27) Do the following example:


She is the woman. I gave the money to her.
The person is the manager. I complained to him.
This is the pan. I boil the milk in it.
The agency is bankrupt. We bought our tickets from it.
{NOTE} In the above-mentioned we noted that (Wh-question) was
used in the middle of the sentence to connect two clauses as a relative
pronoun but we should know that there are a similar form use but for
indirect question; as,
I asked who was speaking.
I dont know who is there.
Tell me what have you done.
Ask what he wants.
Say which you like best.
I want to know where he went.
Do you know where I can find him?
Could you tell me when he comes.

The cost of this book is to pray for me you have the permission to print it or do whatever you want

Get to the heart of English language grammar in easy way

163 of 195

CONJUNCTIONS
2)

Conjuction: It is a word which merely joins together sentences or


sometimes word:
Example: (1) I lent Nader and Yahya some money.
This is a short of saying (I lent Nader some money and I lent Yahya some
money.)
(2) Nader and Yahya are poor but honest.
(Note) some times the conjunction (and) joins words only; as,
Two and two make four.
Fathi al-Matari and Nabil Amer are friends
3) classes of conjunctions:
1)Coordinating Conjunction 2) Subordinating Conjunction
4)

1)Coordinating Conjunction: they are two clauses of equal


rank or importance. e.g. Birds fly and fish swim.

The chief Coordinating Conjunctions are (and, but, for, or, nor, also,
eitheror, neither nor.)
5) Coordinating Conjunctions are of four kinds:o Cumulative or Completive which merely add one statement
to another; as,
we carved not a line, and we raised not a stone.
o Adversative which express opposition or contrast between
two statement; as,
He is slow, but he is sure.
I was annoyed, still I kept quiet.
I would come; only that I am engaged.
He was all right; only he was fatigued.

The cost of this book is to pray for me you have the permission to print it or do whatever you want

Get to the heart of English language grammar in easy way

164 of 195

o Disjunction or Alternative which express a choice between


two alternative; as,
She must weep, or she will die.
Either he is mad, or he feigns madness.
Neither a borrower, nor a lender be.
They toil not, neither do they spin.
Walk quickly, else you will not overtake him.
o Illative which express an inference; as,
Something certainly fell in: for I heard a splash.
All precautions must have been neglected, for the plague
spread rapidly.
He did not study hard, so he failed.
He came late. Therefore, he missed the bus.
He came late. He, therefore, missed the bus.
He came late. He missed the bus, therefore.
He came late. Consequently, he missed the bus.
6)

2) Subordinating Conjunction: It is the sentence contains two


statements or clauses one of which depends on the other and
this one called subordinating clause; as,
I read the paper because it interests me.

7)

The chief subordinating conjunctions are( after, because, if,


that, though, although, till, before, unless, as, when, where,
while, than)

8)

Subordinating Conjunctions may be classified according to


their meaning, as follows:-

1) Purpose

2) Result Or
Consequence

5) Manner

6) Time

3) Contrast
Or
Concession
7) Condition

4) Comparison
8) Cause

9) 1) PURPOSE:

1-1) Present tense+(that/so that/ in order that) subject + may +


infinitive.
He works hard that he may pass.
we eat so that we may live.
He works hard so that he may pass.
He works hard in order that he may pass.
o
2-1) Past tense +(that/ so that/ in order
that ) subject + might + infinitive.
The cost of this book is to pray for me you have the permission to print it or do whatever you want

Get to the heart of English language grammar in easy way

165 of 195

He worked hard in order that he might pass.


He worked hard in order that he might pass.
2. He works hard in order that he might pass.

3-1) Present/past tense + (lest)+ subject + (should/ could) + infinitive.


He held my hand lest I should fall.
He works hard lest he should pass.
He worked hard lest he should pass.
4-1) Present/past tense + (to/ in order )+infinitive.
He works hard to pass.
He works hard in order to pass.
DO THE FOLLOWING EXERCISE:
The police are making an investigation.
They want to arrest the swindler.
I am wearing heavy clothes. I dont want to catch a clod.
I practice in the morning. I want to get healthier.
She ran after her husband. She wanted to stop him from
leaving.
10)

2) RESULT:
Subject+ be+ so + adjective+ that.
He is clever. He can answer any question.
He is so clever that he can answer any question.
o So + adjective +be+ subject+ that.
So clever is he that he can answer any question.
Subject+ be+ such +a/an+ adjective+ noun+ that.
H e is a clever man. He can answer any question.
He is such a clever boy that he can answer any question.
4-2) Such+ a/an+ adjective+ be+ subject+ that
Such a clever boy is he that he can answer any question.
o Subject +be +adjective+ enough to+ infinitive

He is strong. He can lift that box.


He is strong enough to lift that box.
(or) he has enough strength to lift that box.
6-2) With negative S+ be+ too+ adj + to+ infinitive
He is not clever. He cannot answer all questions.
The cost of this book is to pray for me you have the permission to print it or do whatever you want

Get to the heart of English language grammar in easy way

166 of 195

He is not too clever to answer all the questions.


7-2) With different subject we should use (for)
The bag was heavy. The porter could not carry it.
The bag was too heavy for the porter to carry
DO THE FOLLOWING EXERCISE:
The story is interesting. I decided to read it.
The problem is very difficult. I can not solve it.
My brother was very happy. He danced.
Osama is a very clever boy. He got the 1st honor degree.
We met a very ugly man. We could not talk to him.

3) Concession/ Contrast:
1-3) Though/ although:
Although he is rich, he is unhappy.
He is unhappy though he is rich.
a)

However+ adjective/ adverb:


However rich he is, he is unhappy.

b)

Whatever+ verb/noun:
Whatever money he has, he is unhappy.
However money he has, he is unhappy

c)

Despite /In spite of + gerund/noun:


In spite of having some money, he is unhappy.
In spite of being rich, he is unhappy.
He is unhappy despite he is rich.
In spite his richness, he is unhappy.
4-3) Yet/but:
O I want to buy a car but I have no money.
O He is rich, yet he is not happy.

DO THE FOLLOWING EXERCISE:


The chair is dirty. I will sit on it.
He tried hard. He did not succeed.
He played well. He is hurt.
The cost of this book is to pray for me you have the permission to print it or do whatever you want

Get to the heart of English language grammar in easy way

167 of 195

6) Cause:
1-6) Because/ since/ as/ for:
He ate. He was very hungry.
He ate because he was hungry.
He ate since he was hungry.
He ate, as he was hungry.
He ate for he was hungry.
oDue to/ on account of/ owing to+ a noun:
He drank owing to his thirst.
He drank on account of his thirst.
He drank due to his thirst.

3-6) Because of+ noun


He drank because of his thirst.
Because of being thirsty, He drank.
4-6) Being+ adjective+..
Being thirsty he drank.
DO THE FOLLOWING EXERCISE:
The horse could not walk fast. It was raining.
You are impolite. I am not going to talk to you.
7) Time:
1-7) When /as soon as/on:
When the teacher entered the room, he sat down.
As soon as the teacher entered the room, he sat down.
On the teachers entering the room, he sat down.
2-7) As/while + was/were:
While I was walking in the street, it was raining.
As I was walking in the street, it was raining.
3-7) Since:
I have lived here since I was child.
4-7) Before/ after:
Before I entered, I had knocked at the door.
After I had knocked at the door, I entered.
The cost of this book is to pray for me you have the permission to print it or do whatever you want

Get to the heart of English language grammar in easy way

168 of 195

5-7) No sooner hadthan


No sooner had he gone, than I came.
6-7) Hardly hadthan
Hardly had he gone, than I came.
7-7) Scarcely had..when
Scarcely had he gone, when I came.
classify according to their meaning:
o After the shower was over, the sun shone out again.
o A books a book, although, there is nothing in it.
o As he was not there, I spoke to his brother.
o He ran away because he was afraid.
o Answer the first question before you proceed further.
o You will pass if you work hard.
o Sentinels were posted lest the camp should be taken by
surprise.
o Since you say so, I must believe it.
o Tell them that I will come.
o He finished first though he began late.
o Will you wait till I return?
o He will not pay unless he compelled.
o I waited for my friend until he came.
o When I was younger, I thought so.
o I dont know whence he comes.
o He found his watch where he left it.
o I dont understand how it all happened.
o I will go whither fancy leads me.
o I know not why he left us.
o I like you better than he likes you.
o I will not see him, though he comes.
o Though he slay me, yet will I trust him.
o Grievances can not be redressed unless they are known.
o He was so tired that he could scarcely stand.
o My strength is as the strength of ten, because my heart is pure.
o Since you wish it, it shall be done.
o He may enter, as he is a friend.
o You will not succeed unless you work hard.
o We arrived after he had gone.
o I waited till the train arrived.
o Bread and milk is wholesome food.
The cost of this book is to pray for me you have the permission to print it or do whatever you want

Get to the heart of English language grammar in easy way

o
o
o
o
o
o
o
o
o
o
o
o
o
o
o
o
o
o
o
o
o
o
o
o
o

169 of 195

You will get the prize if you deserve it.


When you are called, you must come in at once.
Do not go before I come.
I can not give you any money because I have none.
Since you say so, I must believe it
He fled lest he should be killed.
I will be vexed if you do that.
We got into the port before the storm came on.
He was sorry after he had done it.
I did not come because you did not call me.
My grandfather died before I was born.
I will stay until you return.
Catch me if you can.
Is that story true or false?
You will be late unless you hurry.
He asked whether he might have a holiday.
Give me to drink; else I will die of thirst.
If I feel any doubt, I will ask.
He deserved too succeed, for he worked hard.
He will be sure to come if you invite him.
We can travel by land or water.
Either you are mistaken, or I am.
I will go whether you come or not.
Unless you tell me the truth I will punish you.
I hear that your brother in London.

Join each pair of the following sentences by means of a suitable


Conjunction. Make such changes as are necessary: My brother is well. My sister is ill.
He sells mangoes. He sells oranges.
He did not succeed. He worked hard.
Ameen played well. Ahmed played well.
I honor him. He is a brave man.
You may go. I will stay.
Ali reads for pleasure. Nabeel reads for profit.
We decided to set out. It was late.
He was poor. He was honest
He is not a knave. He is not a fool.
Rustum made twelve runs. He was caught at the wicket.
He is rich. He is not happy.
The sheep are grazing. The oxen are grazing.
He is poor. He is contented.
This mango is large. This mango is sweet.
My brother was not there. My friend was not there.
The cost of this book is to pray for me you have the permission to print it or do whatever you want

Get to the heart of English language grammar in easy way

170 of 195

The boy is here. The girl is here.


The piper played. The children danced.
You must be quiet. You must leave the room.
He sat down. He was tired.
Majed works hard. Kamal is idle.
I lost the prize. I did my best.
I like him. He is dangerous.
I went to shop. I bought a slate.
He is slow. He is sure.
I know. He does not think so.
You are tall. My brother is taller.
He must start at once. He will be late.
I will sit still. I will listen to the music.
Rami did not come. He did not send a letter.
He must be tired. He has walked ten kilos.
It is autumn. The leaves are falling.
I will bring your umbrella. You wish it.
He remained cheerful. He has been wonderful.
He went out. The train stopped.
I came. I was unwilling.
Men have fought for their country. Men have died for their country.
He was afraid of being late. He ran.
The boy is dangerously ill. The boys head was hurt.
The old man fell down the steps. He broke his leg.
He tried to get up. He could not.
We went early to the circus. We could not get a seat.
He must do as he told. He will be punished.
The prisoner fell down on his knees. The prisoner begged for mercy.
Hossam may be in the house. Rami may be in the garden.
{Inversion}
a) Question:
Can I help you?
b) Exclamation:
Am I mad!?
c) With May:
May Allah help you?
d) After (so, either, nor); as,
Neither do I. So do I.
e) After (as, so, than)
So beautiful was she looks that every one admires her.
She is so shy as are her sisters.
The cost of this book is to pray for me you have the permission to print it or do whatever you want

Get to the heart of English language grammar in easy way

171 of 195

f) Conditional:
Were he with us now, we would speak to him.
Had I know his address, I would have written to him.
g) With negative adverb at the beginning; as,
At no time was the dean aware of what was happening.
Under no circumstances can I help you.
Rewrite each of the following sentences so as to place the italicized item at
the beginning of the sentence:
e.g. he sleeps so deeply that even a loud noise cannot wake him
up.
So deeply does he sleep that even a loud noise cannot wake him.
16 He spoke so simply and clearly than even a small
child could understand what he said.
17 He had hardly come out of the house when the roof caved
in.
18 He is in no respect superior to the other applicants.
19 They not only robbed him of all his money but also
assaulted him.
20 It rained so heavily that day that we found it impossible
to go out.
21 He did his work so efficiently that everybody in his
department started admiring him.
22 He presented his point of view so badly that people
present at the meeting had to ask him to stop speaking.
23 He behaved towards her so rudely that she had to
report the matter to the Vice-Chancellor.
24 A university teacher should under no circumstances
be allowed to take an active part in politics.
25 His home was so far away from the collage that he had to
take a taxi every day.
26 You can keep your friends happy only in this way.
27 He started shouting so loudly that the Vice-Chancellor had
to come out of his room and ask him to leave the place.
28 If our chairman were to resign, it would be impossible
for us to find another equally good person for that
post.
29 If he had informed you earlier, things would not have
been so bad.
30 His senior officer troubled him so much that he decided
to resign.
31 They teased that little girl to such an extent that she
started crying.
The cost of this book is to pray for me you have the permission to print it or do whatever you want

Get to the heart of English language grammar in easy way

32
33
34

172 of 195

He had scarcely started speaking when the microphone


went dead.
He behaved so nicely that day that we all started liking him.
He acted so well yesterday that people kept on clapping
for half an hour.

Join the following sentences using:


He went to school. He learns. (to/ in order to)
She failed. She did not study hard. (Since)
She failed. She studied hard. (though/although)
The thief ran away. The police catch him. (lest)
We take a lot of exercise. We may get healthier.(So that)
The army marched quickly. They wanted to surprise the enemy. (To/in order
to)
Ali is ill. He cannot go to school. (sothat)
He is a good player. He wins the match. (such.that)
The book is cheep. I can buy it. (sothat)
The lake is very wide. I cannot swim across it. (tooto)
He is strong. He can defeat any one. (enough to)
He wrote books. He was not popular. (Whatever)
He is poor. He is happy. (However)
ORDINAL NUMBERS
NUMBER
FIRST
THIRD
Fifth
Twentieth
Thirteenth

SHORT
1st
3rd
5th
20th
13th

NUMBER
SECOND
Fourth
Ninth
Twenty first
Hundredth

SHORT
2nd
4th
9th
21st
100th

ORDINARY WORDS
1)
2)
3)
4)
5)
6)

First
Second
After that
Then
Next
Finally

Firstly
Secondly
Thirdly
Fourthly
Fifthly
Sixthly

The cost of this book is to pray for me you have the permission to print it or do whatever you want

Get to the heart of English language grammar in easy way

173 of 195

ADVICE
2

Words used with advice:


{Ought to/ had better/ should better}
You ought to see a doctor.
You had better see a doctor.
You should see a doctor.
You ought not go there. Or you ought not to go there.
You had better not go there.
You should not go there.

No
1)
2)

Affirmative
I have some money
I have some thing in my hand.

3)

Osama and I do our duties

4)

Either Manal or Maram is


studying
Both Maram and Manal are
studying
He always eats fast.
All the pupils passed
I am happy. Me too.
I am sad. So am I.
He is as clever as me.
We went a long away.
I have just found it.
You must be our teacher.
Need we go? Yes, we must.

5)
6)
7)
8)
9)
10)
11)
12)
13)
14)

Negative
I have no money
I have not any thing in my
hand
Neither Osama nor I do our
duties.
Neither Manal nor Maram is
studying
Neither Manal nor Maram is
studying.
He never eats fast.
Not all the pupil passed
I am no happy. Me either.
I am not sad. Neither am I.
He is not so clever as me.
We did not go far away.
I have not found it yet.
You can not be our teacher.
No, we need not.

The cost of this book is to pray for me you have the permission to print it or do whatever you want

Get to the heart of English language grammar in easy way

174 of 195

Linking words
3
as,

There are many words used to link sentences such


Either. Or
NeitherNor
Not Only.
But Also

And
But

Because
Although

So

Though

However
BothAnd

1) The boss is flying to Paris. His secretary is flying to Paris.


The boss and his secretary are flying to Paris.
3) The boss is flying to Paris. His secretary is not flying to
Paris.
The boss, but not his secretary is flying to Paris.
4) The boss may be flying to Paris. His secretary may be
flying to Paris.
Either the boss or his secretary is flying to Paris.
5) The boss is not flying to Paris. His secretary is not flying to
Paris.
Neither the boss nor his secretary is flying to Paris.
6) I met Jane. I met her husband.
I met Jane and her husband.
=I met both Jane and her husband.
7) I did not meet Jane. I did not meet her husband.
I did not meet either Jane or her husband. = I met
neither Jane nor her husband.
14.6.1.1.
The cost of this book is to pray for me you have the permission to print it or do whatever you want

Get to the heart of English language grammar in easy way

175 of 195

It was cold. It was wet.


It was cold and wet.
14.6.1.2.
It was not cold. It was not wet.
It was not cold or wet.
14.6.1.3.
There is a post office. There is a hospital.
There is a post office and hospital.
14.6.1.4.
Osama is playing. He is laughing.
Osama is playing and laughing.
Osama is not only playing but also laughing.
14.6.1.5.
He
washed the car. He polished it.
He washed his car and polished it.
He washed his car and then polished it.
He did not only wash his car, but also polished it as well.
14.6.1.6.
You
can wait here. I will go to fetch him.
You can wait here, and I will go to fetch him.
14.6.1.7.
washed the car. He did not polish it.
He washed the car but did not polished it.

He

14.6.1.8.
went to the doctor. He is sick.
He went to the doctor for he is sick.
He went to the doctor because he is sick.
He is sick so he went to the doctor.

He

14.6.1.9.
raining. I went out.
Although it was raining, I went out.
It was raining, though, I went out.
It was raining, I went out though.
It was raining, however, I went out.

It was

2.27.1.1.

Fill in the blanks with the suitable linking words:

The cost of this book is to pray for me you have the permission to print it or do whatever you want

Get to the heart of English language grammar in easy way

176 of 195

I got out of the car . Walked into the house.


The weather was lovely, .. we stayed in the garden.
..we went to bed, I locked all the doors.
The little boy was wet and cold, . He was not hurt.
We turned off the lights and . Left the room.
I had a bath .we played football.
I sat in the kitchen .read a book.
She worked hard .failed in all her exams.
I do some exercises .i go to work in the morning.
They took me to hospital .the crash.
We had no money .. the banks were closed.
We paid our hotel bill and . Left.
I had to walk to work.. the car would not start.
It was very cold, .. we did not go out.
1) OFFERING OR INVITATION:
0
QUESTION:
Do you want
Would you care for +
Would you like

a
noun?
an

Do you like an apple?


Would you like a cup of tea?
ANSWER:
ACCEPTANCE;

Yes, please.
Yes, thanks.

REFUSAL;

No thanks. I am not
Not thanks. I dont feel like one.

Examples:
Do you want a Pepsi?
Yes, please.
No thanks. I dont feel thirsty.
1

2) REQUEST with I:
Question;
Can/could/ may + I + infinitive , please?
Answer;
ACCEPTANCE:
1- Ok. 2- yes, all right. 3- yes, here you are.

The cost of this book is to pray for me you have the permission to print it or do whatever you want

Get to the heart of English language grammar in easy way

177 of 195

Yes, help yourself. 6- certainly. 7- yes, of course.


sure. 9- of course you can/may.

{Note} we can not use (could) in answer with refusal.


Refusal;
I am sorry. We have not got one.
I am sorry. We have not got any.
I am afraid not.
I am afraid you can not because .
I'd rather you did not.
Certainly not.
{Note} for drinking we use (have) but for things we use (borrow)
Can I have some tea?
Can I borrow your book?
2

3) REQUEST WITH YOU:


Question;
Will/would/can/could/ may + you + infinitive , please?
Answer;
ACCEPTANCE:
1) Certainly. 2) Sure. 3) I would be happy to.
4) I would be glad to.

Refusal;
I would like to, but
3
4) REQUEST with would you mind:
Question;
1) Would you mind+ if + I + past simple ?
Would you mind if I close the window? =( will it cause you
any trouble or discomfort if I close the
window?)
2) Would you mind + verb+ ing?
Would you mind closing the window? =(I dont want to
cause you any trouble or discomfort, but would
you close the window, please?
Answer;
ACCEPTANCE:
Not at all. 2) Not at all. I would be happy to.
3-no, I would be glad to.

0
4

5) SUGGESTION:

The cost of this book is to pray for me you have the permission to print it or do whatever you want

Get to the heart of English language grammar in easy way

QUESTION:
0
1
2

178 of 195

Lets + infinitive a) let's not + infinitive


Let's go to a movie. Let's not go to a movie.
Shall we/I + infinitive +..

ANSWER:
ACCEPTANCE:
0
YES, ALL RIGHT.
1
Yes, that is good idea.
REFUSAL:
0
1
2
3
4

0
1

I dont feel like it.


I am sorry. I can not. I am busy.
No, it is raining.
No, it is too late. (Cold)
No, that is not good idea.

6) PERSUASION:
QUESTION;
Why dont we + infinitive +.
We +would +infinitive
Why dont we go to a movie? +(We could go to a Movie.)
ANSWER;
ACCEPTANCE;

O.k.

REFUSAL;
2- No, thanks. (I dont like it)
7) PREFERENCE:
1) I would (I d) prefer + noun.
I would prefer coffee than tea.
2) I would prefer to+ infinitive + noun
I would prefer to drink coffee.
3) I would prefer to +infinitive +noun +to +noun.
I would prefer to drink coffee to tea.
4) I would rather +infinitive +noun +than +noun
I would rather study tonight than go to the movie.
8) ASKING PERMISSION:
QUESTION;
1) Can I +infinitive +.?
2) May I + infinitive+.?
The cost of this book is to pray for me you have the permission to print it or do whatever you want

Get to the heart of English language grammar in easy way

179 of 195

3) Will I + infinitive +.?


ANSWER;
ACCEPTANCE:
0 Yes, all right. (But dont be late.)
REFUSAL;
0 No, you can not. It is too late.
1 No, you can not. We have a lesson.
2 No, you can not. You must do your
homework.
9) ADVISABILITY:
1-9) Subject +should/ ought to/ had better + simple verb.
0
You should study hard for the exam.
B) You ought to study hard for the exam.
C) You had better study hard for the exam.
2-9) You+ should/ ought / had better + not + verb.
A) You should not study hard for the exam.
B) You ought not study hard for the exam.
C) You had better not study hard for the
exam.
{NOTE} had better is stronger than the others and often use to
Warn from possible bad sequence.
10) EXPECTATION:
A) Subject+ supposed to/ is to+ simple verb.
The game is supposed to begin at 8:00.
The game is to begin at 8:00.
B) Subject+ must be/ be to be+ object of preposition.
I must be at the meeting after five minutes. The
Meeting can not occur without me.
I am at the meeting after five minutes.
The boss ordered me. He will not accept any
excuse.
{NOTE} MUST &BE TO has close meaning but must implies the
meaning that (I think it is necessary to be there) but be to
implies that (someone else expects me to be there.)
2.27.1.2. 1) The use of { HERE &THERE}
The cost of this book is to pray for me you have the permission to print it or do whatever you want

Get to the heart of English language grammar in easy way

180 of 195

Here

+Pronoun + verb!
There
Here you are!
There he is!
There he comes!
Here

+ be+ noun.
There
Here is Nabeel.
There is your father.
Here is your shirt.
{Note} we can also say,
My brother and I are here.
2.27.1.3. 2) The use of {enough}
1-2) Can be used with countable,
I ate enough oranges.
2-2) Can be used with uncountable,
I have enough money.

2.27.1.4. 3)The use of { Mr. , Mrs. , miss. , ms.}


1-3) All of them must be used with either the full
family name;
a) This is Mr. Nabeel Amer. (Or) this is Mr. Amer.

name or

(Mr.) is used with mature boy or gentleman.


Mr. Tom Smith. = Mr. Smith. /master/
3-3) (Miss) is used with single or married woman.
Miss. Jane Johnson. /mes/
4-3) (Mrs.) is used with married woman.
Mrs. Jane Smith. /misiz/
{NOTE} Mrs. Is used with the name of the womans husband family
name.
5-3) (Ms.) is used with mature single girl./miz/
Ms. Jane Johnson.
{NOTE} if we dont know the mans name we call him (sir.) and the
woman (madam)
2.27.1.5. 4) The use of {TIME}
The cost of this book is to pray for me you have the permission to print it or do whatever you want

Get to the heart of English language grammar in easy way

181 of 195

1-4) With questions:


1) What is the time?
2) What time it is?
3) Do you have the time?
4) Can you tell me the time?
5) What do you make the time?
1

With answers:
1) It is ten oclock. Etc.
It is ten in the morning. (From 1 to 12)
It is four oclock in the after noon. (1 to 5)
It is seven oclock in the evening. (5 to 10)
It is ten at night. (10 to 12)
(9:05) It is five past nine. Or nine and five minutes.
(9:15) It is quarter past nine. Or nine fifteen.
(9:30) It is half past nine. Or nine thirty.
(9:45) It is nine forty five. or it is a quarter to ten. Or it is fifteen to ten.
It is exactly ten oclock. = Neither more nor less.
It is nearly 10 oclock. = Almost exactly.
It is about 10 oclock. =I think it is.
From one to twelve in the morning, we use (AM)
From one to twelve in the after noon, we use (PM)
My watch does not work.
Your watch is right.
Your watch is slow.
Your watch is fast.
2.27.1.6. 5) Verbs combine with an object+ adjective:
Drive me crazy/ mad/ wild.
Hold it open/ still.
Push it open.
Pull it shut/ tight.
Keep it shut/ open/ fresh/ cool.
Leave it shut/ open/clean/ dirty.
Get it open/ shut/ clean/ dry/ dirty/ wet.
Wipe it clean/ dry.
Like it hot.
Make it easy/ plain/ safe.
Find it difficult/ easy.
Paint it red/ white.
Want it raw.
Prefer it friend.
2.27.1.7. 6) The answer of;
1-6) Who is this?
a) It is me. B) This is Amer. C) It is Amer.
The cost of this book is to pray for me you have the permission to print it or do whatever you want

Get to the heart of English language grammar in easy way

182 of 195

2-6) Who is he? / Who is this man?


a) He is Mr. Amer.
b) He is my grandfather
1

What are you?


a) I am a man.
b) I am a teacher.
4-6) Whose is this house? Whose house is this?
a) It is my house.
B) It is mine.
c) It is Nabeels house. D) It is Nabeels.
5-6) Whose son is this? Whose son is Osama?
a) It is Nabeels son.
6-6) Whose wife is Jane?
It is Toms wife.

2.27.1.8. 7) The use of {and}


1-7) Addition :
We were talking and laughing. = In addition to.
2-7) Result
:
He fell heavily and broke his arm. = So.
2 Condition:
Carry this for me and I will pay you $5. = If.then.
4-7) Contrast :
Tom is 16 and still sucks his thumb. =Despite.

.27.1.9.

10) The use of {some}


1-10) It is the plural of (a & an)
I ate an apple. I ate some apples.
2-10) It can be used with both countable and uncountable nouns:
I bought some oranges.
I drank some water.
3-10) It is used with offering:
Would you like some more tea?

2.27.1.10.

11) The use of {any}


The cost of this book is to pray for me you have the permission to print it or do whatever you want

Get to the heart of English language grammar in easy way


183 of 195
0 It is used with both countable and uncountable

noun:
I dont have any money.
I dont have any pen.
2-11) It is used with negative and question:
Do you have any milk?
I did not meet any friend.
2.27.1.11. 17) Wh-question with {ever} or compound interrogative:
1-17) Whoever = anyone who.
2-17) Whatever = anything that.
3-17) Whenever = anytime that.
4-17) Wherever = anyplace that
5-17) However = in anyway that.
6-17) Whichever = any of, any one of.

(Note) Whomever is the object of meets. In American English, whomever is


rare and very formal. In British English, Whoever (not whomever) is used
as the object form: e.g., He makes friends easily with whoever meets.
Whoever wants to come is welcome.
He makes friends easily with whomever he wants.
He always says whatever comes into his mind.
You may leave whenever you want.
You can go wherever you want.
The student may dress however they please.
2.27.1.12. Complete the following using (ever) words:He is free to go anyplace he wishes. He can go he wants.
He is free to go anytime he wishes. He can go he wants.
I dont know what should I do about that problem. Do
seems best to you.
There are five fights going to Chicago every day. I dont care which
one we take. We can take .. on fits in best with your
schedule.
I want you to be honest. I hope you feel free to say is on
your mind.
. Leads a life full of love and happiness is rich.
No one can tell him what to do. He does . He wants.
I have a car. I can take you .. you want to go.
The cost of this book is to pray for me you have the permission to print it or do whatever you want

Get to the heart of English language grammar in easy way

184 of 195

He likes to tell people about his problems. He will talk to .


Will listen to him. But he bores he talks to.
I have four. Take one pleases you most.

Punctuation
1-) Full stop:
It marks the long pause at the end of a declarative sentence and
abbreviations; as,
(I am student. / Dr. /Mr. / P.M.)
2-) Comma (,) it represent the short pause.
0 It is used to separate a series of words in the same
construction; as,
England, France and Italy formed an alliance.
1

It separate each pair of words connected by (and); as,


We should be devout and humble, cheerful and serene.
High and low, rich and poor, wise and foolish, must all die.

It marks off a phrase in position; as,


I told you, Nabeel, I can not forget you.
Milton, the greatest English poet, was blind.

3-) Semicolon: (;) It represents a pause of greater importance:


He was brave, large-hearted man; and we all honor him.
To separate a series of loosen related clauses; as,
God gave her peace; her land reposed.
4-26) Colon: (:)
a) Is used to introduce a list of items; as,
The four major cities of Yemen are: Sanaa, Aden,
Hodaidah and Taiz.
The cost of this book is to pray for me you have the permission to print it or do whatever you want

Get to the heart of English language grammar in easy way

185 of 195

b) To introduce statements; as,


Islam says: love for your brother as for yourself.
c) To indicate that the statement before the colon will be elaborated,
explained, illustrated more; as,
Man is full of desires: he only loves those who can satisfy
them all.
d) To separate two contrasting statement; as,
To err is human: To forgive is divine.
5-26) Interrogative mark: (?)
It is used with direct questions; as,
What are you doing?
6-26) Apostrophe: ()
To indicate the possessive of a noun; as,
Alis book.
To indicate the omission of a letter; as,
Hes a doctor.
Multiply four 5s and two 2s.
7-26) Inverted commas: ( )
With quotations: to indicate speech; as,
He said to his son, if your teacher says, you
are clever I will reward you.
8-26) The dash: ( _ )
Is used to change our thought; as,
If my wife were alive_ but why lament the past.
To resume a scatter subject; as,
His relatives, friends, neighbours_ all of them came
to visit him.
9-26) The hyphen: (-)
Used to connect the parts of compound words; as,
Passer-by. Man-of-war. Jack-of-all-trades.
To separate prefix from proper noun; as,
Anti-America. Mid-April.
To connect fraction within a word; as,
Two-thirds =2/3

The cost of this book is to pray for me you have the permission to print it or do whatever you want

Get to the heart of English language grammar in easy way

186 of 195

2.27.1.13. 6-20) Different forms between American and British English:


Base form
Bet
Fit
Get
Quit
Burn
Dream
Kneel
Lean
Leap
Learn
Smell
Spill
Spoil

2.27.1.14.

American
Bet
Fit
Got-gotten
Quit
Burned
Dreamed
Kneeled
Leaned
Leaped
Learned
Smelled
Spilled
Spoiled

British
Bet-betted
Fitted
Got
Quitted
Burnt
Dreamt
Knelt
Leant
Leapt
Learnt
Smelt
Spilt
Spoilt

19) The use of {in, on,& at}

No

In

On

1)

Live in
Sana'a/ the
house/ room

Al-Horia Street

Got in a car

Got on a bus/
train/plane/ horse/
bike

(X)

Sail on the sea

Went at the sea

Laying on the bed

(X)

2)
3)
4)

Swam in the
sea/ pool
Laying in
bed

At
The plane refueled at
Sanaa. He is at
Home/ work
/school/play/ 26
avenue/ 25 Al-Horia
street.

The cost of this book is to pray for me you have the permission to print it or do whatever you want

Get to the heart of English language grammar in easy way

No

In

5)

They play in
the street

6)

In an hour/In
the morning/
after non/
mid night/
January/ in
2003/ 19th
century/
spring

7)

In the corner
of the room

On
I met her on the
street. Trees on both
side of the street.
On my way to
school.
On Monday
afternoon/On
time/On Sunday/
January 2003/ on
January 1st / on the
morning of January
1st

(X)

It is on page 22. /

At the bottom/top of
this page. History/
mathematics/ English/
Algebra

(X)

(X)

13)

(X)

(X)

(X)

15)

(X)

16)
17)

The closet is
in the wall.
In time (as a

At night/ midnight/
2 oclock/ the
moment/ Christmas

There is a face on
the window
(Picture)

12)

14)

(X)

(X)

10)
11)

9)

At

At the corner of AlHoria and Al-Qiada


street.

There is a
face in the
window
(Someone)
Find the
mistake in
the sentence.
The people in
the picture in
this paper.
(X)
(X)

8)

187 of 195

(X)
(X)

I heard it on the
radio. I am talking
on the phone.
The car runs on
petrol.
Who is knocking on
the door?
The picture is on
the wall.
On time (=on right

At $ 20
He drives at 80.
Keep this medicine at
26o
I can see him at
distance.

(X)

Who is standing at
the door?
(X)
(X)

The cost of this book is to pray for me you have the permission to print it or do whatever you want

Get to the heart of English language grammar in easy way

No

In
routine)
He is sitting
in the sun
There is a
coin in his
hand
The cut is in
your left
hand
He is sitting
in the
armchair
A fever of
excitement/
despair/ in
good order/in
ruins/in a
rage/ in
tears/in front
of/in a week/
in a deep
sleep/in debt/
in
difficulties/
in cash/in
ink/in pencil

18)
19)

20)

21)

22)

On
time)
He is walking in the
moon

188 of 195

At
(X)

He put the coin on


his hand

(X)

The bank is on your


left

(X)

He is sitting on the
chair/floor/ground

He is sitting at a
desk/table.

On the basis of/ on


the other hand/

Ease/first/last/end/
regular intervals/
rest/leisure/anchor/
variance

The pronunciation of (S),


/s/
Roof
Book
Lip
Cloth
Bat

2.27.1.15.

/z/
/f/
/k/
/p/
//
/t/

Bag
Can
Rub
Mouth
Car

/Iz/
/g/
/n/
/b/
//
/r/

Glass
Buzz
Inch
Page

/s/
/z/
/t/
//

The pronunciation of (S),


/t/
Stop

/d/
/p/

Rub

/Id/
/b/

Limit

/t/

The cost of this book is to pray for me you have the permission to print it or do whatever you want

Get to the heart of English language grammar in easy way

Wash
Watch
Book
Box
Laugh
2.27.1.16.

//
/t/
/k/
/s/
/f/

Buzz
Move
Drag
Breathe
Judge

/z/
/v/
/g/
//
/d/

Mend

189 of 195

/d/

Others: the following compound letters usually pronounced as;

Th
//
Wr
/r/
Sh
//
Ge
//
ght
/t/
Ing
//
-S- /z/
a-e //
Li /laI/
Tion /hn/
Wa /w/
Ture/thr/

That
Write
Ship
General
Sight
reading
These
Care
Blind
Question
War
Future

Th
//
Wh
/w/
Ch
/t/
C+e/i/y/see/
Gh
/f/
Ph
/f/
Al
/l/
i-e
/ aI /
Lk
/k/
Lood / l /
ld /d/
Sion /hn/

Aluminium

American
English
Aluminum

Axe

Ax

Calibre

Caliber

Cheque

Check

Colour

Color

Defence

Defense

Fervour

Fervor

Fibre

Fiber

Flavour

Flavor

Gaol /jail

Jail

Honour

Honor

British English

Thin
What
Child
City center cycle
Laugh
Phone
Call
Write
Walk
Blood
Would
Mission

The meaning

The cost of this book is to pray for me you have the permission to print it or do whatever you want

Get to the heart of English language grammar in easy way

Jewellery

American
English
Jewelery

Mediaeval

Medieval

Metre

Meter

Offence

Offense

Pyjamas

Pajamas

Plough

Plow

Pretence

Pretense

Programme

Program

Scepter

Scepter

Speciality

Specialty

Theatre

Theater

Traveller

Traveler

Tyre

Tire

Valour

Valor

Wagon

Wagon

American English
Anyplace

British English
Anywhere

Apartment

Flat

Attorney

Barrister

Automobile

Car

Baby carriage

Perambulator

Bar

Pub

Billboard

Hoarding

Bill

Cab

Note
A thousand
million
Guard of a good
train
Taxi

Calling card

Visiting card

Can

Tin

British English

Billion
Brakeman

190 of 195

The meaning

The meaning

The cost of this book is to pray for me you have the permission to print it or do whatever you want

Get to the heart of English language grammar in easy way

American English
Candy

British English
Sweet

Casket

Coffin

Checkroom

Cloakroom

Clerk

Shop assistant

Closet

Cupboard

Clothes-pig

Clothes-pin

Collar-button

Collar-stud
Public
convenience

Comfort station
Composition
book
Cookie

Exercise book
Small sweet cake

Corn

Maize

Cracker

Biscuit

Crazy

Mad

Crematory

Crematorium

Crib

Cot

Cuffs

Turn-ups

Custom built

Made-to-order

Cycler

Cyclist

Derby

Bowler hat

Dessert

Receptionist in
hotel
Sweet

Detour

Diversion

Diaper

Nappy

Dish-towel

Tea-towel

Divided highway

Dual carriageway

Dresser

Dressing-table

Drug store

Pharmacy

Druggist

Chemist

Dumb

Stupid

Desk clerk

191 of 195

The meaning

The cost of this book is to pray for me you have the permission to print it or do whatever you want

Get to the heart of English language grammar in easy way

American English
Elevator

British English
Lift

Engineer

Engine driver

Eraser

Rubber

Faculty

Staff

Fall

Autumn

Fat/heavy

Fat

Faucet

Water tap

Fender

Wing

First floor

Ground floor

Flashlight

Torch

Flat

Flat tire/puncture

Freeway

Motorway

Freight train

Goods train

Freshman

First-year student
at university

Gas/ gasoline

Dustbin/rubbish
bin
Petrol

Gear-shift

Gear-lever

Generator

Dynamo

Grade crossing

Level crossing

Grain

Corn

Gridiron

Football field

Grip

Suitcase

Highway

Main road

Hog

Pig

Hood

Bonnet

Hobo

Tramp

Intermission

Interval

Intersection

Crossroad

Janitor

Caretaker
Third-year
student at
university

Garbage ban

Junior

192 of 195

The meaning

The cost of this book is to pray for me you have the permission to print it or do whatever you want

Get to the heart of English language grammar in easy way

American English
Kerosene

British English
Paraffin

Legal holiday

Bank holiday

Mad

Angry

Line busy

Number engaged

Mail

Post

Mailman

Postman

Mailbox

Postbox

Math

Maths

Mean

Nasty

Motor

Engine

Movie

Film

The movies

The cinema

Muffler

Silencer

Noplace

Nowhere

One-way ticket

Single ticket

Optometrist

Optician

Overpass

Flyover

Pacifier

Dummy

Pants

Trousers

Panty-hose

Tights

Patrolman
Peck
Penitentiary

Police constable
Peep
Prison

Period

Full stop

Pitcher

Jug

Purse

Handbag

Porch
Potato chips
Private school

Veranda
Chips
Public school

Railroad

Railway

Realtor

Estate agent

Round-trip

Return ticket

Rubber

Condom

193 of 195

The meaning

The cost of this book is to pray for me you have the permission to print it or do whatever you want

Get to the heart of English language grammar in easy way

American English

British English

Schedule

Timetable

School

School/university

Sedan
Senior

Shorts
Shoulder
Sick

Saloon
Fourth-year
student at
university
Underpants
Verge
Ill

Sidewalk

Pavement

Silent partner

Sleeping partner

Sneakers

Gym shoes

Soccer

Football

Someplace
Sophomore

Station agent

Somewhere
Second-year
student at
university
Station master

Stingy

Mean

Store

Shop

Stove

Cooker

Street car
Stroller

Suspenders
Thumbtack
Traffic circle

Tram
Push-chair
Underground
/tube
Braces
Drawing-pin
Roundabout

Trailer

Caravan

Trash

Rubbish

Truck

Lorry

Truck line

Main line

Undershirt

Vest

Vacation

Holiday

Subway

194 of 195

The meaning

The cost of this book is to pray for me you have the permission to print it or do whatever you want

Get to the heart of English language grammar in easy way

American English
Vest
Windshield
Do it over

British English
Waistcoat
Windscreen
Do it again

Fill in/out a form


Protest
something
Stay home

Fill in a form

Monday through
Friday

Monday to
Friday

Protest against
Stay at home

195 of 195

The meaning

The cost of this book is to pray for me you have the permission to print it or do whatever you want

Vous aimerez peut-être aussi